You are on page 1of 194

LƯU HÀNH NỘI BỘ |3

MỤC LỤC

BẢN QUYỀN THUỘC VỀ LOVETOAN.WORDPRESS.COM

M t S Kí Hi u V T ng Và Tích ..................................................................................................4
B T Đ NG TH C HOLDER ..........................................................................................................5
M T S K THU T TH NG S D NG.....................................................................................8
1. M T S BÀI TOÁN M Đ U ..............................................................................................8
2. M T S BÀI TOÁN C B N .............................................................................................20
3. M T S BÀI TOÁN NÂNG CAO ........................................................................................58
4. K THU T Đ I BI N S .................................................................................................115
5. NÂNG CAO K THU T S D NG B T Đ NG TH C HOLDER ....................................146
B T Đ NG TH C RADON ........................................................................................................175

Nguy n Quang Huy K49 THPT Chuyên ĐHSP


LƯU HÀNH NỘI BỘ |4

- T ng hoán v .
cyc

- V i 3 bi n a, b, c : a3b = a3b + b3c + c3a .


cyc

- V i 4 bi n : a3b = a3b + b3c + c3d + d 3a .


cyc

- T ng đ i x ng.
sym

- V i 3 bi n a, b, c : ab = ab + bc + ca .
sym

- V i 4 bi n : ab = ab + ac + ad + bc + bd + cd .
sym

- Tích hoán v .
cyc

- V i 3 bi n a, b, c : (a + b) = (a + b)(b + c)(c + a ) .
cyc

- V i 4 bi n : (a + b) = (a + b)(b + c)(c + d )(d + a) .


cyc

- Tích đ i x ng.
sym

- V i 3 bi n a, b, c : (a 2 + b) = (a 2 + b)(b2 + c)(c 2 + a)(b2 + a)(c 2 + b)(a 2 + c) .


sym

- V i 4 bi n : (a + b) = (a + b)(a + c)(a + d )(b + c)(b + d )(c + d ) .


sym

Nguy n Quang Huy K49 THPT Chuyên ĐHSP


LƯU HÀNH NỘI BỘ |5

Định lý 1.1. Cho các s d ng aij ( i = 1, m, j = 1, n ). Khi đó, ta có:

m
m n n m
aij m aij .
i =1 j =1 j =1 i =1

(1.1)

Chứng minh.

Áp d ng b t đ ng th c AM - GM ta có:

a11 a21 am1 a11a21...am1


+ + ... + mm
a11 + a12 + ... + a1n a21 + a22 + ... + a2 n am1 + am 2 + ... + amn (a11 + a12 + ... + a1n )(a21 + a22 + ... + a2 n )...(am1 + am2 + ... + amn )

a12 a22 am 2 a12 a22 ...am 2


+ + ... + mm
a11 + a12 + ... + a1n a21 + a22 + ... + a2 n am1 + am 2 + ... + amn (a11 + a12 + ... + a1n )(a21 + a22 + ... + a2 n )...(am1 + am2 + ... + amn )

a1n a2 n amn a1n a2 n ...amn


+ + ... + mm
a11 + a12 + ... + a1n a21 + a22 + ... + a2 n am1 + am 2 + ... + amn (a11 + a12 + ... + a1n )(a21 + a22 + ... + a2 n )...(am1 + am2 + ... + amn )
.

C ng t ng ng v v i v các b t đ ng th c trên, ta thu đ c Định lý 1.1.

a11 a21 am1


= = ... =
a11 + a12 + ... + a1n a21 + a22 + ... + a2n am1 + am 2 + ... + amn
a12 a22 am 2
Đ ng th c x y ra khi và ch khi = = ... =
a11 + a12 + ... + a1n a21 + a22 + ... + a2n am1 + am 2 + ... + amn

a1n a2 n amn
= = ... =
a11 + a12 + ... + a1n a21 + a22 + ... + a2n am1 + am 2 + ... + amn

n
Định lý 1.2. Cho các s d ng xij ( ). Khi đó v i ki 0 ( "i = 1, n ) th a mãn ki = 1 , ta có:
i =1

kj
n m m n
k
xij xij j .
i =1 j =1 j =1 i =1

(1.2)

Chứng minh.

Nguy n Quang Huy K49 THPT Chuyên ĐHSP


LƯU HÀNH NỘI BỘ |6

m
Gi s xij = 1 ( i = 1, n ), khi đó b t đ ng th c c n ch ng minh tr thành
j =1

m n
k
1 xij j .
j =1 i =1

S d ng b t đ ng th c AM - GM ta có:

m n m n n m n m n
ki xij = ki xij = xij = ki = 1 .
k
xij j kj
j =1 i =1 j =1 i =1 i =1 j =1 i =1 j =1 i =1

Đ nh lý đ c ch ng minh .

Một số trường hợp đặc biệt thường gặp của bất đẳng thức Holder .

V i thì ta có:

(1.3)

Đ ng th c x y ra khi và ch khi .

a b c
= =
m n p
.
a b c
= =
x y z

Và khi thì ta có:

(a3 + b3 + c3 )(m3 + n3 + p3 )2 (am2 + bn2 + cp 2 )3 .

a b c
Đ ng th c x y ra khi và ch khi = = .
m n p

(B t đ ng th c Chrystal ).

Cho các s d ng aij ( i = 1, m, j = 1, n ).

Khi đó ta có:

1 + a11 + a12 + ... + a1n 1 + a21 + a22 + ... + a2 n ... 1 + am1 + am2 + ... + amn

m
1 + m a11a21...am1 + m a12 a22 ...am 2 + ... + m a1n a2 n ...amn .

(1.4)

Đ ng th c x y ra khi và ch khi a11 : a21 :...: am1 = a12 : a22 :...: am 2 = ... = a1n : a2n :...: amn = 1 .

Cho . Khi đó ta có b t đ ng th c:

a 3 b3 c 3 ( a + b + c )3
+ + .
x y z 3( x + y + z )

(1.5)

Nguy n Quang Huy K49 THPT Chuyên ĐHSP


LƯU HÀNH NỘI BỘ |7

Đ ng th c x y ra khi và ch khi .

(B t đ ng th c Radon ).

Cho ak , xk > 0 ( "k = 1, n ) và . Khi đó ta có:

n p +1

xk
n
xkp +1 k =1
p
.
k =1 akp n
ak
k =1

(1.6)

x1 x2 x
Đ ng th c x y ra khi và ch khi = = ... = n .
a1 a2 an

B n đ c xem thêm Phần 3 v b t đ ng th c Radon .

1 1
. Cho và hai s th a mãn + = 1.
p q

Ch ng minh ho c ph đ nh b t đ ng th c sau:
1 1

(a p + b p + c p ) p ( x q + y q + z q ) q max(ax, by, cz ) + max(ay, bz, cx) + max(az , bx, cy ) .

Nguy n Quang Huy K49 THPT Chuyên ĐHSP


LƯU HÀNH NỘI BỘ |8

K thu t tham s hóa-cân b ng h s .

x, y, z 0
Ví dụ 2.1. Cho . Tìm min P = x 3 + y 3 + z 3 .

Lời giải.

Áp d ng b t đ ng th c Holder ta có:
3 3
1 1 3 3
+ +1 2 2 + 3 3 +1 P
2 3

3 3
1 1 3 3
= + +1 2 2 + 3 3 + 1 x3 + y 3 + z 3
2 3

3
2x + 3y + z = 36 .

36
P = x3 + y 3 + z 3 .
36 + 4 3 + 9 2 1 + 81 3 + 16 2

a
x= 1
2a + 3b + c a=
2
b
Đ ng th c x y ra khi và ch khi y= v i 1 .
2a + 3b + c b=
3
c
z= c =1
2a + 3b + c

36
V y, min P = .
36 + 4 3 + 9 2 1 + 81 3 + 16 2

Nhận xét 2.1. Ý t ng gi i bài toán trên nh sau:

Áp d ng b t đ ng th c Holder ta có:

(amx + bny + cpz )3


P = x3 + y 3 + z 3 .
(a3 + b3 + c3 )(m3 + n3 + p 3 )

Ta hãy ch n sao cho đi u ki n đ c t n d ng tri t đ . Do đó, theo cách t


am bn cp
nhiên nh t, ta có th ch n th a mãn = = = 1 . Đ ng th i, d u b ng c a b t đ ng
2 3 1
th c Holder cũng ph i x y ra, t c là

Nguy n Quang Huy K49 THPT Chuyên ĐHSP


LƯU HÀNH NỘI BỘ |9

x y z
= =
a b c
x y z
= =
m n p

2x 3 y z 2x + 3y + z 1
= = = =
2a 3b c 2a + 3b + c 2a + 3b + c

a
x=
2a + 3b + c
b
y=
2a + 3b + c
c
z=
2a + 3b + c
2a 2 3b 2 c2
= =
2a + 3b + c 2a + 3b + c 2a + 3b + c

a
x=
2a + 3b + c
b
y=
2a + 3b + c .
c
z=
2a + 3b + c

1
a=
2
1
b=
3
T đây ta ch n đ c
c =1 .
m=2 2
n=3 3
p =1

Ví dụ 2.2. Cho th a mãn . Tìm min P = x 4 + 2 y 4 + 3z 4 .

Lời giải.

Áp d ng b t đ ng th c Holder ta có:

4 4 4 3

3 3 3
x4 + 2 y 4 + 3z 4 +2 +3
1 1 1 1 1 1
1+ 3 + 3 3
2 1+ 3 + 3 3
3 1+ 3 + 3
2 3 2 3 2 3

3 3 4
3

3 3 3 316
x+2 y+3 z = 12
.
1 1 1 1 1 1 1 1
1+ 3
+ 3
2 1+ 3 + 3 3
3 1+ 3 + 3 1+ 3 + 3
2 33 2 3 2 3 2 3

Nguy n Quang Huy K49 THPT Chuyên ĐHSP


L Ư U H À N H N Ộ I B Ộ | 10

316
12 3
1 1
1+ 3
+
2 33 3
P =3 .
4 4 3 1 1
4
1+ 3 + 3
2 3
3 3 3
+2 +3
1 1 1 1 1 1
1+ 3 + 3 3
2 1+ 3 + 3 3
3 1+ 3 + 3
2 3 2 3 2 3

3
x=
1 1
1+ 3 + 3
2 3
3
y=
Đ ng th c x y ra khi và ch khi 1 1 .
3
2 1+ 3 + 3
2 3
3
z=
1 1
3
3 1+ 3 + 3
2 3

Nhận xét 2.2. Ta có Bài toán Tổng quát cho bài toán trên.
n
Cho m, n Z + , các h ng s d ng a1 , a2 ,..., an và các s x1 , x2 ,..., xn > 0 th a mãn xi = n .
i =1

n
Tìm: min P = ai xim .
i =1

Lời giải.

Áp d ng b t đ ng th c Holder ta có:

m m -1 n -1 m

n n
n n
1 1
n j =1 n -1 aj n n j =1 n -1 aj
ai m
. ai xim ai xi
n -1 ai
i =1 i =1 i =1
ai n -1

m + n -1

n m
n
= n
. xi
1 i =1

j =1 n -1 aj

m n -1

n
= nm n
.
1
n -1 a
j =1 j

Nguy n Quang Huy K49 THPT Chuyên ĐHSP


L Ư U H À N H N Ộ I B Ộ | 11

m n -1

n
nm n
1
j =1 n -1 aj nn
P m -1
= n -1
n
1
n j =1 n -1 a j
m -1 n -1 n
1
n n -1 aj
n j =1
n
.
1 i =1
n -1 ai
j =1 n -1 aj

n
n
1
j =1 n -1 aj n
Đ ng th c x y ra khi và ch khi xi = = ( "i = 1, n ).
n -1 ai n
n -1
1
ai
j =1 n -1 aj

Ví dụ 2.3. Cho th a mãn x + y + 2 x 2 + 2 xy + 3 y 2 = 4 . Ch ng minh r ng:

3
5
x 2 y 32 -1 .
3

Michael Rozenberg

Lời giải.

Ta vi t gi thi t l i thành

2
x 2
2 +2 + y+2 = 20 .
2

Áp d ng b t đ ng th c AM - GM và b t đ ng th c Holder ta có:

2 2 2 4
x 2 x x 2 x 2
20 = 2 +2 + y+2 = +2 + +2 + y+2 33 +2 . y+2
2 2 2 2

2
x x
3 3 . . y + 3 2.2.2
2 2

2
x2 y
= 3 2+ 3 .
4

Nguy n Quang Huy K49 THPT Chuyên ĐHSP


L Ư U H À N H N Ộ I B Ộ | 12

3
5
x 2 y 32 -1 .
3

5 5
Đ ng th c x y ra khi và ch khi ( x, y ) = 4 -1 , 2 -1 .
3 3

BÀI TẬP

1. Cho . Tìm max F = x 2 + y 2 + z 2 .


x1997 + y1997 + z1997 = 3
THTT 241

Lời giải.
Áp d ng b t đ ng th c Holder ta có:
1997
1997 2
x2 = 1997 x
1997 1997
x .1.1.1.....1 31995. x1997 = 31997 .
1995

x2 3.
Đ ng th c x y ra khi và ch khi .

2. Cho và ,m n th a mãn x m + y m = x n + y n . Ch ng minh r ng:


.

Lời giải.
V i luôn t n t i các dãy s h u t an , bn sao cho
lim ak = m, lim bk = n .
k + k +

Do đó ta c n ch ng minh
v i x ak + y ak = xbk + y bk ( "k N * ).
Hay ta ch ng minh
.
+
V i và m, n Q , m n th a mãn x + y = x n + y n .
m m

A C
Đ t m= ,n = ( A, B, C , D Z + , gcd A, B = gcd C, D = 1 ).
B D
Không m t tính t ng quát, gi s m > n DA > BC .
Áp d ng b t đ ng th c Holder ta có:
A
x+ y = A x+ y
A
A B A B

=A A x B
+A y B

A A A A A A
xB + yB x B + y B ... x B + y B . 1 + 1 1 + 1 ... 1 + 1
A
A- B
B

Nguy n Quang Huy K49 THPT Chuyên ĐHSP


L Ư U H À N H N Ộ I B Ộ | 13

A A B

= A 2 A- B. x B + y B

B
A A DA - BC
A- B
=A2 . DA- BC x +yB B

B
C C DA

x +y D D

A- B
=A2 . DA- BC BC
A A
xB + yB

B
DA
A BC A BC

DA x B
+ DA y B

= A 2 A - B. DA - BC
BC
A A
xB + yB

B
A A A A A A
x +y
B B
x +y B B
... x + y
B B
. 1 + 1 1 + 1 ... 1 + 1
DA - BC

A
2 A - B. DA - BC
BC
BC
A A
xB + yB

B
A A BC
DA - BC
2 . x +y B B

A- B
=A2 . DA - BC BC
A A
xB + yB

= 2.
Đ ng th c x y ra khi và ch khi .

3. Cho . Tìm .
x3 + y 3 = 1

4. Cho . Tìm .

5. Cho . Tìm .
a+b+c = 3
6. Cho th a mãn các đi u ki n:
+
k
0<a b c d
1 2 d
+ + 3 .
a b c
2 d
+ 2
b c
Ch ng minh r ng:
.

Nguy n Quang Huy K49 THPT Chuyên ĐHSP


L Ư U H À N H N Ộ I B Ộ | 14

Lời giải.
S d ng phép nhóm Abel ta d th y:
k k k
d 2 1
d k + 2k + 1 = c k . + bk . + ak .
c b a
k k k k k k
d 2 d 1 2 d
= ck - bk . + bk - a k . + + ak . + + .
c b c a b c
Áp d ng b t đ ng th c Holder ta có:
k k k k k k k
1 2 d 1 2 d 1 2 d
+ + 3k -1 + + 3k + + 3.
a b c a b c a b c
k k k k k
2 d 2 d 2 d
+ 2k -1 + 2k + 2.
b c b c b c
Suy ra
k k k k k k
d 2 d 1 2 d
ck - bk . + bk - a k . + + ak . + +
c b c a b c
.
a =1
Đ ng th c x y ra khi và ch khi b=2 .
c = d =1

Lưu ý 2.1. Ta nh c l i v phép nhóm Abel .


Cho 2 dãy s d ng x1 , x2 ,..., xn , y1 , y2 ,..., yn .
i
Đ t Si = y j ( i = 1, n . Khi đó:
j =1
n n
xi yi = xi - xi +1 Si (Quy c xn +1 = x1 ).
i =1 i =1

Ví dụ 2.4. Cho . Ch ng minh r ng:

Lời giải.

Áp d ng b t đ ng th c AM - GM và b t đ ng th c Holder ta có:

x 2 + xy + y 2 3 6 x 2 + xy + y 2 . z 2 + y 2 + yz . zx + x 2 + z 2

3
36 3
x 2 .z 2 .zx + 3 xy. y 2 .x 2 + 3 y 2 . yz.z 2 = 3 xy + yz + zx .

Đ ng th c x y ra khi và ch khi .

BÀI TẬP
7. Cho . Ch ng minh r ng:
a6 b6 c6 abc a + b + c
+ 2 + 2 2 .
b +c c +a a +b
22 2
2

Nguy n Quang Huy K49 THPT Chuyên ĐHSP


L Ư U H À N H N Ộ I B Ộ | 15

Lời giải.
Áp d ng b t đ ng th c Holder ta có:
3 3 2
a6 a2 a2 a2
= = .
b + c2
2
b2 + c2 6 a2 6
2
a2 abc a
Ta ch ng minh . Th t v y,
6 2
2 2
a2 bc 3 ca.ab abc a
= .
6 6 6 2
Đ ng th c x y ra khi và ch khi a = b = c .

8. Cho . Ch ng minh r ng:

abc + 3 1 + a3 1 + b3 1 + c3 ab + bc + ca .

Lời giải.
Áp d ng b t đ ng th c Holder ta có: 3 1 + a3 ab + c .
Hay ta c n ch ng minh b t đ ng th c abc + c bc + ca
c a -1 b -1 0 .
Đi u này hi n nhiên đúng vì theo nguyên lí Dirichlet , trong 3 s luôn có ít nh t 2
s cùng nh h n ho c b ng 0 ho c cùng l n h n ho c b ng 0 . Không m t tính t ng quát, gi s đó
là 2 s a - 1 và b - 1 , suy ra .
Đ ng th c x y ra khi và ch khi a = b = c = 1 .

9. Cho . Ch ng minh r ng:


a + 1 a + b b + c c + 16
abc .
81
Đ ng th c x y ra khi nào ?
10. Cho th a mãn . Ch ng minh r ng:
a b + c + b 3 c2 + a 2 + c 3 a 2 + b2
3 2 2
12 .

Lời giải.
Áp d ng b t đ ng th c Holder ta có:
2
a 3 b2 + c2 = 3 a.a 2 . b2 + c 2 3 a a2 b2 + c2 = 23 a a2 .
Đ ng th c x y ra khi và ch khi a = b = c = 2 .

Ví dụ 2.5. Cho . Ch ng minh r ng:

a b c 27
3
+ 3
+ 3 2
.
b+c c+a a+b 8 a+b+c

Lời giải.

Áp d ng b t đ ng th c Holder ta có:

3
a 2 a 27
a .
b+c
3
b+c 8

Nguy n Quang Huy K49 THPT Chuyên ĐHSP


L Ư U H À N H N Ộ I B Ộ | 16

a 27
3 2
.
b+c 8 a

Đ ng th c x y ra khi và ch khi a = b = c .

BÀI TẬP
11. Cho . Ch ng minh r ng:
1 1 1 27
+ + .
a b +c b c+a c a +b 2 a +b+c
2

12. Cho . Ch ng minh r ng:


a b c a +b+c
a +b +c .
b+c c+a a +b 2
13. Cho . Ch ng minh r ng:
a +b b+c c+ a 3
a 2b + 3 b 2 c + 3 c 2 a
3 .
8 3

Lời giải.
3
3
a +b a 2b
Ta ch ng minh b t đ ng th c .
8 27
Áp d ng b t đ ng th c Holder ta có:
3 3
3
a 2b = 3
1.a.ab 3 a bc .

Bài toán quy v ch ng minh


a +b a bc
.
8 9
(2.1)
Áp d ng b t đ ng th c AM - GM ta có:
a bc
a+b a bc - abc a bc - a bc
= 9 = .
8 8 8 9
Đ ng th c x y ra khi và ch khi a = b = c .

Lưu ý 2.2. B t đ ng th c (2.1) bài toán trên:


Cho . Khi đó ta có:
a + b b + c c + a a + b + c ab + bc + ca
. . . .
2 2 2 3 3
Đ ng th c x y ra khi và ch khi a = b = c ho c và các hoán v .
Đây là 1 k t qu h t s c quen thu c hay đ c s d ng khi gi i toán.
14. Cho . Ch ng minh r ng:
4
a 4 + 3 + 4 b4 + 3 + 4 c4 + 3 4 108 a + b + c .

Nguy n Quang Huy K49 THPT Chuyên ĐHSP


L Ư U H À N H N Ộ I B Ộ | 17

Ví dụ 2.6. Cho . Ch ng minh r ng:

2 ab a+b a+b+c
8+ 3 a. . 3 a + ab + 3 abc .
a+b 2 3

Lời giải.
3
2 ab a+b a+b+c 3
Ta ch ng minh 8+ a. . 27 a + ab + 3 abc .
a +b 2 3

Áp d ng b t đ ng th c Holder ta có:

3 a a a a ab b a b c
a + ab + 3 abc 27 + + + + + + .
3 3 3 3 3 3 3 3 3

V y ta c n ch ng minh
3
2 ab a +b a +b+c a a a a ab b a b c
8+ a. . 93 + + + + + + .
a+b 2 3 3 3 3 3 3 3 3 3 3

3
2 ab a +b
8+ . 243 a + ab + b .
a +b 2

Th t v y, áp d ng b t đ ng th c AM - GM ta có:
3

2 ab
3
a +b 2 a + ab + b a+b
8+ . = 3+ 3+ .
a+b 2 a+b 2

2 a + ab + b a + b
27.9. .
a+b 2

= 243 a + ab + b .

Đ ng th c x y ra khi và ch khi a = b = c .

Ví dụ 2.7. Cho . Ch ng minh r ng:

9a a + b 6bc
39 +3 4.
2 a+b+c
2
a +b a +b+c

Lời giải.

Áp d ng b t đ ng th c Holder và b t đ ng th c AM - GM ta có:
3
9a a + b 6bc
39 +3 =
2 a +b+c
2
a +b a +b+c

3
2 2 2
2a 9 a +b 2a 9 a +b 2a 9 a +b 2b 3c
= 9 1. . + 9 1. . + 9 1. . + 3 1. .
a +b 4 a +b+c 2
a +b 4 a +b+c 2
a +b 4 a +b+c 2
a +b a +b +c

Nguy n Quang Huy K49 THPT Chuyên ĐHSP


L Ư U H À N H N Ộ I B Ộ | 18

3
2 2 2
2a 9 a+b 2a 9 a+b 2a 9 a+b 2b 3c
= 3 1. 3 .3 + 3 1. 3 .3 + 3 1. 3 .3 + 3 1. .
a +b 4 a +b+c 2
a+b 4 a+b+c 2
a +b 4 a +b+c 2
a+b a+b+c

2
2a 2b 9 a +b 3c
4 33 + 33 +
a+b a+b 4 a+b+c
2
a+b+c

2a 2b 3 a+b 3 a+b 3c
4 1+1+ + 1+ + +
a +b a +b 2 a +b+c 2 a +b+c a+b+c

= 43 .

Đ ng th c x y ra khi và ch khi a = b = c .

Nhận xét 2.3. Ta có Bài toán Tổng quát cho bài toán trên.

n
Cho a1 , a2 ,..., an > 0 . G i g n = n ai .
i =1

k
ai n
G i Ak = i =1
( "k = 1, n ). G i Gn = n Ai . Ch ng minh r ng:
k i =1

Gn g n
nn + n +1.
An Gn

IMO Shortlist 2004

Lời giải.

Ak -1
Đ t A0 = 0 . Và x1 = 1 , xk = ( ).
Ak

Ta có:

n
Ai
Gn n 2

= i =1
= n x2 x32 ...xnn -1 .
2
n
An Ann

gn n
ak n
kAk - k - 1 Ak -1 n
k - 1 Ak -1 n
=n = n = n k- = n k - k - 1 xk .
Gn k =1 Ak k =1 Ak k =1 Ak k =1

Ta c n ch ng minh

n
nn x2 x32 ...xnn -1 + n k - k - 1 xk n +1.
2

k =1

. Áp d ng b t đ ng th c Holder ta có:

n
n n x2 x32 ...xnn -1 + n k - k - 1 xk =
2

k =1

Nguy n Quang Huy K49 THPT Chuyên ĐHSP


L Ư U H À N H N Ộ I B Ộ | 19

1 2 n -1 1 2 n -1 1 2 n -1 n
n n n
= 1.x2n .x3n ...xn n + 1.x2n .x3n ...xn n + ... + 1.x2n .x3n ... xn n + n 1. k - k - 1 xk
k =2
n

1 2 k -1 n -1
n n + 1 nx2n + 2 - x2 nx3n + 3 - 2 x3 ... nxk n + k - k - 1 xk ... nxn n + n - n - 1 xn .

Áp d ng b t đ ng th c AM - GM ta có:

V i ,

k -1
nxk n + k - k - 1 xk = n n xkk -1.1n-k +1 + k - k - 1 xk

= n +1 .

1 2 k -1 n -1
n n + 1 nx2n + 2 - x2 nx3n + 3 - 2 x3 ... nxk n + k - k - 1 xk ... nxn n + n - n - 1 xn n +1.

. Áp d ng b t đ ng th c AM - GM ta có:

n n n +1 n
n2
n n x2 x32 ...xnn -1 + n k - k - 1 xk = n x1 x2 x32 ...xnn -1 + n k - k - 1 xk
2
2

k =1 k =1

1 n n +1 n
1 n
x1 + k - 1 xk + k - k - 1 xk
n 2 k =2 n k =1

n +1 1 n n +1 1 n
= + k - 1 xk + - k - 1 xk
2 n k =2 2 n k =2

= n +1 .

Nguy n Quang Huy K49 THPT Chuyên ĐHSP


L Ư U H À N H N Ộ I B Ộ | 20

Trong ph n này, ta s xét m t s d ng toán áp d ng b t đ ng th c Holder đ n gi n, m t s bài toán là


nh ng k t qu quen thu c, m t s bài toán trong các kì thi Olympic các n c và khu v c, các kì thi IMO,…

Ví dụ 2.8. Cho th a mãn x + y = 2 a + b + c . Ch ng minh r ng:

ax3 + by 3 + c 27abc .

Lời giải.

Áp d ng b t đ ng th c Holder ta có:
2
1 1 1 3
ax3 + by 3 + c + + x + y +1 .
a b c

3
x + y +1
ax3 + by 3 + c 2
1 1 1
+ +
a b c

3
abc 2 a + b + c + 1
= 2
.
ab + bc + ca

3 2
Nên ta c n ch ng minh 2 a + b + c +1 27 ab + bc + ca .

Áp d ng b t đ ng th c AM - GM ta có:

3 3
2 a + b + c +1 3 3 1. a + b + c . a + b + c

2
= 27 a + b + c

2
27 ab + bc + ca .

1
Đ ng th c x y ra khi và ch khi a = b = c = và .
3

Ví dụ 2.9. Cho . Ch ng minh r ng:

2 3
2
a 2 + ab + b2 b2 + bc + c 2 c 2 + ca + a 2 a+b b+c c+a 3 - abc 3 .

Lời giải.

B t đ ng th c t ng đ ng v i

2 1 2
abc 3 + a 2 + ab + b 2 3
a+b 3 .

3
2
a 2 + ab + b 2 a +b
1+ 3 .
a 2b 2 c 2 a 2b 2c 2

Áp d ng b t đ ng th c Holder ta có:

Nguy n Quang Huy K49 THPT Chuyên ĐHSP


L Ư U H À N H N Ộ I B Ộ | 21

3
2
a 2 + ab + b 2 a 2 + ab + b2 a+b
2
a+b
1+ 3 1+ = = .
a 2b2c 2 ab ab a 2b2c 2

Đ ng th c x y ra khi và ch khi a = b = c .

Ví dụ 2.10. Cho . Ch ng minh r ng:

Vasile Cirtoaje-Gazeta Matematica

Lời giải.
4 4 4
Ta ch ng minh r ng 24 a3 + 1 1 + abcd a2 + 1 .
a ,b ,c , d a ,b , c , d

Áp d ng b t đ ng th c Holder ta có:
4
1 + a4 1 + abcd .
a ,b , c , d

Do đó, ta ch c n ch ng minh r ng
4 4
24 a3 + 1 1+ a4 a2 +1 .
a ,b , c , d a ,b ,c ,d

4 4
Ta s ch ng minh 2 a3 + 1 1 + a4 a2 + 1 .

Th t v y, áp d ng b t đ ng th c ta có:

4
4 2 2 2 a2 +1
2 a +1 = 2 a +1
3 3
a +1 3
2 a +1 . 3
2
.
a +1

B t đ ng th c c n ch ng minh t ng đ ng v i
2 2
2 a3 + 1 a +1 1+ a4

4
a -1 0 (Đúng).

Đ ng th c x y ra khi và ch khi a = b = c = d = 1.

Ví dụ 2.11. Cho x1 , x2 ,..., x3n > 0 . Ch ng minh r ng:

n
3n
1 + xk2 3n 1
2n 1+ xk n .
k =1 1 + xk k =1

Mihaly Bencze

Lời giải.

Áp d ng b t đ ng th c Holder ta có:

Nguy n Quang Huy K49 THPT Chuyên ĐHSP


L Ư U H À N H N Ộ I B Ộ | 22

3n n 3n 3n 3n
1 1
1+ xk n = 3 1+ xk n 3 1 + xk3 .
k =1 k =1 k =1

3n
1 + xk2 3n
Bài toán quy v ch ng minh 2n 3 1 + xk3 .
k =1 1 + xk k =1

Ta s ch ng minh

1 + xk2
3
2. 3
1 + xk3 ( ).
1 + xk

3 3 4
2 1 + xk2 1 + xk3 1 + xk = 1 - xk + xk2 1 + xk .

Th t v y, áp d ng b t đ ng th c AM - GM ta có:

4 1 2 2
1 - xk + xk2 1 + xk = . 1 + xk . 1 + xk . 4 1 - xk + xk2
4

2 2 3
1 1 + xk + 1 + xk + 4 1 - xk + xk2
.
4 3

3
= 2 1 + xk2 .

Đ ng th c x y ra khi và ch khi x1 = x2 = ... = x3n = 1 .

BÀI TẬP

15. Cho th a mãn . Ch ng minh r ng:


4 4 4 2
1+ 2 2 1+ 1+ 3 a+b+c .
a +b b + c2
2
c + a2
2

Đ ng th c x y ra khi nào ?

Lời giải.
Áp d ng b t đ ng th c Holder và b t đ ng th c AM - GM ta có:
3

3
4 4 4 2
1+ 1+ 3 1+ = 27 3 a + b + c .
a + b2
2
a + b2
2
a +b2 2

3
Đ ng th c x y ra khi và ch khi a = b = c = 1 .

16. Cho th a mãn abcd = 1 . Ch ng minh r ng:


4
1 1 1 1
44 a 4 + 1 b 4 + 1 c 4 + 1 d 4 + 1 a+b+c+d + + + + .
a b c d
Gabriel Dospinescu

Lời giải.
1
Ta ch ng minh 44 a4 + 1 a+ .
a ,b , c , d a ,b , c , d a
Th t v y, áp d ng b t đ ng th c Holder ta có:

Nguy n Quang Huy K49 THPT Chuyên ĐHSP


L Ư U H À N H N Ộ I B Ộ | 23

1
4 a4 + 1 = 4 a 4 + 1 1 + b4 1 + c 4 1 + d 4 a + bcd = a + .
a ,b ,c , d a
Đ ng th c x y ra khi và ch khi a = b = c = d = 1 .

Nhận xét 2.4. Ta có Bài toán Tổng quát cho bài toán này.
n
Cho x1 , x2 ,..., xn > 0 th a mãn xi = 1 . Ch ng minh r ng:
i =1
n
n n n
1
nn xin + 1 xi + .
i =1 i =1 i =1 xi
Gabriel Dospinescu
17. Cho . Ch ng minh r ng:

x y z t
+ + + 1
2x + 3 x+ y x+ z x+t 2y + 3 y+z y+t y+x 2z + 3 z+t z + x z + y 2t + 3 t+x t+y t+z

Lời giải.
Áp d ng b t đ ng th c Holder ta có:
x x
=
x , y , z ,t 2 x + 3 x+ y x+ z x+t x , y , z ,t y z t
2x + x 3 1 + 1+ 1+
x x x
x
x , y , z ,t yzt
2x + x 1+ 3
x3
1
. =
yzt x , y , z ,t
3
3+
x
Do b t đ ng th c là đ ng b c nên không m t tính t ng quát, ta gi s .
3
3 yzt ztx txy xyz 3 3
Đ t = a, = b, = c, =d abcd = 1 .
x y z t
B t đ ng th c tr thành
1
1.
a ,b , c , d 3 + a
Ta ch ng minh Bài toán Tổng quát c a bài toán trên.
n
Cho a1 , a2 ,..., an > 0 th a mãn ai = 1 . Khi đó ta có:
i =1
n
1
1.
i =1 ai + n - 1
Romania TST 1999

B t đ ng th c c n ch ng minh t ng đ ng v i
n
ai
1.
i =1 ai + n - 1

.
. Áp d ng b t đ ng th c và b t đ ng th c AM - GM ta có:
n 2

n
ai
ai i =1

ai + n - 1 n
i =1
ai + n - 1
i =1

Nguy n Quang Huy K49 THPT Chuyên ĐHSP


L Ư U H À N H N Ộ I B Ộ | 24

n
ai + 2 ai a j
i =1 1 i< j n
= n
ai + n n - 1
i =1
n
n n -1 n n -1
ai + 2. 2 ai a j
i =1 2 1 i< j n
n
ai + n n - 1
i =1
n
ai + n n - 1
= i =1
n
= 1.
ai + n n - 1
i =1

. Áp d ng b t đ ng th c AM - GM ta có:
n n
ai ai
=
i =1 ai + n - 1 i =1
n
ai + n - 1 n ai
i =1
n -1
n
ai n
= n -1
i =1
ai n + n - 1 n aj
j i

n -1
n
ai n
=
n -1 n -1
i =1
ai n + n - 1 n -1 ajn
j i

n -1
n n
a i
n -1 n -1
i =1
a i
n
+ ajn
j i

.
. Ta s d ng ph ng pháp ph n ch ng.
n
1
Gi s > 1.
i =1 ai + n - 1
n
ai
Hay <1.
i =1 ai + n - 1
Áp d ng b t đ ng th c AM - GM ta có:
n -1 ai
= 1-
ai + n - 1 ai + n - 1
n aj
>
j i a j + n -1
aj
n -1 n -1
j i a j + n -1
ai + n - 1
= n -1 n
.
n -1
ai ai + n - 1
i =1
n
ai ai + n - 1
i =1
n -1
>1.
ai + n - 1

Nguy n Quang Huy K49 THPT Chuyên ĐHSP


L Ư U H À N H N Ộ I B Ộ | 25

n
ai ai + n - 1
1= i =1
n -1
> 1 (Vô lý).
i =1 ai + n - 1

Đ ng th c x y ra khi và ch khi x = y = z = t .

18. Cho . Ch ng minh r ng:


a) .
3 3 3 2 2 2
a b c a b c
b) 1+ 1+ 1+ 1+ 1+ 1+ .
b3 c3 a3 bc ca ab
b3c 3 c3 a 3 a 3b3 bc3 ca3 ab3
c) 1+ 1+ 1+ 1+ 1+ 1+ .
a3 b3 c3 a b c
a3 b3 c3 a b c
d) 1+ 1+ 1+ 1+ 1+ 1+ .
b3c 3 c3a 3 a 3b3 bc3 ca3 ab3
e) .

f) .
12 12 12 7 7 7
a b c a b c
g) 1+ 1+ 1+ 1+ 1+ 1+ .
b3 c 6 c 3a 6 a3b6 b4 c4 a4
a12 b12 c12 a7 b7 c7 b 3c c 3a a 3b
h) 1+ 1+ 1+ 1+ 1+ 1+ 1+ 1+ 1+ .
b8 c 4 c8 a 4 a 8b 4 b7 c7 a7 a4 b4 c4
i) .
19. Cho . Ch ng minh r ng:
a) .

b) .
20. Cho a1 , a2 ,..., an > 0 . Ch ng minh r ng:
n n
1 + ai3 1 + ai2 ai +1 Quy c an +1 = a1 ).
i =1 i =1

Bây gi , ta s xem cách s d ng b t đ ng th c Holder gi i m t s b t đ ng th c có d ng tích c a các bi u


th c 3 bi n đ i x ng.

Ví dụ 2.12. Cho . Ch ng minh r ng:

Vasile Cirtoaje-Mircea Lascu

Lời giải.

Khi thay a, b, c b i a , b , c , ta th y v trái c a b t đ ng th c ho c không đ i ho c gi m xu ng, còn v


ph i thì ho c không đ i ho c tăng lên. Do đó, không m t tính t ng quát, ta có th gi s .

3
Ta ch ng minh b t đ ng th c 3 1 - a + a2 1 + a3 + a6 .

Th t v y,
3 4
3 1 - a + a 2 - 1 + a3 + a 6 = 1 - a 2 - a + 2a 2 0.

Nguy n Quang Huy K49 THPT Chuyên ĐHSP


L Ư U H À N H N Ộ I B Ộ | 26

3 1 - a + a2 3 1 + a3 + a 6 .

3
Mà, theo b t đ ng th c Holder ta có: 1 + a3 + a6 1 + abc + a 2b 2c2 .

Đ ng th c x y ra khi và ch khi a = b = c = 1 .

Ví dụ 2.13. Cho . Ch ng minh r ng:

2
9 1 + a 4 1 + b4 1 + c 4 8 1 + abc + a 2b2c 2 .

Vasile Cirtoaje

Lời giải.

Khi thay a, b, c b i a , b , c , ta th y v trái c a b t đ ng th c không thay đ i, còn v ph i thì ho c không


đ i ho c tăng lên. Do đó, không m t tính t ng quát, ta có th gi s . Khi a = b = c , b t đ ng th c
t ng đ ng v i
3 2
9 1 + a4 8 1 + a3 + a 6 .

N u a = 0 , b t đ ng th c trên hi n nhiên đúng.

Ta xét tr ng h p a > 0 .
3 2
1 1
Hay ta c n ch ng minh 9 + a2 8 1 + a3 + .
a2 a3

1
Đ t a+ =x x 2.
a

Ta vi t b t đ ng th c c n ch ng minh v d ng
3 2
9 x2 - 2 8 x3 - 3 x + 1 0.

2
x-2 x x3 - 8 + 4 x3 - 5 + 6 x 2 0 Đúng do x 2 ).

3 2
9 1+ a4 83 1 + a3 + a 6 .

B t đ ng th c c n ch ng minh t ng đ ng v i:
3
1 + a3 + a 6 1 + abc + a 2b 2c2 (Luôn đúng .

Đ ng th c x y ra khi và ch khi a = b = c = 1 .

Nhận xét 2.5. T l i gi i Ví dụ 2.13, b n đ c hãy gi i thích cách đánh giá b t đ ng th c Ví dụ 2.12.

BÀI TẬP

21. Cho a, b, c R . Ch ng minh r ng:


.
Vasile Cirtoaje

Nguy n Quang Huy K49 THPT Chuyên ĐHSP


L Ư U H À N H N Ộ I B Ộ | 27

Lời giải.
Khi thay a, b, c b i a , b , c , ta th y v trái c a b t đ ng th c không thay đ i, còn v ph i thì ho c
không đ i ho c tăng lên. Do đó, không m t tính t ng quát, ta có th gi s . Khi a = b = c ,
b t đ ng th c t ng đ ng v i
3 3
2 1 + a2 1+ a 1 + a3 .
4
a -1 a2 + a + 1 0 Đúng .
3 3
2 1 + a2 1 + a3 . 1+ a .
Suy ra ta c n ch ng minh
3
1 + a3 1 + abc (Đúng).
Đ ng th c x y ra khi và ch khi a = b = c .

22. Cho . Ch ng minh r ng:


.
Titu Andreescu

Lời giải.
Khi thay a, b, c b i a , b , c , ta th y v trái c a b t đ ng th c ho c không đ i ho c gi m xu ng,
còn v ph i thì ho c không đ i ho c tăng lên. Do đó, không m t tính t ng quát, ta có th gi s
.
2
N u a = 0 , b t đ ng th c t ng đ ng v i b2 c 2 b - c 0 (Đúng . Do đó, không m t tính t ng
quát, ta có th gi s .
3
Ta ch ng minh b t đ ng th c 3 a 2 - ab + b2 a 6 + a3b3 + b6 .
4
3 a -b 2a 2 + 2b2 - ab
B t đ ng th c này đúng do 3 a 2 - ab + b 2 - a 6 + a 3b3 + b 6 = 0.
a 3b 3
3
3 a 2 - ab + b2 a 6 + a3b3 + b6 . Ta c n ch ng minh b t đ ng th c
3
a 6 + a3b3 + b6 a3b3 (Đúng theo b t đ ng th c Holder ).
Đ ng th c x y ra khi và ch khi a = b = c ho c và các hoán v ho c 2 trong 3 s a, b, c
b ng 0.

Nhận xét 2.6. Ta th y toán này s d ng ý t ng g n gi ng 2 bài toán trong các Ví dụ 2.11, 2.12 và
Bài tập 21.
Ta có 1 bài toán nh sau:
Cho . Ch ng minh r ng:
.
23. Cho . Ch ng minh r ng:
3
a 2 + ab + bc b2 + bc + ca c 2 + ca + ab ab + bc + ca .
Trần Quốc Anh

Lời giải.
Không m t tính t ng quát, gi s là s n m gi a và c .
. Ta có:
b-c b-a 0.
.
Do đó bài toán quy v ch ng minh

Nguy n Quang Huy K49 THPT Chuyên ĐHSP


L Ư U H À N H N Ộ I B Ộ | 28

3
a 2 + b2 + ca b2 + bc + ca c 2 + ca + ab ab + bc + ca .
Th t v y, áp d ng b t đ ng th c Holder ta có:

3
ab + bc + ca .
.
Áp d ng b t đ ng th c Holder ta có:
3
a 2 + ab + bc b 2 + ca + bc ab + ca + c 2 ab + a 3 bc 2 + c 3 b2c .

Bài toán quy v ch ng minh a 3 bc 2 + c 3 b 2c bc + ca .


Th t v y, ta có:
a 3 bc 2 + c 3 b2c - bc - ca = a 3
bc 2 - c + c 3 b 2c - bc
2
= 3 c2 a -b 3
b - 3 c + 3 b2 3
b-3c

0 (Do là s n m gi a và c ).
.
a 3 b 3 c
Đ t x3 = ,y = ,z = và b t đ ng th c tr thành
b c a
3
x3 + y 3 + 1 xy 2 .
Không m t tính t ng quát, gi s y là s n m gi a z và . Khi đó, ta có:
x x- y y-z 0.
xy + zx
2 2
x y + xyz .
2

xy + y z + zx
2 2 2
x2 y + y2 z + 1 .
Áp d ng b t đ ng th c Holder ta có:
3 3 3
xy 2 + y 2 z + zx 2 x2 y + y 2 z + 1 = 3
x3 .x 3 . y 3 + 3 y 3 . y 3 .z 3 + 3 1.1.1 x3 + y 3 + 1 .

Đ ng th c x y ra khi và ch khi a = b = c .

Ví dụ 2.14. Cho . Ch ng minh r ng:

3
a 5 - a 2 + 3 b5 - b 2 + 3 c 5 - c 2 + 3 a+b+c .

USAMO 2004

Lời giải.

Ta ch ng minh b t đ ng th c .
2
Th t v y, a5 - a 2 + 3 - a3 + 2 = a - 1 a +1 a2 + a +1 0.

a5 - a 2 + 3 a3 + 2 .

3
Theo b t đ ng th c Holder ta có: a3 + 2 = a3 + 1 + 1 a .

Đ ng th c x y ra khi và ch khi a = b = c = 1 .

Nguy n Quang Huy K49 THPT Chuyên ĐHSP


L Ư U H À N H N Ộ I B Ộ | 29

BÀI TẬP

24. Cho . Ch ng minh r ng:


3
2a 2 + bc 2b 2 + ca 2c 2 + ab ab + bc + ca .
Vasile Cirtoaje

Lời giải.
Áp d ng b t đ ng th c Holder ta có:
3
2a 2 + bc = a 2 + a 2 + bc b 2 + ca + b2 ab + c 2 + c 2 ab .
Ta khai tri n hai v c a b t đ ng th c
3 a3b3 + 3a 2b 2c 2 + 2abc a3 3 a 2b 2 bc + ca .
Áp d ng b t đ ng th c AM - GM và b t đ ng th c Schur ta có:
3 a3b3 + 3a 2b2c2 + 2abc a3 3 a3b3 + 3a 2b2c 2 + 6a 2b2c2
=3 a3b3 + 3a 2b2c 2
3
=3 ab + 3 ab . bc . ca

a 2b2 bc + ca .

Đ ng th c x y ra khi và ch khi a = b = c .

25. Cho . Ch ng minh r ng:


2
2 1 + a 2 1 + b2 1 + c 2 1 + ab + bc + ca .
Michael Rozenberg

Lời giải.
Ta ch ng minh
2 4
4 1 + a 2 1 + b2 1 + c2 1+ ab
4
4 1 + a 2 1 + b2 1+ ab
4
4 a 2b 2 + a 2 + b 2 + 1 1+ ab .
Th t v y, áp d ng b t đ ng th c Holder ta có:

4
1+ ab .

26. Cho th a mãn . Ch ng minh r ng:


.
Vasile Cirtoaje

Lời giải.
B ng cách bình ph ng 2 v đ ng th c trong gi thi t, ta có:
a 2 + 2 bc = a 4 + 2 b2c 2 .
.
V y ta c n ch ng minh a4 a 2 . Th t v y, theo b t đ ng th c Holder ta có:
2 3
a4 a2 a2 .
a4 a2

Nguy n Quang Huy K49 THPT Chuyên ĐHSP


L Ư U H À N H N Ộ I B Ộ | 30

bc b 2c 2 .
Đ ng th c x y ra khi và ch khi a = b = c ho c a, b, c = 0,0,1 và các hoán v ho c
và các hoán v .

Ví dụ 2.15. Cho . Ch ng minh r ng:

a2 b2 c2
2
+ 2
+ 2
1.
b2 + c + a c2 + a + b a2 + b + c

Lời giải.

Áp d ng b t đ ng th c Holder ta có:
2
a2 2 3
2
a b2 + c + a a .
b + c+a
2

3 2
Bài toán quy v ch ng minh a a b2 + c + a .

Th t v y, ta có:
3 2
a - a b2 + c + a =2 a 2b + 6abc 0 .

Đ ng th c x y ra khi và ch khi a, b, c = 1,0,0 .

Ví dụ 2.16. Cho . Ch ng minh r ng:

y+z z+x x+ y 4( x + y + z )
+ + .
x y z x+ y y+z z+x

Darij Grinberg

Lời giải.

Áp d ng b t đ ng th c Holder ta có:
2
y+z 2 3 3
. x2 y + z y+ z =8 x .
x

2
Ta c n ch ng minh x y+z 2 x2 y + z .

2
yz y - z 0 Đúng).

Đ ng th c x y ra khi và ch khi .

Nguy n Quang Huy K49 THPT Chuyên ĐHSP


L Ư U H À N H N Ộ I B Ộ | 31

Ví dụ 2.17. Cho . Ch ng minh r ng:

1 1 1 3
+ + .
a b +1 b c +1 c a +1 3
abc 1 + 3 abc

MOSP 2003

Lời giải.

. Ta ch ng minh b t đ ng th c
2
1 9
.
a b +1 3
abc
2
1 + 3 abc
2

2
Áp d ng b t đ ng th c quen thu c x + y + z 3 xy + yz + zx và b t đ ng th c Holder ta có:

2
1 1 3 a + bc
3 =
a b +1 ab a + 1 b + 1 abc a +1

3 3 abc + 1
= -
abc abc a +1

3 3 abc + 1
- 3
abc abc 1 + 3 abc

9
= 2
.
2
3
abc 1 + 3 abc

ky kz kx
. Đ t abc = k 3 . Khi đó sao cho a = ,b = ,c = .
x y z

B t đ ng th c c n ch ng minh t ng đ ng v i

x 3
.
y + kz k +1

Áp d ng b t đ ng th c ta có:

2 2
x x x 3
= .
y + kz k +1 yz x
2
k +1
k +1
3

. Áp d ng b t đ ng th c AM - GM ta có:

1
1 + abc +3 =
a b +1

1 + abc
= +1
a b +1

a +1 b c +1
= +
a b +1 a +1

Nguy n Quang Huy K49 THPT Chuyên ĐHSP


L Ư U H À N H N Ộ I B Ộ | 32

a +1 b c +1
= +
a b +1 a +1

a +1 b c +1
33 + 33
a b +1 a +1

3
= 3
+ 3 3 abc .
abc

3
+ 3 3 abc - 3
1 3
abc 3
= .
a b +1 1 + abc 3
abc 1 + 3 abc

Đ ng th c x y ra khi và ch khi a = b = c .

Ví dụ 2.18. Cho x1 , x2 , x3 , y1 , y2 , y3 , z1 , z2 , z3 > 0 . Tìm h ng s t t nh t đ b t đ ng th c sau đúng:

Japan 2006

Lời giải.

3
Ta ch ng minh k = là h ng s t t nh t c n tìm.
4

Áp d ng b t đ ng th c Holder và b t đ ng th c AM - GM ta có:

3
x1 + x2 + x3
3 +1
x13 + x23 + x33 + 1 3
x1 + x2 + x3 x1 + x2 + x3
3

3
3 3 3
x1 + x2 + x3 1 1 1 y + y2 + y3 1 1 1 z +z +z
3 + + +3 1 + + +3 1 2 3
3 2 2 2 3 2 2 2 3
= 3
x1 + x2 + x3
3

3
3 3 3
x1 + x2 + x3 1 1 y + y + y3 1 1 z +z +z 1 1
3 3 . . +33 1 2 . . +33 1 2 3 . .
3 2 2 3 2 2 3 2 2
3
x1 + x2 + x3
3

3
3 x1 + x2 + x3
4 3
= 3
x1 + x2 + x3
3

3
= .
4

Nguy n Quang Huy K49 THPT Chuyên ĐHSP


L Ư U H À N H N Ộ I B Ộ | 33

1
Đ ng th c x y ra khi và ch khi x1 = x2 = x3 = y1 = y2 = y3 = z1 = z2 = z3 = 3
.
6

Bây gi , ta s xét m t s bài toán trong các kì thi h c sinh gi i, Olympic các n c và khu v c trong vài
năm g n đây.

Ví dụ 2.19. Cho th a mãn a + b + c = 1 . Ch ng minh r ng:

13 a b c
ab + bc + ca 1 + 4abc 2
+ 2
+ 2
.
4 a +1 b +1 c +1

Chọn học sinh giỏi THPT Chuyên Khoa học Tự Nhiên ĐHQGHN 2011-2012

Lời giải.

Áp d ng b t đ ng th c Holder ta có:

a 2 3
2
. a a +1 a .
a +1

3 3
a 4abc a 4abc a 13
1 + 4abc 2
1+ 2
. Hay ta ch c n ch ng minh 1 + 2
bc .
a +1 a a +1 a a +1 4

4abc 13
1+ 2
1- a2 .
a +1 2 8

H n n a, theo b t đ ng th c Schur ta có:


3 2
4abc 4 4 a. bc - a 4 4 bc - 1 4 2 a -2 a2 - 1 4 1- 2 a2
1+ 2
1+ 2
= 1+ 2
= 1+ 2
= 1+ 2
a2 + 1 9 a2 + 1 9 a2 + 1 9 a2 +1 9 a2 + 1

1
Đ t a2 = t t . Ta ch ng minh b t đ ng th c
3

4 1 - 2t 13
1+ 2
1- t
9 t +1 8

3t - 1 39t 2 + 76t + 13 0 Đúng).

1
Đ ng th c x y ra khi và ch khi a = b = c = .
3

Ví dụ 2.20. Cho . Ch ng minh r ng:

1 1 1 3
+ + .
1 1 1 1
a + +1 b + +1 c + +1 3
abc + 3
+1
b c a abc

David Stoner- ELMO Shortlist 2013

Lời giải.

Nguy n Quang Huy K49 THPT Chuyên ĐHSP


L Ư U H À N H N Ộ I B Ộ | 34

1 3
Xét -
1 1
a + +1 3
abc + 3 +1
b abc

1 3
= -1 + 1-
1 1
a + +1 3
abc + 3 +1
b abc

2
abc - 1
2 3
abc - 1
=- +
ab + b + 1 3
a b c 2 + 3 abc + 1
2 2

2 3
3
abc - 1 ab + b + 1 - 3
a 2b2c 2 + 3 abc + 1
= .
ab + b + 1 3
a 2b2c 2 + 3 abc + 1

B t đ ng th c này đúng do áp d ng b t đ ng th c Holder ta có:


3
ab + b + 1 3
a 2b2c 2 + 3 abc + 1 .

rx ry rz
.Đ t . Khi đó sao cho a = ,b = , c = .
y z x

B t đ ng th c c n ch ng minh t ng đ ng v i

y 3
.
r x + ry + z
2
r + r +1
2

Áp d ng b t đ ng th c ta có:

2
y y
r 2 x + ry + z r 2 +1 zx + r y2

2
y
= 2 2
r y + r -1 zx

2
y
2
2 2 y
r y + r -1
3

3
= .
r2 + r +1

Đ ng th c x y ra khi và ch khi a = b = c .

a b c d
Ví dụ 2.21. Cho th a mãn abcd = 1 và a + b + c + d > + + + . Ch ng minh r ng:
b c d a

b c d a
a +b+c+d < + + + .
a b c d

IMO 2008 Shortlist

Nguy n Quang Huy K49 THPT Chuyên ĐHSP


L Ư U H À N H N Ộ I B Ộ | 35

Lời giải.

. Áp d ng b t đ ng th c Holder ta có:

a b c d a b c d b c d a a b c d 4
+ + + + + + + + + + + + a+b+c+ d .
b c d a b c d a c d a b d a b c

3
a b c d b c d a 4
Hay + + + + + + a+b+c+d .
b c d a a b c d
3
b c d a
+ + +
a b c d a+b+c+d
> 1.
a+b+c+d a b c d
+ + +
b c d a

. Áp d ng b t đ ng th c AM - GM ta có:

a4 1 a a b a
a= 4 + + +
abcd 4 b b c d
b4 1 b b c b
b= 4 + + +
abcd 4 c c d a
.
c4 1 c c d c
c= 4 + + +
abcd 4 d d a b
d4 1 d d a d
d= 4 + + +
abcd 4 a a b c

a b c d
Mà a +b+c+d > + + + .
b c d a

b c d a
a+b+c+d < + + + .
a b c d

. Áp d ng b t đ ng th c và b t đ ng th c AM - GM ta có:

a b c d b c d a 1 1 1 1
+ + + + + + + = a+c + + b+d +
b c d a a b c d b d a c

1 1
= a+c b+d +
ac bd
2
1 1 1
a+c b+d +
2 ac bd

1 2 2 2
a+c b+d +
2 a+c b+d ac . bd

= 2 a+b+c+d .

Bây gi , ta s đ n v i m t bài toán quen thu c.

Nguy n Quang Huy K49 THPT Chuyên ĐHSP


L Ư U H À N H N Ộ I B Ộ | 36

Ví dụ 2.22. Cho th a mãn ab + bc + ca = 1 . Ch ng minh r ng:

a 3 + a + b3 + b + c 3 + c 2 a+b+c .

Iran MO 2008

Lời giải.

. Áp d ng b t đ ng th c Holder ta có:

2
a2 3
a a2 +1 . a .
a +1
2

Do đó ta c n ch ng minh

2 a2
a 4 .
a +12

T i đây ta có nhi u h ng gi i.

. Ta có 1 b đ : N u cho thì khi đó ta có:

A A+C
.
B B +C

(2.2)

Đ ng th c x y ra khi và ch khi C = 0 ho c .

Phép ch ng minh k t qu này r t đ n gi n, xin nh ng l i cho b n đ c.

Tr l i bài toán chính.

Không m t tính t ng quát, ta gi s c = min a, b, c .

a2 a2 2abc 2 8abc
D th y 4 =4 = 4 1- . V y ta s ch ng minh a + 4.
a +12
a+b a+c b+c b+c

2
a 8abc
+ 4.
bc b+c

2
a 8abc
+ 2.
bc b+c

Áp d ng b đ trên ta có:
2 2
a
2
8abc a + c2 8abc a + c2 8abc
+ + . Nên ta ch c n ch ng minh + 2.
bc b+c c+a c+b b+c c+a c+b b+c

2
b-c a + b - 2c 0 (Đúng).

. B ng bi n đ i đ i s ta có:

2 a2 2 a2
a -4 = a -4
a +1
2
a+b a+c

Nguy n Quang Huy K49 THPT Chuyên ĐHSP


L Ư U H À N H N Ộ I B Ộ | 37

2 a2
= a -4 bc.
a+b a+c

2 2
a b -c b+c-a
=
b+c

0.

2 a2
V y, ta có: a 4 .
a +1
2

. Áp d ng b t đ ng th c Minkowski ta có:

a3 + a = a3 + a bc

= a2 a + abc

2 2 3
a2 a + 3 abc = a + 9abc

4 a. bc (Áp d ng b t đ ng th c Schur )

=2 a.

1
Đ ng th c x y ra khi và ch khi a = b = c = .
3

Lưu ý 2.3. B t đ ng th c (2.2) bài toán trên:

Cho thì khi đó ta có:

A A+C
.
B B +C

Đ ng th c x y ra khi và ch khi C = 0 ho c .

Ta ti p t c v i m t bài toán quen thu c khác: Bài b t đ ng th c IMO 2 hay còn đ c bi t t i v i tên
g i “B t đ ng th c Hàn Qu c”.

Nhận xét 2.7. Ta có Bài toán Tổng quát cho bài toán này.

Cho và 4 d 13 + 3 33 th a mãn ab + bc + ca + abc = d . Ch ng minh r ng:

a+ b+ c 8 + abc .

Ji Chen

Khi d = 4 ta thu đ c bài toán trên.

Nguy n Quang Huy K49 THPT Chuyên ĐHSP


L Ư U H À N H N Ộ I B Ộ | 38

Ví dụ 2.23. Cho . Ch ng minh r ng:

a b c
+ + 1.
a 2 + 8bc b 2 + 8ca c 2 + 8ab

Hojoo Lee-IMO 2001

Lời giải.

. Áp d ng b t đ ng th c Holder ta có:
2
a 3
a a 2 + 8bc a .
a + 8bc
2

Do đó ta ch c n ch ng minh b t đ ng th c
3
a a a 2 + 8bc .

a bc 9abc Đúng theo AM - GM ).

. B t đ ng th c c n ch ng minh t ng đ ng v i
3 3
a2 a2
1 hay 1
1.
a 3 + 8abc a + 8abc
3 2

Áp d ng b t đ ng th c Radon và b t đ ng th c AM - GM ta có:

3 3
3
a2 a 2 a a3 + 3 b+c a3 + 24abc
= = =1.
1 1
a + 24abc
3
a + 24abc
3
a3 + 24abc
a 3 + 8abc 2 a 3 + 24abc 2

. Áp d ng b t đ ng th c ta có:

2 2 2
a a2 a a a
= = .
a 2 + 8bc a a 2 + 8bc a a 2 + 8bc a . a a 2 + 8bc a a a 2 + 8bc

C n ch ng minh r ng
3
a a a 2 + 8bc .

a bc 9abc (Đúng theo AM - GM ).

4
a a3
. Ta s ch ng minh 4 4 4
.
a 2 + 8bc a +b +c 3 3 3

4 2 2
a 3
a 3 a 2 + 8bc .

4 4 4 4 2
b3 + c3 + 2 b3c 3 8a 3 bc Đúng theo AM - GM ).

Nguy n Quang Huy K49 THPT Chuyên ĐHSP


L Ư U H À N H N Ộ I B Ộ | 39

Đ ng th c x y ra khi và ch khi a = b = c .

Nhận xét 2.8. Ta có Bài toán Tổng quát và Bài toán Mở rộng cho bài toán này.

Bài toán Tổng quát.

Cho s d ng x1 , x2 ,..., xn . Đ t yi = x j . Ch ng minh r ng:


1 j n
j i

n
xi
1.
n -1
i =1 n -1 x i + nn-1 - 1 yi

Lời giải.

Áp d ng b t đ ng th c Holder ta có:
n -1
n n n n
xi
xi xin -1 + nn -1 - 1 yi xi .
i =1 n -1 xin -1 + n n-1 - 1 yi i =1 i =1

n n n n
Bài toán quy v ch ng minh xi xin + nn - n xi .
i =1 i =1 i =1

Ta có th th y b t đ ng th c này đúng.

Đ ng th c x y ra khi và ch khi x1 = x2 = ... = xn .

Bài toán Mở rộng.

Cho . Ch ng minh r ng k = 8 là h ng s t t nh t đ b t đ ng th c sau đúng :

a b c 3
+ + .
a + kbc
2
b + kca
2
c + kab
2
1+ k

Ta có 1 bài toán nh sau:

Cho . Ch ng minh r ng:

a b c
+ + 1.
8a + bc
2
8b + ca
2
8c + ab 2

BÀI TẬP

27. Cho . Ch ng minh r ng:


a b c
+ + 1.
2 2 2
a +2 b+c
2
b +2 c+a
2
c +2 a +b
2

Lời giải.
Áp d ng b t đ ng th c Holder ta có:

Nguy n Quang Huy K49 THPT Chuyên ĐHSP


L Ư U H À N H N Ộ I B Ộ | 40

a 2 3
a a2 + 2 b + c a .
2
a +2 b+c
2

3 2
Ta c n ch ng minh a a a2 + 2 b + c .

ab a + b 6abc
c ab 9abc (Đúng theo AM - GM ).
Đ ng th c x y ra khi và ch khi a = b = c .

n
28. Cho s d ng x1 , x2 ,..., xn ( n 3 ) th a mãn xi = 1 . Ch ng minh r ng:
i =1
n
xi
a) 1.
i =1 n2 - 1
x +
2
i
xi
n
xi
b) 1.
i =1 xi + n2 - 1

Lời giải.
. Áp d ng b t đ ng th c Holder . Xin nh ng l i bài t p cho b n đ c.
n
1
. Đ t xi = ( "i = 1, n ) yi = 1 .
yi i =1

B t đ ng th c tr thành
n
1
1.
i =1 1 + n2 - 1 yi
n
1
Ta s d ng ph ng pháp ph n ch ng. Gi s <1.
i =1 1 + n2 - 1 yi
n
Khi đó ta c n ch ng minh yi > 1 .
i =1

1 1 - zi2
Ti p t c đ t zi = ( "i = 1, n ) 0 < zi < 1 và yi = ( "i = 1, n ).
1 + n 2 - 1 yi n2 - 1 zi2

Nên ta ph i ch ng minh
n n n
1 - zi2 > n2 - 1 zi2 .
i =1 i =1

Th t v y, áp d ng b t đ ng th c AM - GM ta có:
n n n 2

1 - zi2 > zi - zi2


i =1 i =1 i =1

n n n
= zi - zi zi + zi
i =1 i =1 i =1

n
n2 - 1 zj . zi2 zj
n -1 n +1
1 j n 1 j n
j i j i

n n
= n2 - 1 zi2 .
i =1

Đ ng th c x y ra khi và ch khi x1 = x2 = ... = xn = 1 .

Nguy n Quang Huy K49 THPT Chuyên ĐHSP


L Ư U H À N H N Ộ I B Ộ | 41

c) (Bài toán Tổng quát).


n
xi
m 1.
i =1 xi + nm - 1
29. Cho . Ch ng minh r ng:
2
a b2 c2 3
a) + + .
a + 24b c
4 2 2
b + 24c a
4 2 2
c + 24a b
4 2 2 5
a b c
b) + + 1.
3
a3 + 26abc 3
b3 + 26abc 3
c3 + 26abc

min a + b, b + c, c + a > 2
Ví dụ 2.24. Cho th a mãn . Ch ng minh r ng:

a b c 3
2
+ 2
+ 2 2
.
b+c-a c + a -b a +b-c abc

IMO 2011 Shortlist

Lời giải.

. Áp d ng b t đ ng th c Holder ta có:

a 3
2
a3 b + c - a a2 b + c - a a2 = 27 .
b+c -a

a 27
.
b+c-a
2
a b + c - a . a2 b + c - a
3

Áp d ng b t đ ng th c Schur ta có:

27 27 9 3
= .
a b + c - a . a2 b + c - a
3
abc a . 3abc 3 a 2
abc
2
abc
2

. Vì b + c > 2 .

T ng t ta có c + a - b và a + b - c > 0 .

Áp d ng b t đ ng th c Radon ta có:
3 3
a a2 a2 27
2
= 2 2
= 2
.
b+c-a 5 5 5
a 2
b+c-a a 2
b+c-a a 2
b+c-a

5
Bài toán quy v ch ng minh a2 b + c - a 3abc .

Th t v y, áp d ng b t đ ng th c Schur ta có:
3
a2 a - b a - c 0.

5
a2 b + c - a abc a abc 3 3 a2 3abc .

Nguy n Quang Huy K49 THPT Chuyên ĐHSP


L Ư U H À N H N Ộ I B Ộ | 42

Đ ng th c x y ra khi và ch khi a = b = c = 1 .

BÀI TẬP

30. Cho . Ch ng minh r ng:


2 2 2
2 3 2 3 2 3
3a 2 + 1 + 2 1 + 3b2 + 1 + 2 1 + 3c 2 + 1 + 2 1 + 483 .
b c a
AZE JBMO TST

Lời giải.
. Áp d ng b t đ ng th c AM - GM ta có:
4
2 2 2 2
2 3 3 2 3 3 3
3a + 1 + 2 1 +
2
= 3a + 1 + 1 +
2
+ 1+ 27 3a + 1
2 3 1+ .
b b b b
4
2
3 3
V y ta ch c n ch ng minh 3a 2 + 1 3 1+ 163 .
b
2
1
3 3
3a + 1
2 3 1+ 43 .
b

Áp d ng b t đ ng th c Holder ta có:
2 2
1 1
3 3 3 3 a2
3a + 1
2 3 1+ = 3a + 1 2 3 +1 33 +1 .
b b b2
Bài toán quy v ch ng minh
a2
33 +1 43 Đúng theo b t đ ng th c Holder ).
b2
. Áp d ng b t đ ng th c AM - GM ta có:
2
2
6 a
2 3a + + 3
2 12
2 3 b b 144a
3a 2 + 1 + 2 1 + = .
b 3 3 3b
2
2 3 144a
3a 2 + 1 + 2 1 + = 483 .
b 3b

Đ ng th c x y ra khi và ch khi a = b = c = 1 .

31. Cho . Ch ng minh r ng:


a3 + 5b3 b3 + 5c3 c 3 + 5a3 2 2 2 2
+ + (a + b + c ) .
3a + b 3b + c 3c + a 3
USAJMO
32. Cho th a mãn . Ch ng minh r ng:
3
x3 y3 z3 x+ y+z
+ + .
1 + 9 xy z 1 + 9 yz x 1 + 9 zx 2 y
2 2
18
Kyiv 2006

Lời giải.

Nguy n Quang Huy K49 THPT Chuyên ĐHSP


L Ư U H À N H N Ộ I B Ộ | 43

Áp d ng b t đ ng th c Holder ta có:
3 3
x3 x x
= .
1 + 9 xy 2 z 3 1 + 9 xy 2 z 3 3 + 9 xyz x
V y, ta c n ch ng minh 3 3 + 9 xyz x 18 .
2
3xyz x 1= yz (Đúng).
1
Đ ng th c x y ra khi và ch khi x = y = z = .
3

Nhận xét 2.9. Bài toán trên là đ thi Uzbekizstan 2012, đ thi ch n đ i tuy n HSG Qu c Gia chuyên
Qu c H c Hu 2014-2015.
33. Cho th a mãn ab + bc + ca = 1 . Ch ng minh r ng:
3 3 3 1
4 + 6 3b + 4 + 6 3c + 4 + 6 3a .
a b c abc
Hong Kong National MO 2013

Lời giải.
Áp d ng b t đ ng th c Holder ta có:
4
3 3 3
+ 6 3b 1+1+1 4 + 6 3b .
a a

3 1
V y ta ch ng minh 27 + 6 3b 4
hay
a abc

1 3 1 1
3
+ 6 3b 4
(Do 27 3
).
3 3 abc a abc 3 3 abc
1 1 1
6b + . B t đ ng th c này đúng, b n đ c hãy t ch ng minh.
3 abc a
1
Đ ng th c x y ra khi và ch khi a = b = c = .
3

34. Cho th a mãn ab + bc + ca = 3 . Ch ng minh r ng:


1 1 1 34 3
+ 2b + 4 + 2c + 4 + 2a
4 .
a b c abc
35. Cho th a mãn ab + bc + ca = 1 . Ch ng minh r ng:
1 1 1 1
3 +b + 3 +c + 3 +a .
a b c abc
IMO 2004 Shortlist
36. Cho th a mãn ab + bc + ca = 3 . Ch ng minh r ng:
3 3 3
a+b b+c c+a
+ + 12 .
3 2 a + b a 2 + b2 3 2 b + c b2 + c2 3 2 c + a c2 + a2
Korea 2013

Lời giải.
. Áp d ng b t đ ng th c Holder ta có:
3 3
a +b
3
a+b 8 c
=
3 2 a + b a 2 + b2 3 3 2 a + b a 2 + b2 3 3 2 a + b a 2 + b2

Nguy n Quang Huy K49 THPT Chuyên ĐHSP


L Ư U H À N H N Ộ I B Ộ | 44

3
8 c
2 + a + b + a 2 + b2
3
8 c
= .
2 3+ c+ c2
Do đó ta c n ch ng minh
3
c 3 3+ c+ c2 .
Ta s d ng ph ng pháp .
Vi t b t đ ng th c trên v d ng p3 3 p 2 - 2q + p + 3
p - 3 p2 - 3 0 (Đúng do q = ab = 3 ).
. Áp d ng b t đ ng th c Holder ta có:
3
3
a +b 2 4
2 a 2 + b2 a+b .
3 2 a + b a 2 + b2
3
2 4 2 4 2 4
a +b
3
a+b a+b a+b
= = .
2 a 2 + b2
2 2
3 2 a + b a 2 + b2 2 a + b - 4ab 2 a + b - 12
2
Đ t a+b = t , b t đ ng th c c n ch ng minh t ng đ ng v i
4
t
123 t - 2 t 3 + 12t 2 + 144t - 1728 0 Đúng).
2t - 12
3
a+b
. Ta ch ng minh b t đ ng th c 4ab .
3 2 a + b a 2 + b2
8
Hay a +b 128a3b3 a 2 + b 2 .
Th t v y, ta d th y b t đ ng th c này đúng:
4
a +b 2 8
128a3b3 a 2 + b2 = 64a 2b2 . 2ab a 2 + b2 64a 2b 2 . = 4ab a + b a +b .
4
3
a+b
T đó ta thu đ c 4 ab = 12 .
3 2 a + b a 2 + b2
10 2
. Ta s d ng b t đ ng th c a + b 44 a 3b3 a 2 + b2 (Phép ch ng minh xin đ c dành l i
cho b n đ c).
3
a +b
2 ab a + b
3 2 a + b a2 + b2
3
a +b
2 ab a + b 4 ab = 12 .
3 2 a + b a 2 + b2

Đ ng th c x y ra khi và ch khi a = b = c = 1 .

37. Cho th a mãn x 2 y 2 + y 2 z 2 + z 2 x 2 = 6 xyz . Ch ng minh r ng:


x y z
+ + 3.
x + yz y + zx z + xy
MMC 2013

Lời giải.

Nguy n Quang Huy K49 THPT Chuyên ĐHSP


L Ư U H À N H N Ộ I B Ộ | 45

yz zx xy x 1
Đ t a= ,b = ,c = a + b + c = 6 và = .
x y z x + yz 1+ a
Khi đó áp d ng b t đ ng th c Holder ta có:
2 2
1 1
9 = 1+ a . 27 .
1+ a 1+ a
Đ ng th c x y ra khi và ch khi .

1 1 1
38. Cho th a mãn a + b + c = + + . Ch ng minh r ng:
a 2 b2 c 2
2(a + b + c) 3
7a 2b + 1 + 3 7b2c + 1 + 3 7c 2a + 1 .
Middle European Mathematical Olympiad 2013

Lời giải.
Áp d ng b t đ ng th c Holder ta có:
3
3
7 a 2b + 1
3
7a b + 1 =
2 3 .a.a
a2

7 a 2b + 1 2 1 2
c = 7 + b c
a2 b2
3
= 2 b
3
7 a 2b + 1 2 b
Đ ng th c x y ra khi và ch khi a = b = c = 1 .

1 1 1 1
39. Cho th a mãn a + b + c + d = + + + . Ch ng minh r ng:
a 2 b2 c2 d 2
2 a +b+c + d 3
a 3 + 7 + 3 b3 + 7 + 3 c 3 + 7 + 3 d 3 + 7 .
40. Cho th a mãn abc = 1 . Ch ng minh r ng:
2 a +b +c 3
a 3 + 7 + 3 b3 + 7 + 3 c 3 + 7 .
Vasile Cirtoaje
1 1 1
41. Cho th a mãn + + = 3 . Ch ng minh r ng:
a b c
1 1 1 3
+ + .
a3 + b b3 + c c3 + a 2
Baltic Way 2014

Ta có các bài toán sau:


1 1 1
Cho th a mãn
+ + = 3 . Ch ng minh r ng:
a b c
1 1 1 3
a) 2 + + .
a + b b2 + c c2 + a 2
1 1 1 3
b) 3 + + .
a + b b3 + c c 3 + a 2
a b c 3
c) 3 + 3 + 3 .
a +b b +c c +a 2

Ví dụ 2.25. Cho th a mãn a + b + c = 3 . Ch ng minh r ng:

a2 a + b a + c + b2 b + c b + a + c2 c + a c + b 6.

Nguy n Quang Huy K49 THPT Chuyên ĐHSP


L Ư U H À N H N Ộ I B Ộ | 46

Lời giải.

. Áp d ng b t đ ng th c Holder ta có:

2
a 3
a2 a + b a + c a = 27 .
a +b a +c

27
a2 a + b a + c .
a
a +b a +c

a 3 9
Do đó, ta c n ch ng minh = .
a +b a +c 4 4 a

2
a b-c 0 Đúng .

. Ta ch ng minh b t đ ng th c:
2
3 a a +b a +c 2 a .

3 2 2
a a+b + a+c - a+b - a+c 2 a
2

3 2
a2 - bc a a+b - a+c
2
2
2 a b-c
b-c 3 2
a+b + a+c

3a 2
1- 2
b-c 0.
a +b + a +c

Th t v y,

3a 2 3a 2 1 2
1- 2
b-c 1- 2
b-c = b-c 0.
a+b + a+c a+ a 4

Đ ng th c x y ra khi và ch khi a = b = c = 1 .

n
Ví dụ 2.26. Cho n 3 , x1 , x2 ,..., xn > 0 th a mãn xi2 = n . Ch ng minh r ng:
i =1

x13 x23 xn3 n


+ + ... + .
x22 + x32 + ... + xn2-1 x32 + x42 + ... + xn2 x12 + x22 + ... + xn2- 2 n-2

Lời giải.

Nguy n Quang Huy K49 THPT Chuyên ĐHSP


L Ư U H À N H N Ộ I B Ộ | 47

Đ t x0 = xn . Khi đó ta có:

x13 x23 xn3 n


xi3
+ 2 + ... + 2 = .
x + x + ... + xn-1 x3 + x4 + ... + xn
2
2
2
3
2 2 2
x1 + x2 + ... + xn2-2
2
i =1 n - xi2-1 - xi2

Áp d ng b t đ ng th c Holder ta có:
2 4
n
xi3 n n n
xi2 n - xi2-1 - xi2 n - xi2-1 - xi2 xi2 = n4 .
i =1 n - xi2-1 - xi2 i =1 i =1 i =1

n n
2
Bài toán quy v ch ng minh n2 n - 2 xi2 n - xi2-1 - xi2 n - xi2-1 - xi2 .
i =1 i =1

Đ ý r ng:
n
n - xi2-1 - xi2 = n2 - 2n .
i =1

n n n
xi2 n - xi2-1 - xi2 = n2 - xi2-1 xi2 - xi4
i =1 i =1 i =1

n
xi4-1 + xi4
= n2 - xi2-1 xi2 +
i =1 2

n
1 2
= n2 - xi2-1 + xi2 .
2 i =1

Nên ta c n ph i ch ng minh
n
1 2
n n-2 n2 - xi2-1 + xi2 .
2 i =1

n 2
xi2-1 + xi2 4n .
i =1

B t đ ng th c này đúng vì áp d ng b t đ ng th c ta có:

n n 2
2 1
xi2-1 + xi2 xi2-1 + xi2 = 4n .
i =1 n i =1

Đ ng th c x y ra khi và ch khi x1 = x2 = ... = xn = 1 .

BÀI TẬP

42. Cho . Ch ng minh r ng:


a b
+ 1.
a + 3b
2 2
b + 3a 2
2

Lời giải.
. Áp d ng b t đ ng th c Holder ta có:
2
a b 3
+ . a a 2 + 3b2 + b b2 + 3a 2 a+b .
a + 3b
2 2
b + 3a
2 2

Nguy n Quang Huy K49 THPT Chuyên ĐHSP


L Ư U H À N H N Ộ I B Ộ | 48

3
a b a+b
+ = 1.
a 2 + 3b2 b 2 + 3a 2 a a 2 + 3b 2 + b b 2 + 3a 2
. Ta s d ng ph ng pháp ph n ch ng.
a b
Đ t x= ,y= x 2 + y 2 + 8x 2 y 2 = 1 .
a + 3b
2 2
b + 3a
2 2

2
Gi s 4 xy x+ y 1.

1
xy (Mâu thu n).
4
a b
V y hay + 1.
a + 3b
2 2
b + 3a 2
2

Đ ng th c x y ra khi và ch khi a = b .

43. Cho th a mãn abc = 1 . Ch ng minh r ng:


a b c
+ + 3.
b + 2c 2
c + 2a2
a + 2b
2

Trần Quốc Anh

Lời giải.
Áp d ng b t đ ng th c Holder ta có:
2
a 3
a b 2 + 2c a .
b + 2c
2

Bài toán quy v ch ng minh


3
a 3 a b2 + 2c .
a3 + 3 a 2b + 6 6 ab .
Th t v y, áp d ng b t đ ng th c AM - GM ta có:
a3 + c 2a + c 2a + c2a + 1 + 1 6 6 c6a 6 = 6ca .
Đ ng th c x y ra khi và ch khi a = b = c = 1 .

44. Cho . Ch ng minh r ng:


a b c a b c
+ + a+b+c + + .
a + 2b b + 2c c + 2a 2a + b 2b + c 2c + a
Vasile Cirtoaje
45. Cho th a mãn . Ch ng minh r ng:
1 1 1 3
5 5
+ 5
+ 5
.
a b+c
3
b c+a
3
c a +b
3
2
46. Cho . Ch ng minh r ng:
3
a b3 c3 d3 1
+ + + .
a+b a+c a+d b+c b+d b+a c+d c+a c +b d +a d +b d +c 2

Lời giải.
Áp d ng b t đ ng th c Holder ta có:
3
a3
a +c a +b a +d a
a ,b , c , d a +b a +c a +d a ,b , c , d a ,b , c , d a ,b , c , d

Nguy n Quang Huy K49 THPT Chuyên ĐHSP


L Ư U H À N H N Ộ I B Ộ | 49

2 3
a3
.2 a. a a
a ,b , c , d a+b a+c a+d a ,b , c , d a ,b ,c , d a ,b , c , d

a3 1
.
a ,b , c , d a+b a+c a+d 2
Đ ng th c x y ra khi và ch khi a = b = c = d .

47. Cho th a mãn abc = 1 . Ch ng minh r ng:


1 1 1 1
2
+ 2
+ 2
.
a 5 b + 2c b 5 c + 2a c5 a + 2b 3
Titu Andreescu-USA TST 2010

Lời giải.
. Áp d ng b t đ ng th c Holder và b t đ ng th c AM - GM ta có:
3
1 2 1
2
a b + 2c
a5 b + 2c a
3
= bc
2
3 bc .
2
1 3 bc
2 2
a5 b + 2c a b + 2c
2
3 bc
= 2
9 bc
1
= .
3
1 1 1
. Đ t a = ,b = , c = .
x y z
B t đ ng th c tr thành
x3 1
2
.
2y + z 3
Áp d ng b t d ng th c Radon ta có:
3
x3 x
2 2
2y + z 2y + z
x
=
9
3 3 xyz
9
1
= .
3
. Không m t tính t ng quát, ta gi s x y z xy zx yz .
Áp d ng b t đ ng th c ta có:
5 2

x2
x3 x5
2
= 2 2
.
2y + z x2 2 y + z x2 2 y + z
Bài toán quy v ch ng minh

Nguy n Quang Huy K49 THPT Chuyên ĐHSP


L Ư U H À N H N Ộ I B Ộ | 50

5 2
2
3 x2 x2 2 y + z .

5 5
3 x5 + 6 y2z2 5 y2 z2 + 4 x.
Áp d ng b t đ ng th c và b t đ ng th c AM - GM ta có:
5 5
2 1
y z =
2 2
yz . yz y2 z2 yz y2 z 2 .
3
Do đó ta ch c n ch ng minh
3 x5 + y2 z2 4 x.
Th t v y, áp d ng b t đ ng th c AM - GM ta có:
13
3x 5 + y 2 z 2 4x 4 .
13
3 x5 + y2 z2 4 x4
13
x 4
12
3
9
x 4
= 4 x
3
4 x.
Áp d ng b t đ ng th c AM - GM ta có:
x3 2y + z 2y + z x
2
+ + .
2y + z 27 27 3

x3 x
2
.
2y + z 9

Đ ng th c x y ra khi và ch khi a = b = c = 1 .

Ví dụ 2.27. Cho a, b, c là đ dài 3 c nh c a 1 tam giác. Ch ng minh r ng:

1 1 1 1
3
+ 3
+ 3
.
8abc + a + b - c 8abc + b + c - a 8abc + c + a - b 3abc

Samin Riasat

Lời giải.

B t đ ng th c c n ch ng minh t ng đ ng v i
3
a+b-c 1 1 1
= - 3
.
8abc 8abc + a + b - c
3
8abc 8abc + a + b - c 24abc

3
a +b-c 1
3
.
8abc + a + b - c 3

Áp d ng b t đ ng th c Holder ta có:
3 3
a+b-c
3
a+b-c c 1
3
= .
8abc + a + b - c
3 3
3 8abc + a + b - c 3 24abc + a +b-c 3

Nguy n Quang Huy K49 THPT Chuyên ĐHSP


L Ư U H À N H N Ộ I B Ộ | 51

Đ ng th c x y ra khi và ch khi a = b = c .

BÀI TẬP

48. Cho th a mãn abc = 1 . Ch ng minh r ng:


a3 b3 c3 1
+ + .
4 + 2b c + a + c 4 + 2c a + b + a 4 + 2a b + a + b3
2 3 2 3 2
3
49. Cho th a mãn . Ch ng minh r ng:
5
x y5 z5
+ + 1.
yz x 2 + y 2 zx y 2 + z 2 xy z 2 + x 2
George Apostolopoulos

Lời giải.
Áp d ng b t đ ng th c Holder ta có:
x5 x6
=
yz x 2 + y 2 xyz x 2 + y 2
3
x2
6 xyz x2
2
x2
=
6 xyz
2 2
x
3
2 3
x
9
.
2
Đ ng th c x y ra khi và ch khi x = y = z = .
3

Ví dụ 2.28. Cho th a mãn a + b + c = 1. Ch ng minh r ng:

a b c 3
ab + bc + ca + + .
b2 + b c2 + c a 2 + a 4

Vasile Cirtoaje

Lời giải.

Áp d ng b t đ ng th c AM - GM và b t đ ng th c Holder ta có:

a a a a 4 33 a
= - -
b +b
2
b b +1 b 44 b

a 4 33 a
= - 4 .a.a.a
b 4 b

a 4 33 a 3 a 4 33 a
- 4 a = - 4
b 4 b b 4 b

Nguy n Quang Huy K49 THPT Chuyên ĐHSP


L Ư U H À N H N Ộ I B Ộ | 52

a
+9
a b 3 a 3 a
- = + -3
b 16 4 b 16 b

3 a
.
4 b

Áp d ng b t đ ng th c ta có:

a 2
. ab a =1.
b

3 a 3
.
4 b 4 ab

1
Đ ng th c x y ra khi và ch khi a = b = c = .
3

BÀI TẬP

a b c
50. Cho th a mãn a + b + c + + . Ch ng minh r ng:
b c a
a 3c b3 a c 3b 3
+ + .
b a +c c b+a a c +b 2
Romania 2005

Lời giải.
Áp d ng b t đ ng th c Holder ta có:
a3c b 3
a+c a .
b a+c c

b b
a
2 33
a 3c a c c =3.
b a+c
2
b 2 2 2 2
c
Đ ng th c x y ra khi và ch khi a = b = c = 1 .

51. Cho . Ch ng minh r ng:


a +b 3 b+c 3 c +a a+b+c
3 + + .
a+c c+a a +b 3
abc

Lời giải.
Áp d ng b t đ ng th c Holder và b t đ ng th c AM - GM ta có:
a+b 1
3 3 3 a+b
a +c a+c
1 1 1
3 6 b +
4 a c

1
=33 b
b

Nguy n Quang Huy K49 THPT Chuyên ĐHSP


L Ư U H À N H N Ộ I B Ộ | 53

ac
=33 b .
abc
2
b
3
3 b . 3
abc
a
= 3
.
abc
Đ ng th c x y ra khi và ch khi a = b = c .

52. Cho . Ch ng minh r ng:


a +b 6 b+c 6 c +a a+b+c
6 + + .
a+c b+a c +b 3
abc

Lời giải.
Áp d ng b t đ ng th c Holder ta có:
3
a +b 1 a+b
6 3 a+b =6 +9 .
a+c a+c a+c
3
a +b a+b a+b
Vì 3 6 +9.
a+c a+c a+c
Bài toán quy v ch ng minh
a +b a
.
a+c 3
abc
Áp d ng b t đ ng th c ta có:
a+b 1
2 a .
a+c b+c
Ta c n ch ng minh
1 a
2
b+c 3 2 2 2
abc
a b+c 3
2 abc 2 2 2
a2 + 3 bc .
Áp d ng b t đ ng th c (2.1) và b t đ ng th c AM - GM ta có:
8 2 83 2 2 2 2
a b+c a bc abc a .
9 3
Do đó ta ch ng minh
4 2
a a 2 + 3 bc
3
2
a 3 bc Đúng .
Đ ng th c x y ra khi và ch khi a = b = c .

53. (Bài toán Tổng quát).


Cho và k 2 . Ch ng minh r ng:
a +b k b+c k c +a a+b+c
k + + .
a+c b+a c +b 3
abc
Võ Quốc Bá Cẩn

Lời giải.
Áp d ng b t đ ng th c Holder và b t đ ng th c AM - GM ta có:

Nguy n Quang Huy K49 THPT Chuyên ĐHSP


L Ư U H À N H N Ộ I B Ộ | 54

k
k
2
-1 a +b a+b 2
3 k
a+c a+c
k
-1
a +b a+b 2
= k k
a+c a+c
k
-1 a+b
32 k .
a+c
a+b a+b
k .
a +c a+c
Nên ta c n ch ng minh
a +b a
(Ch ng minh Bài tập 52).
a+c 3
abc
Đ ng th c x y ra khi và ch khi a = b = c .

54. Cho . Ch ng minh r ng:


b 2 + c 2 3 c 2 + a 2 3 a 2 + b2 a +b+c
3 + 2 + 2 .
a 2 + bc b + ca c + ab 3
abc
Phạm Hữu Đức
Xem Ví dụ 2.48.
55. Cho th a mãn a + b + c = 1. Ch ng minh r ng:
a b c 1
+ + .
b + 3c
2
c + 3a
2
a + 3b
2
1 + 3abc

Lời giải.
Áp d ng b t đ ng th c Holder ta có:
2
a 3
a b2 + 3c a =1.
b + 3c
2

Bài toán quy v ch ng minh


1 + 3abc a b2 + 3c .
3
a + 3abc ab2 + 3 a ab
a 3
ab 2
Đúng theo b t đ ng th c AM - GM ).
1
Đ ng th c x y ra khi và ch khi a = b = c = .
3

Ví dụ 2.29. Cho th a mãn abc = 1 . Ch ng minh r ng:

a4 - b b4 - c c4 - a
a. + b. + c. 0.
a 4 + 2b b4 + 2c c 4 + 2a

Lời giải.

B t đ ng th c t ng đ ng v i

a5 a
.
a + 2b 2 ac
4
3

Áp d ng b t đ ng th c Holder ta có:

Nguy n Quang Huy K49 THPT Chuyên ĐHSP


L Ư U H À N H N Ộ I B Ộ | 55

a5 3
a 4 + 2b2 ac a a2 .
a 4 + 2b2 ac

3 2
Nên ta ch c n ch ng minh 3 a2 a a 4 + 2b2 ac .

Áp d ng b t đ ng th c ta có:

2 2 2 2 2 3
a a 4 + 2b 2 ac a a 4 + 2abc a = a a2 3 a2 a2 =3 a2 .

Đ ng th c x y ra khi và ch khi a = b = c = 1 .

BÀI TẬP

56. Cho . Ch ng minh r ng:


2 2 2 2
3 a 2 + ab + b2 + 3 b 2 + bc + c 2 + 3 c 2 + ca + a 2 3 3 2a 2 + bc + 2b2 + ca + 2c 2 + ab .
57. Cho th a mãn a + b + c = 1. Ch ng minh r ng:
a b c 3
+ + .
b+c b c+a c a+b a 2 ab + bc + ca

Lời giải.
Áp d ng b t đ ng th c Holder ta có:
2
a 2 4
ab a b+c a .
b+c b
4 2
Ta c n ch ng minh r ng 4 a 9 a b+c .
Ch ng minh b t đ ng th c này đ n gi n, xin nh ng l i cho b n đ c.
1
Đ ng th c x y ra khi và ch khi a = b = c = .
3

58. Cho th a mãn a + b + c = abc . Ch ng minh r ng:


a2 b2 c2 3
+ + a+b+c .
a +1
2
b +1
2
c +1
2 2
59. Cho th a mãn . Ch ng minh r ng:
2a 3 2b3 2c 3
3 + 3 + 3 3.
a 2 + b2 b2 + c2 c2 + a2

Lời giải.
Áp d ng b t đ ng th c Holder ta có:
3
2a 3 a2
3 . 2a . a2 + c2 .
a 2 + b2 a 2 + b2 a 2 + c2

a2 3
Mà (Ch ng minh b t đ ng th c này đã đ c trình bày Cách 1 Ví dụ
a +b
2 2
a +c2 2
4
2.25).
3
2a 3
3 3 a a2 27
a + b2
2

Nguy n Quang Huy K49 THPT Chuyên ĐHSP


L Ư U H À N H N Ộ I B Ộ | 56

2a 3
3 3.
a 2 + b2
Đ ng th c x y ra khi và ch khi a = b = c = 1 .

Ví dụ 2.30. Cho . Ch ng minh r ng:

1 1 1 9
+ + .
a 2 a + bc
2
b 2 b + ca 2
c 2 c + ab 2 2 ab + bc + ca

Lời giải.

Áp d ng b t đ ng th c Holder ta có:
2

1 2 a 2 + bc 1
3

.
a 2 a 2 + bc a a

5 2
Nên ta s ch ng minh 2 ab 81 abc a 2b2 + abc a .

Đ t . Khi đó b t đ ng th c tr thành

5
2 x 81xyz x2 + yz .

Vì 9xyz x yz .

4
Ta ch ng minh r ng 2 x 9 yz x2 + yz .

x2 - yz 2 x2 + yz 0 Đúng .

Đ ng th c x y ra khi và ch khi a = b = c .

BÀI TẬP

60. Cho , trong đó không có 2 s nào đ ng th i b ng 0. Ch ng minh r ng:


b+c c+a a+b
+ + 4.
a 2 + bc b2 + ca c 2 + ab
Phạm Kim Hùng

Lời giải.
Áp d ng b t đ ng th c Holder ta có:
2
b+c 3 3
b + c a 2 + bc b+c =8 a .
a + bc
2

3
Bài toán quy v ch ng minh a 2 b + c a 2 + bc .
a3 + 3abc bc b + c Đúng .
Đ ng th c x y ra khi và ch khi và các hoán v .

61. Cho , trong đó không có 2 s nào đ ng th i b ng 0. Ch ng minh r ng:

Nguy n Quang Huy K49 THPT Chuyên ĐHSP


L Ư U H À N H N Ộ I B Ộ | 57

b+c c+a a+b


+ + 3.
a 2 + 3bc b 2 + 3ca c 2 + 3ab
Michael Rozenberg
Nhận xét 2.10. Ta có Bài toán Tổng quát cho 2 bài toán này.
Cho , trong đó không có 2 s nào đ ng th i b ng 0 và k 0 . Ch ng minh r ng:
b+c c+a a +b 2 6
+ + min + 2, .
a + kbc
2
b + kca 2
c + kab
2
k k +1
62. Cho . Ch ng minh r ng:
3 3 3
a+b b+c c+a
+ + 1.
8ab 4a + 4b + c 8bc 4b + 4c + a 8ca 4c + 4a + b
63. Cho , trong đó không có 2 s nào đ ng th i b ng 0. Ch ng minh r ng:
a b c
+ + 1.
4b + bc + 4c
2 2
4c + ca + 4a
2 2
4a + ab + 4b2
2

Phạm Kim Hùng – Võ Quốc Bá Cẩn


Đ ng th c x y ra khi nào ?
64. Cho , trong đó không có 2 s nào đ ng th i b ng 0. Ch ng minh r ng:
a b c a +b+c
+ + .
b + bc + c
2
c + ca + a
2 2
4a + ab + 4b 2 2 2
ab + bc + ca
Đ ng th c x y ra khi nào ?
65. Cho th a mãn x 2 + y 2 + z 2 = 1 . Ch ng minh r ng:
1 1 1 9
+ + 3
.
2 2 2
x+ y y+z z+x 18
3
1- 3
1- 3
1-
4 4 4
Võ Quốc Bá Cẩn
Đ ng th c x y ra khi nào ?
66. Cho a, b, c là đ dài 3 c nh c a 1 tam giác th a mãn . Ch ng minh r ng:
a b c
+ + 3.
b+c-a c + a -b a+b-c

Nguy n Quang Huy K49 THPT Chuyên ĐHSP


L Ư U H À N H N Ộ I B Ộ | 58

Ví dụ 2.31. Cho . Ch ng minh r ng:

1
a b c ab + bc + ca 3
+ + 3 2 2 2 .
a 2 + ab + b 2 b 2 + bc + c 2 c 2 + ca + a 2 a +b +c

Lời giải.

Áp d ng b t đ ng th c Holder ta có:
2
a 3
a a 2 + ab + b 2 a .
a + ab + b
2 2

2 3 3 2
a c c c
= = .
a 2 + ab + b 2 a a 2 + ab + b 2 c c2 c2

2 2
c ab + bc + ca 3
Ta c n ch ng minh 3 .
c2 a 2 + b2 + c 2

6 2
c 27 ab c2 (Đúng theo b t đ ng th c AM - GM ).

Đ ng th c x y ra khi và ch khi a = b = c .

Ví dụ 2.32. Cho , trong đó không có 2 s nào đ ng th i b ng 0. Ch ng minh r ng:

1 1 1 1 2
+ 2 + 2 + 2 2 2.
a + ab + b b + bc + c c + ca + a 2
2 2 2
bc + ca + ab a + b + c

Dương Đức Lâm

Lời giải.

. Áp d ng b t đ ng th c AM - GM ta có:

1 2 1 2 2
b2 + bc + c 2 ab + bc + ca b 2 + bc + c 2 + ab + bc + ca = b+c a +b +c .
4 4

1 4 ab + bc + ca
b + bc + c 2
2
b+c
2
a+b+c
2

1 4 ab 1
.
b + bc + c 2
2
c
2
b+c
2

2
4 ab 1 1 2 c
Do đó ta s ch ng minh 2 2
+ = .
c a +b ab c2 ab c2

2 1 4
4 c2 ab 2
c .
a +b

Th t vây, áp d ng b t đ ng th c Holder ta có:

Nguy n Quang Huy K49 THPT Chuyên ĐHSP


L Ư U H À N H N Ộ I B Ộ | 59

2 1 2 1 4
4 c2 ab 2
= c2 c a +b 2
c .
a+b a+b

. Ta ch ng minh b t đ ng th c ch t h n

1 1 1 5 4
+ 2 + 2 + .
a + ab + b b + bc + c c + ca + a 2
2 2 2
3 bc + ca + ab 3 a + b2 + c 2
2

(2.3)

B t đ ng th c t ng đ ng v i

c a+b ab 5 4 ab
+ + .
a 2 + ab + b 2 a 2 + ab + b 2 3 3 c2

Ta có 1 b đ : N u cho , trong đó không có 2 s nào đ ng th i b ng 0. Khi đó, ta có:

c a +b
2.
a 2 + ab + b2

(2.4)

Đ ng th c x y ra khi và ch khi a = b = c ho c và các hoán v .

Tr l i v i bài toán chính, s d ng b đ này, ta ch c n ch ng minh

1 ab 4 ab
+ .
3 a + ab + b2
2
3 c2

Áp d ng b t đ ng th c AM - GM ta ch ng minh b t đ ng th c ch t h n:

1 2 ab 4 ab
+ .
3 3 a + b2
2
3 c2

ab 4 ab
1+ 2
a + b2
2
c2

2ab 4 ab
1+ +2
a + b2
2
c2

2 2
a+b 2 a
Đúng theo b t đ ng th c ).
a 2 + b2 a2

. Ta xét 2 tr ng h p.

:N u a2 4 bc .

Ta có 1 b đ : N u cho , trong đó không có 2 s nào đ ng th i b ng 0. Khi đó, ta có:

1 9
2
.
b+c 4 bc

(2.5)

Đ ng th c x y ra khi và ch khi a = b = c ho c và các hoán v .

Nguy n Quang Huy K49 THPT Chuyên ĐHSP


L Ư U H À N H N Ộ I B Ộ | 60

Tr l i v i bài toán chính. Ta có

2 1 2 1 3
+ .
a2 2 bc a2 bc 2 bc

1 3
Nên ta ch c n ch ng minh .
b2 + bc + c 2 2 bc

Áp d ng b đ trên, ta có:

1 1 9 3
.
b + bc + c 2
2
b+c
2
4 bc 2 bc

:N u a2 4 bc .

Ta có 1 b đ : N u cho . Khi đó, ta có:

1 9
.
a 2 + ab + b2 c
2

(2.6)

Đ ng th c x y ra khi và ch khi a = b = c .

9 2 1
Áp d ng b đ trên, ta ch ng minh 2
+ .
a a2 bc

Đúng .

Đ ng th c x y ra khi và ch khi a = b = c > 0 .

Lưu ý 2.4. B t đ ng th c (2.3) trong bài toán trên:

Cho , trong đó không có 2 s nào đ ng th i b ng 0. Khi đó, ta có:

1 1 1 5 4
+ + + .
a 2 + ab + b2 b2 + bc + c 2 c 2 + ca + a 2 3 bc + ca + ab 3 a 2 + b2 + c 2

Dương Đức Lâm

Đ ng th c x y ra khi và ch khi a = b = c ho c và các hoán v .

Lưu ý 2.5. B t đ ng th c (2.4) trong bài toán trên:

Cho , trong đó không có 2 s nào đ ng th i b ng 0. Khi đó, ta có:

c a +b a b+c b c+a
+ 2 + 2 2.
a + ab + b b + bc + c c + ca + a 2
2 2 2

Darij Grinberg

Đ ng th c x y ra khi và ch khi a = b = c ho c và các hoán v .

Nguy n Quang Huy K49 THPT Chuyên ĐHSP


L Ư U H À N H N Ộ I B Ộ | 61

Lời giải.

. Ta có:

1 4 ab + bc + ca
(Ch ng minh Cách 1 Ví dụ 32).
b + bc + c 2
2
b+c
2
a +b+c
2

a b+c 4a ab + bc + ca
.
b + bc + c 2
2
b+c a+b+c
2

a b+c 4 ab + bc + ca a
.
b + bc + c 2
2
a +b+c
2
b+c

Áp d ng b t đ ng th c ta có:

2
4 ab + bc + ca a 4 bc a
. = 2.
a +b+c
2
b+c a
2
a b+c

a b+c a b+c a +b +c
a . -2 = - 2a
b + bc + c 2
2
b2 + bc + c 2

a ab + ac - b 2 - c 2
=
b2 + bc + c 2

ab a - b - ca c - a
=
b 2 + bc + c 2

ab a - b ab a - b
= -
b2 + bc + c 2 c 2 + ca + a 2
2
ab a - b
= a . 0.
b2 + bc + c 2 c 2 + ca + a 2

. Ta có 1 b đ :

N u cho , trong đó không có 2 s nào đ ng th i b ng 0. Khi đó, ta có:

1 1 2 a+b
+ .
b + bc + c 2 c 2 + ca + a 2
2
ab a + b + bc b + c + ca c + a

(2.7)

Đ ng th c x y ra khi và ch khi a = b > 0 .

Chứng minh.

Áp d ng b t đ ng th c AM - GM ta có:

1 1 2
+ 2 .
b + bc + c c + ca + a 2
2 2
b + bc + c
2 2
c 2 + ca + a 2

Nguy n Quang Huy K49 THPT Chuyên ĐHSP


L Ư U H À N H N Ộ I B Ộ | 62

ab a + b
Do đó ta c n ch ng minh b2 + bc + c 2 c 2 + ca + a 2 .
a +b

Áp d ng b t đ ng th c ta có:

bc ca + b2 + bc + c 2 c 2 + ca + a 2 c 2 + ca + a 2 + ca b2 + bc + c 2 + bc = a + c b + c = ab + c a + b + c 2

b2 + bc + c 2 c 2 + ca + a 2 ab + a + b - ab c + c 2 .

Áp d ng b t đ ng th c AM - GM ta có:

2ab
ab + a + b - ab c + c 2 ab + a + b - c + c2
a +b

a 2 + b2
= ab + c + c2
a +b

ab a + b
= .
a +b

Đ ng th c x y ra khi và ch khi a = b > 0 .

Tr l i v i bài toán chính. Áp d ng b đ trên, ta có:

a b+c ab ac
= +
b 2 + bc + c 2 b2 + bc + c 2 b 2 + bc + c 2

ab ab
= +
b + bc + c 2
2
c + ca + a 2
2

1 1
= ab +
b2 + bc + c 2 c 2 + ca + a 2

2 a +b
ab . = 2.
ab a + b

Đ ng th c x y ra khi và ch khi a = b = c ho c và các hoán v .

Nhận xét 2.11. Ta có các Bài toán Mở rộng cho bài toán này:

Cho , trong đó không có 2 s nào đ ng th i b ng 0. Ch ng minh r ng:

c a+b a b+c b c+a a 2b + b 2c + c 2 a ab2 + bc 2 + ca 2


a) + + + .
a + ab + b2 b2 + bc + c 2 c 2 + ca + a 2
2
ab2 + bc 2 + ca 2 a 2b + b 2c + c 2 a
Đ ng th c x y ra khi nào ?
2 2 2
c a+b a b+c b c+a a -b b-c c-a
b) + + 2+4 .
a 2 + ab + b2 b2 + bc + c 2 c 2 + ca + a 2 a +b
2
b+c
2
c+a
2

Vasile Cirtoaje

Ta có 1 bài toán nh sau:

Cho , trong đó không có 2 s nào đ ng th i b ng 0. Ch ng minh r ng:

Nguy n Quang Huy K49 THPT Chuyên ĐHSP


L Ư U H À N H N Ộ I B Ộ | 63

c2 a + b a2 b + c b2 c + a
+ + 2.
a 3 + abc + b3 b3 + abc + c3 c 3 + abc + a3
Dương Đức Lâm

Đ ng th c x y ra khi nào ?

Lưu ý 2.6. B t đ ng th c (2.5) trong bài toán trên (B t đ ng th c )

Cho , trong đó không có 2 s nào đ ng th i b ng 0. Khi đó, ta có:

1 1 1 9
+ + .
b+c
2
c+a
2
a+b
2
4 bc + ca + ab

Ji Chen

Đ ng th c x y ra khi và ch khi a = b = c ho c và các hoán v .

Lời giải.

. Áp d ng b t đ ng th c ta có:

1 2 1
11 a 2 + 14 bc 2
= 3a + b + c 2
b+c b+c

2
3a + b + c
b+c

2
a
= 9 1+ .
b+c

a
2
11 a 2 + 14 bc
Do đó ta c n ch ng minh 4 1+ .
b+c bc

Do b t đ ng th c là đ ng b c nên không m t tính t ng quát, ta gi s a = 1.

4q - 1
Ta s d ng ph ng pháp r max 0, (Do p = a = 1 ).
9

B t đ ng th c c n ch ng minh tr thành
2
1+ q 11 - 8q
4 -2 .
q-r q

N u . Ta có:

2 2
1+ q 11 - 8q 1+ q 11 - 8q
4 -2 - 4 -2 -
q-r q q q

4 - 3q 1 - 4q
=
q2

0.

N u . Ta có:

Nguy n Quang Huy K49 THPT Chuyên ĐHSP


L Ư U H À N H N Ộ I B Ộ | 64

2
2
1+ q 11 - 8q 1+ q 11 - 8q
4 -2 - 4 -2 -
q-r q 4q - 1 q
q-
9

1 - 3q 4q - 1 11 - 17 q
= 2
q 5q + 1

0.

. Ta có 1 b đ :

N u cho , trong đó không có 2 s nào đ ng th i b ng 0 và a b c . Khi đó, ta có:

1 1 1 2 1
+ + + .
a+b
2
b+c
2
c+a
2
a+c b+c 4ab

(2.8)

Đ ng th c x y ra khi và ch khi a = b > 0 .

Chứng minh.

1 2 1 1 1 2 1 1
- - = + - - -
a+b
2
a+c b+c 4ab b+c
2
c+a
2
a+c b+c 4ab a + b 2

2
a -b
2
1 1
= - -
b+c c+a 4ab a + b
2

2 2
a -b a -b
= 2 2
- 2
b+c c+a 4ab a + b

2 2
a -b a -b
2
- 2
a+b b+b a+a 4ab a + b

=0.

Đ ng th c x y ra khi và ch khi a = b > 0 .

Tr l i bài toán chính. Áp d ng b đ trên, ta có:

1 2 1
bc bc + .
b+c
2
a+c b+c 4ab

2 1 9
Bài toán quy v ch ng minh bc + .
a + c b + c 4ab 4

2 ab + bc + ca ab + bc + ca 9
+
a+c b+c 4ab 4

2c 2 1 c a +b 9
2- + +
a+c b+c 4 4ab 4

Nguy n Quang Huy K49 THPT Chuyên ĐHSP


L Ư U H À N H N Ộ I B Ộ | 65

c a +b 2c 2
4ab a+c b+c

a+b 8abc Đúng theo b t đ ng th c AM - GM ).

Ta có 1 b đ : N u cho , trong đó không có 2 s nào đ ng th i b ng 0 và a b c . Khi đó, ta có:

2 2 2
a b c 1 5 a 2 + b2 + c 2
+ + + . .
b+c c+a a+b 2 4 ab + bc + ca

Trần Quốc Anh-Dương Đức Lâm

(2.9)

Đ ng th c x y ra khi và ch khi a = b = c ho c và các hoán v .

Chứng minh.

Áp d ng b t đ ng th c ta có:

2
a2 a2
2 2
.
b+c a2 b + c

2
a2 1 5 a2
Nên ta ch c n ch ng minh 2
+ . .
a2 b + c 2 4 bc

Do b t đ ng th c là đ ng b c nên không m t tính t ng quát, ta gi s a = 1.

4q - 1 1 - q
Ta s d ng ph ng pháp r max 0, (Do p = a = 1 ).
6

2
1 - 2q 5
B t đ ng th c tr thành +6 .
q2 - r 2q

1
N uq . Ta có:
4
2
1 - 2q 5 2 - 5q 1 - 4q
+6- = 0.
q -r
2
2q 2q 2

1
N uq . Ta có:
4
2
1 - 2q 5 5 - 14q 1 - 3q 4q - 1
+6- = 0.
q -r
2
2q 2q 10q 2 - 5q + 1

Đ ng th c x y ra khi và ch khi a = b = c ho c và các hoán v .

Tr l i bài toán chính.

Nguy n Quang Huy K49 THPT Chuyên ĐHSP


L Ư U H À N H N Ộ I B Ộ | 66

2
2 1 9 a
Ta ch ng minh a 2
b+c 4 bc

2 2
a 9 a
1+
b+c 4 bc

2
a a 3 9 a2
+2 + .
b+c b+c 2 4 bc

Áp d ng b đ trên ta có:
2
a a 5 a2 1 a
+2 - +2 .
b+c b+c 4 bc 2 b+c

Bài toán quy v ch ng minh


2
a a
Đúng theo b t đ ng th c ).
b+c 2 bc

Đ ng th c x y ra khi và ch khi a = b = c ho c và các hoán v .

Nhận xét 2.12. Ta có Bài Toán Mở rộng cho bài toán trên.

Cho , trong đó không có 2 s nào đ ng th i b ng 0 và k > 0 . Ch ng minh r ng:

1 1 1 9
2 + 2 + 2 2
.
kb + c kc + a ka + b k +1 bc + ca + ab

Đ ng th c x y ra khi và ch khi a = b = c .

Lưu ý 2.7. B t đ ng th c (2.6) trong bài toán trên:

Cho . Ch ng minh r ng:

1 1 1 9
+ 2 + 2 .
a + ab + b b + bc + c c + ca + a 2
2 2 2
a+b+c
2

Vasile Cirtoaje

Đ ng th c x y ra khi và ch khi a = b = c .

Lời giải.

. Ta có:

1 4 ab + bc + ca
(Ch ng minh Cách 1 Ví dụ 2.32).
b2 + bc + c 2 b+c
2
a +b+c
2

1 4 bc 1
.
a + ab + b2
2
a
2
b+c
2

Nguy n Quang Huy K49 THPT Chuyên ĐHSP


L Ư U H À N H N Ộ I B Ộ | 67

Áp d ng b t đ ng th c ta có:

4 bc 1 9
2 2 2
.
a b+c a+b+c

. Ta ch ng minh

1 9 a2 + bc
a2 + bc
b 2 + bc + c 2 a
2

2
b 2 + bc + c 2 + a 2 + ab + ac 9 a - bc
b + bc + c
2 2
a
2

a 9 bc
a + 6.
b2 + bc + c 2 a
2

Áp d ng b t đ ng th c và b t đ ng th c AM - GM ta có:

2
a 9 bc a 9 bc
a + a +
b + bc + c
2 2
a
2
a b + bc + c2 2
a
2

a 9 bc
= a + 2
bc a

2
a 9 bc
2 . 2
bc a

=6.

. Do b t đ ng th c là đ ng b c nên không m t tính t ng quát, ta gi s a = 1.

4q - 1
Ta s d ng ph ng pháp r max 0, (Do p = a = 1 ).
9

Đ ý r ng .

Do đó, b t đ ng th c tr thành

1
9.
1- a - q

9q3 - 6q 2 - 3q + 1 + 9r 0 .

1
N u 0<q . Ta có:
4

3 1
9q3 - 6q 2 - 3q + 1 + 9r > 9q 3 - 6q 2 - 3q + 1 = 4q - 1 48q 2 - 20q - 21 + >0.
64 64

1
N uq . Ta có:
4

Nguy n Quang Huy K49 THPT Chuyên ĐHSP


L Ư U H À N H N Ộ I B Ộ | 68

Đ ng th c x y ra khi và ch khi a = b = c .

BÀI TẬP

67. Cho th a mãn ab + bc + ca = 3 . Ch ng minh r ng:


bc ca ab
+ + 1.
6a + 3
2
6b + 3
2
6c 2 + 3

Vasile Cirtoaje

Lời giải.
Áp d ng b t đ ng th c Holder ta có:
2
bc 3
b 2 c 2 6a 2 + 3 bc .
6a 2 + 3
3
Bài toán quy v ch ng minh bc b 2c 2 6a 2 + 3 = b 2c 2 6a 2 + bc .
2
bc bc - b2c 2 18a 2b 2c 2

2abc a 6a 2b2c2
1 2
abc a b-c 0 Đúng .
3
Đ ng th c x y ra khi và ch khi a = b = c = 1 ho c và các hoán v .

68. Cho , trong đó không có 2 s nào đ ng th i b ng 0. Ch ng minh r ng:


4
a a +b a +c b b+c b+a c c +a c +b a +b+c
3
+ 3
+ 3 2
.
b+c c+a a +b 6 ab + bc + ca

Lời giải.
Do b t đ ng th c là đ ng b c nên không m t tính t ng quát, ta gi s bc = 3 .
B t đ ng th c tr thành
4
a a2 + 3 a
3
.
b+c 54
4
a3 a a
3
+3 3
.
b+c b+c 54
Áp d ng b t đ ng th c Holder ta có:
3 a3 4
a b+c 3
a a .
b+c

3 a 4
a b+c 3
a .
b+c
2 3 2 2 4 4
3 a a a 3 bc . a =3 a a a a .

Nguy n Quang Huy K49 THPT Chuyên ĐHSP


L Ư U H À N H N Ộ I B Ộ | 69

4 4 4
a3 a a a +3 a a
3
+3 3 3
.
b+c b+c a b+c 54

Đ ng th c x y ra khi và ch khi a = b = c .

69. Cho , trong đó không có 2 s nào đ ng th i b ng 0. Ch ng minh r ng:


1 1 1 1 8
+ + + .
a 2 + ab + b2 b2 + bc + c 2 c 2 + ca + a 2 3 bc + ca + ab a 2 + b2 + c 2
Phạm Kim Hùng

Ví dụ 2.33. Cho , trong đó không có 2 s nào đ ng th i b ng 0. Ch ng minh r ng:

1 1 1 4
+ + .
4a + bc
2
4b + ca 2
4c + ab
2 a+b+c

Phạm Kim Hùng

Lời giải.

. Áp d ng b t đ ng th c Holder ta có:
2
1 3 3 3
b+c 4a 2 + bc b+c =8 a .
4a + bc
2

5 3
Bài toán quy v ch ng minh a 2 b+c 4a 2 + bc .

2
a3 a - b a - c + 4 ab a + b a - b + abc 19 a 2 - 18 bc 0 Đúng .

Không m t tính t ng quát, ta gi s a b c .

a
. Ta có: a -c b-c .
b

Áp d ng b t đ ng th c AM - GM ta có:

3 1 1
a+ b+ c
1 2 2 2
=
4a 2 + bc 3 1 1
a + b + c 4a 2 + bc
2 2 2

3 1 1
a+ b+ c
2 2 2 2
2
3 1 1
a + b + c + 4a 2 + bc
2 2 2

3a + b + c
=4 2
.
16a 2 + 4bc + 3a + b + c

Nên ta c n ch ng minh

3a + b + c 1
2
16a + 4bc + 3a + b + c
2
a

3a + b + c a
2
1
16a + 4bc + 3a + b + c
2

Nguy n Quang Huy K49 THPT Chuyên ĐHSP


L Ư U H À N H N Ộ I B Ộ | 70

3a + b + c a 1
2
- 0
16a + 4bc + 3a + b + c
2
3

b2 + c 2 - 8a 2 + 3a b + c
2 2
0
16a 2 + 4bc + 3a + b + c

c + 4a c - a + b + 4a b - a
2
0
16a 2 + 4bc + 3a + b + c

c + 4a c - a b + 4a b - a
2
+ 2
0
16a + 4bc + 3a + b + c
2
16a + 4bc + 3a + b + c
2

c + 4b a - b b + 4a b - a
2
+ 2
0
16b + 4ca + 3b + c + a
2
16a + 4bc + 3a + b + c
2

2
a -b 7 a 2 + 7b2 - c 2 + 34ab - 6bc - 6ca
7 2 2
0
16b 2 + 4ca + 3b + c + a 16a 2 + 4bc + 3a + b + c

2 2
7a 2 + 7b2 - c 2 + 34ab - 6bc - 6ca 16c 2 + 4ab + 3c + a + b a -b 0.

Ta có:
2 2
7a 2 + 7b2 - c 2 + 34ab - 6bc - 6ca 16c 2 + 4ab + 3c + a + b a -b

2 2
7b2 + 7c 2 - a 2 + 34bc - 6ca - 6ab 16a 2 + 4bc + 3a + b + c b-c

2 2
+ 7c 2 + 7a 2 - b2 + 34ca - 6ab - 6bc 16c 2 + 4ca + 3b + c + a c-a

2 2
(Do a b c nên 7a 2 + 7b 2 - c 2 + 34ab - 6bc - 6ca 16c 2 + 4ab + 3c + a + b a -b 0)

2 2
7b2 + 7c 2 - a 2 + 34bc - 6ca - 6ab 16a 2 + 4bc + 3a + b + c b-c

a2 2 2
+ 7c 2 + 7 a 2 - b 2 + 34ca - 6ab - 6bc 16c 2 + 4ca + 3b + c + a b-c
b2
2
b-c
2 a 2 7c 2 + 7a 2 - b2 + 34ca - 6ab - 6bc 16c 2 + 4ca + 3b + c + a
=
b2 +b 2 7b 2 + 7c 2 - a 2 + 34bc - 6ca - 6ab 16a 2 + 4bc + 3a + b + c
2

2
b-c 2
b 2 16a 2 + 4bc + 3a + b + c 7c 2 + 7a 2 - b 2 + 34ca - 6ab - 6bc + 7b 2 + 7c 2 - a 2 + 34bc - 6ca - 6ab
b2

(Do a b c )
2
b -c 2 2
= b 2 16a 2 + 4bc + 3a + b + c 6 a - b + 28c a + b
b2

0.

a +b
.Đ t t= c . Ta ch ng minh b t đ ng th c
2

Nguy n Quang Huy K49 THPT Chuyên ĐHSP


L Ư U H À N H N Ộ I B Ộ | 71

1 2 1
+ .
4a 2 + bc 4t 2 + tc 4c 2 + t 2

Áp d ng b t đ ng th c AM - GM ta có:

1 1 1 2 1 2 1
+ + + = +
4a 2 + bc 4b 2 + ca 4c 2 + ab 4 4a + bc 4b + ca
2 2
a +b
2
4 4a + bc 4b + ca
2 2
4c 2 + t 2
4c 2 +
4

2
2 2 c a+b
Do đó ta c n ch ng minh 4a 2 + bc 4b2 + ca 4t 2 + tc = a+b + .
2

2 c2
a -b + a 2 + b 2 + 6ab - 3bc - 3ca 0 Đúng .
4

2 1 4 4
Bây gi ta ch ng minh + =
4t + tc 2
4c + t
2 2 a 2t + c

1 2 1 1 2 4
- + - -
t 4t + tc
2 t 4c 2 + t 2 t 2t + c

2 1 4c
c - - 0.
t 2t + c 4t + tc 2t + 4t + tc t 4c + t t + 4c 2 + t 2
2 2 2 2

Th c ch t b t đ ng th c này là h qu c a 2 b t đ ng th c sau:

1 1
.
3t 2t + c 4t + tc 2t + 4t 2 + tc
2

5 4c
.
3t 2t + c t 4c + t 2 2
t + 4c 2 + t 2

Chứng minh.

1 1
3t 2t + c 4t + tc 2t + 4t 2 + tc
2

2
2
9t 2 2t + c 4t 2 + tc 2t + 4t 2 + tc

t 2 + 6ta + 2c 2 2t 4t 2 + tc + c 4t 2 + tc .

B t đ ng th c này đúng do t c . B n đ c t ki m tra.

5 4c
3t 2t + c t 4c + t 2 2
t + 4c 2 + t 2

5 4c 2 + t 2 t + 4c 2 + t 2 12c 2t + c .

Áp d ng b t đ ng th c ta có:

5 4c 2 + t 2 t + 4c2 + t 2 = 5 4c 2 + t 2 5t + 5 4c 2 + t 2

Nguy n Quang Huy K49 THPT Chuyên ĐHSP


L Ư U H À N H N Ộ I B Ộ | 72

4c + t 4c + 5 +1 t .

V y ta c n ch ng minh 4c + t 4c + 5 +1 t 12c 2t + c

5 + 1 t 2 + 16 - 4 5 tc + 4c 2 0.

B t đ ng th c này đúng do D < 0 . B n đ c t ki m tra.

Đ ng th c x y ra khi và ch khi và các hoán v .

Nhận xét 2.13. Ta có Bài toán Mở rộng cho bài toán này.

Cho , trong đó không có 2 s nào đ ng th i b ng 0. Tìm h ng s t t nh t đ b t đ ng th c sau


đúng:

1 1 1 4 + 2k
+ + .
k a + bc
2 2
k b + ca
2 2
k c + ab
2 2 k a +b+c

BÀI TẬP

70. Cho , trong đó không có 2 s nào đ ng th i b ng 0. Ch ng minh r ng:


1 1 1 2
+ + .
4a + bc
2
4b + ca
2
4c + ab
2
ab + bc + ca
Võ Quốc Bá Cẩn

Lời giải.
Áp d ng b t đ ng th c Holder ta có:
2
1 3 3 3
b+c 4a 2 + bc b+c =8 a .
4a + bc
2

3 3
Bài toán quy v ch ng minh 2 a bc b+c 4a 2 + bc

Đúng .
Đ ng th c x y ra khi và ch khi và các hoán v .

Ví dụ 2.34. Cho th a mãn a + b + c = 3 . Ch ng minh r ng:

a3 + 3b + b3 + 3c + c3 + 3a 6.

Lời giải.

Ta có 1 b đ : N u cho th a mãn a + b + c = 3 . Khi đó ta có:

(2.10)

Đ ng th c x y ra khi và ch khi a = b = c = 1 ho c a, b, c = 0,1, 2 và các hoán v .

Nguy n Quang Huy K49 THPT Chuyên ĐHSP


L Ư U H À N H N Ộ I B Ộ | 73

Tr l i bài toán chính.

Áp d ng b t đ ng th c ta có:

2
a3 + 3b a + 3b a 2 + 3b .

a 2 + 3b
a3 + 3b .
a + 3b

a 2 + 3b
a3 + 3b .
a + 3b

Áp d ng b t đ ng th c Holder ta có:
2
a 2 + 3b 3 3
a 2 + 3b a + 3b a 2 + 3b = a2 + 9 .
a + 3b

3
Bài toán quy v ch ng minh a2 + 9 36 a 2 + 3b a + 3b .

Ta s d ng ph ng pháp .

Áp d ng b đ trên, ta có:

24 5 - q .

3
Nên ta ch ng minh 9-q 108 5 - q

2
3- q 21 - q 0 Đúng .

Đ ng th c x y ra khi và ch khi a = b = c = 1 .

Lưu ý 2.8. B t đ ng th c (2.10) trong bài toán trên:

Cho th a mãn a + b + c = 3 . Khi đó ta có:

.
Vasile Cirtoaje

Đ ng th c x y ra khi và ch khi a = b = c = 1 ho c a, b, c = 0,1, 2 và các hoán v .

Lời giải.

. Không m t tính t ng quát, gi s là s n m gi a và c . Khi đó, ta có:

a b -c b-a 0.

3
2 1 1 2b + c + a + c + a
b a+c = .2b c + a c + a = 4.
2 2 3

Nguy n Quang Huy K49 THPT Chuyên ĐHSP


L Ư U H À N H N Ộ I B Ộ | 74

. Ta ch ng minh b t đ ng th c
3
4 a 27 a 2b + abc .

Không m t tính t ng quát, ta gi s a = min a, b, c . Khi đó, đ t b = a + x và c = a + y v i .

B t đ ng th c t ng đ ng
2
9 x2 - xy + y 2 a + 2 x - y x + 4y 0 Đúng).

Không m t tính t ng quát, ta gi s a b c .

Do b t đ ng th c là đ ng b c nên không m t tính t ng quát, ta gi s a = 1.

Ta có:

a 2b + ab2 + a 2b - ab2
a 2b + abc = + abc
2

a 2b + ab2 + 3abc - 3abc - ab2 - a 2b


= + abc
2

ab c - 3abc - a -b
= + abc
2

ab c - 3abc + a -b
= + abc (Do a b c )
2

2
ab c - 3abc + a -b
= + abc .
2

Ta s d ng ph ng pháp

q - 3r + q 2 - 4q 3 + 2 9q - 2 r - 27r 2
a 2b + abc = + r (Do p = a =1)
2

q - r + q 2 - 4q3 + 2 9q - 2 r - 27 r 2
= = f r .
2

Ta có:

9q - 2 - 27r - q 2 - 4q3 + 2 9q - 2 r - 27 r 2
f r = .
2 q 2 - 4q3 + 2 9q - 2 r - 27 r 2

7 9q - 2 - 1 - 3q 7 1 - 3q
f r =0 r = r0 = .
189

Và 9q - 2 - 27r0 = q 2 - 4q3 + 2 9q - 2 r0 - 27r02 .

Nguy n Quang Huy K49 THPT Chuyên ĐHSP


L Ư U H À N H N Ộ I B Ộ | 75

. N u 7 9q - 2 1 - 3q 7 1 - 3q .

Ta có: f r 0.

1 - 4q + 1 1 - 1 - 4q
f r f 0 = .
8

1 - 4q + 1 1 - 1 - 4 q 4
Ta ch ng minh .
8 27

Th t v y, ta có:
2
4 1 - 4q + 1 1 - 1 - 4 q 3 1 - 4q - 1 3 1 - 4q + 5
- = 0.
27 8 216

. N u 7 9q - 2 1 - 3q 7 1 - 3q .

3
1 - 3q 7 1 - 3 1 - 3q

3
1 - 3q .
2 7

Ta có: f r 0.

q - r0 + q 2 - 4q3 + 2 9q - 2 r0 - 27 r02
f r f r0 =
2

q - r0 + 9q - 2 - 27 r0
=
2

= 5q - 14r0 - 1

9q + 1 + 2 1 - 3q 7 1 - 3q
=
27

3
2 7 1 - 3q - 3 1 - 3q + 4
= .
27

3
2 7 1 - 3q - 3 1 - 3q + 4 4
Ta ch ng minh
27 27

1 - 3q 2 7 1 - 3q - 3 0 Đúng .

Đ ng th c x y ra khi và ch khi a = b = c = 1 ho c a, b, c = 0,1, 2 và các hoán v .

Đây là 1 k t qu quen thu c và đ p m t có th dùng đ gi i nhi u bài toán.

Nguy n Quang Huy K49 THPT Chuyên ĐHSP


L Ư U H À N H N Ộ I B Ộ | 76

Ví dụ 2.35. Cho th a mãn a + b + c = 3 . Ch ng minh r ng:

a 5 + a 4 + a 3 b5 + b 4 + b3 c 5 + c 4 + c 3
+ + 3.
a 3 + b 2 + c b3 + c 2 + a c 3 + a 2 + b

Rahim Shahbazov

Lời giải.

. Áp d ng b t đ ng th c Holder ta có:

a5 + a 4 + a3 2
a3 + a 2 + a a2 + a + 1 =
a3 + b2 + c

a5 + a 4 + a3 2
= a3 + b 2 + c a2 + a +1
a3 + b2 + c

3
a3 + a 2 + a .

a5 + a 4 + a3 2 2
a2 + a + 1 a3 + a 2 + a .
a3 + b 2 + c

Do đó ta c n ch ng minh
2 2
a3 + a 2 + a 3 a2 + a +1 .

Đ t abc = r và ab = 3 1 - t 2 t 0,1 .

Đ ý r ng

a 2 = 3 1 + 2t 2 .

a 3 = 3 9t 2 + r .

B t đ ng th c tr thành
2 2
t 2 127t 2 - 4 + 22r + r - 1 = 127t 4 - 4t 2 + 22t 2 r + r - 1 0.

Ta s ch ng minh

1 - 4t 2 2 + t 2
Ta có r max 0, (B t đ ng th c Schur ).
3

N u 4t 2 1 . Ta có:

99
115t 2 - 4 + 22r > 115t 2 - 4 > 0.
4

N u 4t 2 1 . Ta có:

Nguy n Quang Huy K49 THPT Chuyên ĐHSP


L Ư U H À N H N Ộ I B Ộ | 77

22 1 - 4t 2 2 + t 2
115t 2 - 4 + 22r 115t 2 - 4 +
3

-88t 4 + 227t 2 + 32
=
3

>0.

. Áp d ng b t đ ng th c AM - GM và b t đ ng th c ta có:

a5 + a 4 + a3 3 3 a5 .a 4 .a3
a3 + b2 + c a3 + b 2 + c

a4
=3
a3 + b2 + c

a a3 + b 2 + c - a b 2 + c
=3
a 3 + b2 + c

a b2 + c
=3 a-
a3 + b2 + c

b2 + c
=3 a-
b2 c 2
a2 + +
a ac

b2 + c
3 a- 2
a+b+c
1 + a + ac

1 + a + ac b2 + c
= 3 3-
9

2
b +2 ca + 3
= 3 3- .
9

Bài toán quy v ch ng minh


2
b +2 ca + 3 18

ca 3 Đúng .

Đ ng th c x y ra khi và ch khi a = b = c = 1 .

BÀI TẬP

71. Cho th a mãn abc = 1 . Ch ng minh r ng:

Nguy n Quang Huy K49 THPT Chuyên ĐHSP


L Ư U H À N H N Ộ I B Ộ | 78

c a 3 + b3 a b3 + c 3 b c 3 + a 3 3
+ 2 2 + 2 .
a 2 + b2 b +c c + a2 2

Lời giải.
Áp d ng b t đ ng th c ta có:
2
a3 + b3 a + b a 2 + b2

c a 3 + b3 c
a 2 + b2 a +b
c a 3 + b3 c
.
a 2 + b2 a+b
Áp d ng b t đ ng th c Holder ta có:
2
c 3
c a+b c .
a+b
3
Bài toán quy v ch ng minh 2 c 9 c a+b
3
c 9 ab .
Th t v y,
3 2
c = c c 3 3 abc 3 ab = 9 ab .
Đ ng th c x y ra khi và ch khi a = b = c = 1 .

72. Cho th a mãn a + b + c = 3 . Ch ng minh r ng:


a3 + 2bc + b3 + 2ca + c3 + 2ab 3 3.

Nguyễn Văn Quý

Lời giải.
Áp d ng b t đ ng th c ta có:
2
a3 + 2bc a + 2bc a 2 + 2bc
a 2 + 2bc
a3 + 2bc
a + 2bc
a 2 + 2bc
a3 + 2bc .
a + 2bc
L i áp d ng b t đ ng th c Holder ta có:
2
a 2 + 2bc 2 6
a + 2bc a 2 + 2bc a 2 + 2bc = a .
a + 2bc
Nh v y ta c n ch ng minh r ng
6
a 27 a + 2bc a 2 + 2bc
2
3 ab a - b 0 Đúng .
3 3
Đ ng th c x y ra khi và ch khi a = b = c = 1 ho c a, b, c = 0, , và các hoán v .
2 2

Ví dụ 2.36. Cho . Ch ng minh r ng:

a 2 + 3bc + b2 + 3ca + c2 + 3ab a + b + c + 3 ab + bc + ca .

Nguy n Quang Huy K49 THPT Chuyên ĐHSP


L Ư U H À N H N Ộ I B Ộ | 79

Lời giải.

Ta s ch ng minh a 2 + 3bc - a 3 bc

3bc
3 bc
a 2 + 3bc + a

bc bc
.
a + 3bc + a
2 3

Áp d ng b t đ ng th c Holder ta có:
2
2
bc 3
bc a 2 + 3bc + a bc .
a + 3bc + a
2

Bài toán quy v ch ng minh

2 2
3 bc bc a 2 + 3bc + a

2 a a 2 + 3bc .

Áp d ng b t đ ng th c ta có:

a 2 + 3bc 3 a 2 + 3bc

= 3 a2 + 9 bc

2
= 4 a + bc - a2

2 a.

Đ ng th c x y ra khi và ch khi a = b = c .

Ví dụ 2.37. Cho . Ch ng minh r ng:

a 2 + bc b 2 + ca c 2 + ab 3 2
+ + .
b+c c+a a +b 2

Vasile Cirtoaje

Lời giải.

. Áp d ng b t đ ng th c Holder ta có:
2
a 2 + bc 2 2 3
a 2 + bc b+c a 2 + bc .
b+c

3 9 2 2
Bài toán quy v ch ng minh a 2 + bc a 2 + bc b+c .
2
3
2 a6 + 6 a5 b + c + 12abc bc b + c - 4 bc - 6 a 4 b 2 + c 2 - 66a 2b 2c 2 0

Nguy n Quang Huy K49 THPT Chuyên ĐHSP


L Ư U H À N H N Ộ I B Ộ | 80

2 a 6 + 3a 2b2c 2 - a 4 b2 + c 2 +4 a5 b + c - a 4 b2 + c 2

3
+2 a5 b + c - 2 bc + 12abc bc b + c - 6abc 0.

B t đ ng th c này đúng theo b t đ ng th c Schur và b t đ ng th c AM - GM .

2
a 2 + bc a 2 + bc b 2 + ca
Vì 3 .
b+c b+c c+a

Ta ch ng minh b t đ ng th c

a 2 + bc b2 + ca 3
b+c c+a 2

2 a +b a 2 + bc b 2 + ca 3 a +b .

Áp d ng b t đ ng th c ta có:

2 a +b a 2 + bc b2 + ca = 2 a + b a 2 + bc a + b b 2 + ca

2 a3 + cb b3 + ca

3
=2 ab + a b5 + c5 + abc .

3
Ta ch ng minh 2 ab + a b5 + c 5 + abc 3 a+b

3
2 ab + 2 ab a 2 + b 2 3 ab a + b

ab 2ab + 2 a 2 + b 2 - 3 ab a + b 0

2
ab a- b 2a + ab + 2b 0 Đúng .

Đ ng th c x y ra khi và ch khi a = b = c .

BÀI TẬP

73. Cho . Ch ng minh r ng:


a b c 27 ab + bc + ca
+ + 4 .
a +b b+c c+a 4
Phạm Hữu Đức

Lời giải.
Áp d ng b t đ ng th c Holder ta có:

Nguy n Quang Huy K49 THPT Chuyên ĐHSP


L Ư U H À N H N Ộ I B Ộ | 81

2
a 3
a a +b a .
a +b
3 3
Bài toán quy v ch ng minh a a2 + bc 3 bc
2
3 2
2 a +3 bc 3 bc 3 a 3 bc .
Áp d ng b t đ ng th c AM - GM ta có:
3 3 3 2
2 a +3 bc 3 bc 33 a . a .3 bc 3 bc = 3 a 3 bc .
Đ ng th c x y ra khi và ch khi a = b = c .

Ví dụ 2.38. Cho , trong đó không có 2 s nào đ ng th i b ng 0. Ch ng minh r ng:

1 1 1 2
+ + 2 .
a + bc
2
b + ca
2
c + ab
2 ab + bc + ca

Phạm Kim Hùng

Lời giải.

Không m t tính t ng quát, ta gi s a = max a, b, c .

Áp d ng b t đ ng th c Holder ta có:
2
1 3 3
b+c a 2 + bc + a 3 b 2 + ca + a 3 c 2 + ab 2a + b + c .
a + bc
2

Bài toán quy v ch ng minh


3 3
2a + b + c . bc 8 b + c a 2 + bc + 8a 3 b 2 + c 2 + ca + ab .

Do b t đ ng th c này là đ ng b c đ i v i và c nên không m t tính t ng quát, ta gi s b + c = 1 . Khi đó,


b t đ ng th c tr thành
3
2a + 1 a + bc 8 a 2 + bc + 8a3 1 - 2bc + a

3 3
bc 2a + 1 + 16a3 - 8 + a 2a + 1 - 8a 2 - 8a 3 a + 1 0.

3 3
B t đ ng th c này đúng do 2a + 1 b + c + 1 = 8 và .

Đ ng th c không x y ra.

Nhận xét 2.14. Ta có các k t qu sau r t đ p m t c a tác gi Võ Quốc Bá Cẩn:

Cho , trong đó không có 2 s nào đ ng th i b ng 0. Ch ng minh r ng:

1 1 1 3 2 -4 2
a) + + +2 .
a + bc
2
b + ca c + ab
2
a +b +c 2 2 2 2 ab + bc + ca
9
b) k = - 2 là h ng s t t nh t đ b t đ ng th c sau đúng:
4
1 1 1 8
+ + .
a + kbc
2
b + kca
2
c + kab
2 k ab + bc + ca
c) k = 9 là h ng s t t nh t đ b t đ ng th c sau đúng:

Nguy n Quang Huy K49 THPT Chuyên ĐHSP


L Ư U H À N H N Ộ I B Ộ | 82

1 1 1 3
+ + 3 .
ka + bc
2
kb + ca
2
kc + ab
2 k + 1 ab + bc + ca
1 1 1 2 +1
d) + + .
2a + bc
2
2b + ca
2
2c + ab
2
ab + bc + ca
1 1 1 3+2
e) + + .
3a + bc
2
3b + ca
2
3c + ab
2
3 ab + bc + ca

Ta có 1 bài toán nh sau:

Cho , trong đó không có 2 s nào đ ng th i b ng 0 th a mãn ab + bc + ca = 1 . Ch ng minh r ng:

1 1 1
+ + 2 2.
a + bc b + ca c + ab

Võ Quốc Bá Cẩn

Ví dụ 2.39. Cho . Ch ng minh r ng:

a a 2 + 2bc + b b2 + 2ca + c c 2 + 2ab 3 ab + bc + ca .

Lời giải.

. Không m t tính t ng quát, ta gi s a b c .

Áp d ng b t đ ng th c Holder ta có:
2
a 3
a a 2 + 2bc a .
a 2 + 2bc

3
a a
Do đó ta c n ch ng minh .
3 bc
2
a + 2bc
2

3 2
a a a a
Mà ta có: 2
= 2
.
3 bc 3 bc bc

Bài toán quy v ch ng minh

a a
.
a + 2bc
2
bc

(2.11)

bc
a 1- 0
a + 2bc
2

a a -b a -c
0.
a 2 + 2bc

Th t v y,

a a -b a -c a a -b a - c b b -c b - a c c - a c -b
= + +
a 2 + 2bc a 2 + 2bc b2 + 2ca c 2 + 2ab

a a -b a -c b b -c b - a
+
a 2 + 2bc b2 + 2ca

Nguy n Quang Huy K49 THPT Chuyên ĐHSP


L Ư U H À N H N Ộ I B Ộ | 83

2
c a -b 2a a - c + 2b b - c + 3ab
=
a 2 + 2bc b 2 + 2ca

0.

. Không m t tính t ng quát, ta gi s a b c

. N u a 2 b + c . Áp d ng b t đ ng th c AM - GM ta có:

a a 2 + 2bc > a 2 + b2 + c 2

a2 2
+ 2 b + c + b2 + c 2
2

a2 a2
= + 3b2 + + 3c 2 + 4bc
4 4

a2 2 a2 2
>2 .3b + 2 .3c + 3bc
4 4

= 3 ab + bc + ca .

. N u a < 2 b + c . Ta ch ng minh b t đ ng th c

2 2
a a 2 + 2bc 3 bc

2
a 4 + 2abc a+2 bc b2 + 2ca c 2 + 2ab 3 bc .

Mà ta có:

a 4 + 2abc a+2 bc b 2 + 2ca c 2 + 2ab b2c 2 + 2abc a+2 bc b 2 + 2ca c 2 + 2ab

2
= bc + 2 bc b2 + 2ca c 2 + 2ab .

Bài toán s đúng n u ta ch ng minh đ c

2
bc b 2 + 2ca c 2 + 2ab bc .

Th t v y, ta s ch ng minh bc b2 + 2ca c 2 + 2ab bc bc

2
b-c 2b + 2c - a 0 Đúng .

Đ ng th c x y ra khi và ch khi a = b = c .

Lưu ý 2.9. B t đ ng th c (2.11) trong bài toán trên:

Cho , trong đó không có 2 s nào đ ng th i b ng 0. Khi đó ta có:

a b c a +b+c
+ + .
a 2 + 2bc b 2 + 2ca c 2 + 2ab ab + bc + ca

Nguy n Quang Huy K49 THPT Chuyên ĐHSP


L Ư U H À N H N Ộ I B Ộ | 84

Vasile Cirtoaje

Đ ng th c x y ra khi và ch khi a = b = c ho c và các hoán v .

Ta có 1 bài toán nh sau:

Cho , trong đó không có 2 s nào đ ng th i b ng 0. Ch ng minh r ng:

a b c a +b+c
+ + .
a + 2bc
2
b + 2ca
2
c + 2ab
2
ab + bc + ca

Hồ Phú Thái

Đ ng th c x y ra khi nào ?

Ví dụ 2.40. Cho th a mãn ab + bc + ca + abc = 4 . Ch ng minh r ng:

a b c a +b+c
+ + .
b+c c+a a+b 2

Cezar Lupu

Lời giải.

Ta có 1 b đ : N u cho th a mãn ab + bc + ca + abc = 4 . Khi đó ta có:

a bc .

(2.12)

Đ ng th c x y ra khi và ch khi a = b = c = 1 ho c a, b, c = 2, 2,0 và các hoán v .

Tr l i bài toán chính

. Áp d ng b t đ ng th c Holder ta có:
2
a 3
a b+c a .
b+c

Bài toán quy v ch ng minh a bc .

. Áp d ng b t đ ng th c và b đ trên ta có:

2 2
a a a a a
= a .
b+c a b+c 2 a bc 2 bc 2

Đ ng th c x y ra khi và ch khi a = b = c = 1 .

Lưu ý 2.10. B t đ ng th c (2.12) trong bài toán trên:

Cho th a mãn ab + bc + ca + abc = 4 . Khi đó ta có:

a + b + c ab + bc + ca .

VMO 1996

Nguy n Quang Huy K49 THPT Chuyên ĐHSP


L Ư U H À N H N Ộ I B Ộ | 85

Đ ng th c x y ra khi và ch khi a = b = c = 1 ho c a, b, c = 2, 2,0 và các hoán v .

Lời giải.

. Không m t tính t ng quát, ta gi s a b c

Áp d ng b t đ ng th c AM - GM ta có:
2
a -1 + b -1 4 a -1 b -1 ab a - 1 b - 1

4-a-b
a + b - ab a + b -1
ab + 1

c ab + 1
a + b - ab a + b -1
ab + 1

a + b + c ab + bc + ca .

Vì ab + bc + ca + abc = 4 nên , trong đó không có 2 s nào đ ng th i b ng 0 th a mãn:

2x 2y 2z
a= ,b = ,c = .
y+z z+x x+ y

Khi đó b t đ ng th c tr thành

x yz
2
y+z z+x x+ y

x3 + 3xyz yz y + z Đúng theo b t đ ng th c Schur ).

Đ ng th c x y ra khi và ch khi a = b = c = 1 ho c a, b, c = 2, 2,0 và các hoán v .

Ví dụ 2.41. Cho . Ch ng minh r ng:

a b c ab + bc + ca
+ + 3 .
a +b b+c c+a 2 a 2 + b2 + c2

Võ Quốc Bá Cẩn-Nguyễn Văn Thạch

Lời giải.

Ta có 1 b đ : N u cho , trong đó không có 2 s nào đ ng th i b ng 0. Khi đó ta có:

3 a 2 + b2 + c 2 2a 2 2b2 2c 2
+ + .
a +b+c a+c b+a c+b

(2.13)

Nguy n Quang Huy K49 THPT Chuyên ĐHSP


L Ư U H À N H N Ộ I B Ộ | 86

Đ ng th c x y ra khi và ch khi a = b = c ho c và các hoán v .

Chứng minh.

Th t v y ta có:

2a 2 a 2 + b2
3 a2 - a =3 a2 - a
a+c a+b

c a2 + b2
= a2 -
a +b
2
ab a - b
=
a+c b+c

0 Đúng .

Đ ng th c x y ra khi và ch khi a = b = c ho c và các hoán v .

Tr l i bài toán chính.

Áp d ng b t đ ng th c Holder và b đ trên ta có:


4
4 a2 a a
2 a =2 4 . . . a a +b a +c
a+c a+b a+b

2
a2 a
2 a a+b a +c
a+c a+b

2
3 a2 a
a a+b a+c .
a a +b

Ta c n ch ng minh
5
4 a 27 bc a a+b a+c .

Do b t đ ng th c là đ ng b c nên không m t tính t ng quát, ta gi s a = 1.

Ta s d ng ph ng pháp q2 3r . B t đ ng th c tr thành

4 27q 1 - 2q + 3r .

Th t v y
2
4 - 27q 1 - 2q + 3r 4 - 27q 1 - 2q + q 2 = 4 - 3q 1 - 3q 0.

. Áp d ng b t đ ng th c AM - GM ta có:

2a 2a 4a
= .
a+b 2a a + b 2a + a + b

Áp d ng b t đ ng th c ta có:

Nguy n Quang Huy K49 THPT Chuyên ĐHSP


L Ư U H À N H N Ộ I B Ộ | 87

2
2a a 4 a
4 .
a +b 2a + a + b a 3a + b

2
4 a ab
Nên ta c n ch ng minh 3 .
a 3a + b a2

ab
Đ tt= 1 . B t đ ng th c tr thành
a2

4 2t 2 + 1
3t .
t2 + 3

Th t v y, ta có:
2
4 2t 2 + 1 1- t 3 1- t + t +1
- 3t = 0.
t +3
2
t +3
2

Đ ng th c x y ra khi và ch khi a = b = c .

Ví dụ 2.42. Cho . Ch ng minh r ng:

11 a + b + c a 3 + b3 + c 3
8 3 abc + 3 3 .
3 3

Lời giải.

Ta có 1 b đ : N u cho . Khi đó ta có:

a 3 + b3 + c 3 8abc 11
+ a+b+c .
a + b + c ab + bc + ca
2 2 2
9

(2.14)

Đ ng th c x y ra khi và ch khi a = b = c .

Chứng minh.

Ta bi n đ i b t đ ng th c v d ng nh sau:
2
2bc b + c + 13a3 - 11abc b - c 0.

Mà áp d ng b t đ ng th c AM - GM ta có:

2
2bc b + c + 13a3 - 11abc 4bc bc + 13a3 - 11abc 3 3 2bc bc .13a 3 - 11abc = 3 3 52 - 11 abc > 0 .

Đ ng th c x y ra khi và ch khi a = b = c .

Tr l i bài toán chính.

Do b t đ ng th c là đ ng b c nên không m t tính t ng quát, ta gi s a = 3.

Nguy n Quang Huy K49 THPT Chuyên ĐHSP


L Ư U H À N H N Ộ I B Ộ | 88

a3
B t đ ng th c tr thành 8 3 abc + 3 3 11 .
3

Áp d ng b t đ ng th c Holder và b đ trên ta có:


3 3
a3 8 3 abc 3 a3
8 abc + 3
3 3 = 27 +
3 3 3

3
8abc 8 8 c3 1
= 27 3 . ab . + 3 . c .2

ab 3 9 c2 3

c3 8abc 8 8 1
27 + c2 + ab +
c2 ab 3 9 3

2
11 11 3 c +2 ab
27. c
9 9 3

113 .

Đ ng th c x y ra khi và ch khi a = b = c .

Ta có 1 bài toán nh sau:

Cho . Ch ng minh r ng:

a 3 + b3 + c 3
3 a +b+c 8 3 abc + 3 .
3

Taiwan TST 2014

Đ ng th c x y ra khi nào ?

Ví dụ 2.43. Cho th a mãn a + b + c = 3 . Ch ng minh r ng:

a 2a + b 2a + c + b 2b + c 2b + a + c 2c + a 2c + b 9.

Lời giải.

. Áp d ng b t đ ng th c Holder ta có:

2 a2 3
a 2a + b 2a + c a = 27 .
2 a + b 2a + c

Bài toán quy v ch ng minh:

a2 1
.
2 a + b 2a + c 3

(2.15)

Th t vây, áp d ng b t đ ng th c ta có:

Nguy n Quang Huy K49 THPT Chuyên ĐHSP


L Ư U H À N H N Ộ I B Ộ | 89

2 2
9a 2 2a + a 2a + a a2 2a a2
= + 2 = + 2 .
2 a + b 2a + c 2a a + b + c + 2a 2 + bc 2a a + b + c 2a + bc a + b + c 2a + bc

a2 a2
2+ .
2a + b 2 a + c 2a 2 + bc

a2
Ta ch c n ch ng minh 1
2a + bc
2

1 a2 1
- 2
2 2a + bc 2

bc
1.
2a 2 + bc

B t đ ng th c này là 1 k t qu r t quen thu c. Ch ng minh còn l i xin dành l i cho b n đ c.

Không m t tính t ng quát, ta gi s a b c .

Ta ch ng minh a 2a + b 2a + c a 3 a

a 2a + b 2a + c - a 3 a 0

a
a-b a -c 0.
2a + b 2a + c + 3a a

Ta có:

a
a -b a -c =
2a + b 2a + c + 3a a

a b
= a-b a-c - b-c
2a + b 2a + c + 3a a 2b + c 2b + a + 3b a

c
+ c - a c -b
2c + a 2c + b + 3a a

a b
a-b a-c - b-c .
2a + b 2a + c + 3a a 2b + c 2b + a + 3b a

Do đó ta ch ng minh

a b
a-c b-c
2a + b 2a + c + 3a a 2b + c 2b + a + 3b a

a -b 3ab a + a-c a 2b + c 2b + a b-c b 2a + b 2a + c .

Ta s ch ng minh b t đ ng th c ch t h n là

Nguy n Quang Huy K49 THPT Chuyên ĐHSP


L Ư U H À N H N Ộ I B Ộ | 90

a-c a 2b + c 2b + a b-c b 2a + b 2a + c .

N u b = c , b t đ ng th c hi n nhiên đúng.

Trong tr ng h p b > c , ta có:

a -c a a
.
b-c b b

Nên ta ph i ch ng minh a 2 2b + c 2b + a b 2 2a + b 2a + c

a 2 2ab + 2bc + ca b 2 2ab + 2ca + bc .

B t đ ng th c này s đúng n u a 2ab + 2bc + ca b 2ab + 2ca + bc .

Th t v y, ta có:

a 2ab + 2bc + ca - b 2ab + 2ca + bc = a - b 2ab + bc + ca 0.

3 3
Đ ng th c x y ra khi và ch khi a = b = c = 1 ho c a, b, c = 0, , và các hoán v .
2 2

Lưu ý 2.11. B t đ ng th c (2.15) trong bài toán trên:

Cho , trong đó không có 2 s nào đ ng th i b ng 0. Khi đó ta có:

a2 b2 c2 1
+ + .
2a + b 2a + c 2b + c 2b + a 2c + a 2c + b 3

Tigran Sloyan

Đ ng th c x y ra khi và ch khi a = b = c ho c và các hoán v .

Ví dụ 2.44. Cho th a mãn ab + bc + ca = 3 . Ch ng minh r ng:

1 1 1 3
3
+ 3
+ 3 3
.
a + kb b + kc c + ka k +1

Trần Quốc Anh

Lời giải.

Áp d ng b t đ ng th c Holder ta có:

3 4
1 3
3
c a + kb c 4
.
a + kb

3 4
3
3
Bài toán quy v ch ng minh c4 3 ab = 9 3 ab 2 .

3 3 3 4 4 4
Đ t x = a4 , y = b4 , z = c4 ab = xy 3 , bc = yz 3 , ca = zx 3 .

Nguy n Quang Huy K49 THPT Chuyên ĐHSP


L Ư U H À N H N Ộ I B Ộ | 91

3
4 4 2
B t đ ng th c tr thành x 9 3 xy 3 .

Do b t đ ng th c là đ ng b c nên không m t tính t ng quát, ta gi s a =3.

4
Khi đó b t đ ng th c vi t l i thành xy 3 3.

Áp d ng b t đ ng th c AM - GM ta có:
4 1
x + y +1 4- z 1
xy 3 = xy. xy 3 xy. = xy. = 4 xy - 3xyz .
3 3 3

Nên ta c n ch ng minh

4 xy - 3xyz 9

3
4 z xy 27 + 9 xyz = z + 9 xyz Đúng theo b t đ ng th c Schur ).

Đ ng th c x y ra khi và ch khi a = b = c = 1 .

BÀI TẬP

74. Cho th a mãn ab + bc + ca = 3 . Ch ng minh r ng:


a2 b2 c2
+ 2 + 2 1.
a +b+c b +c +a c +a +b
2

Vasile Cirtoaje

Lời giải.
Áp d ng b t đ ng th c ta có:
3 2

a2
a2
a2 + b + c a a2 + b + c
3
a3 + 2 bc 2
= .
a3 + 6
3
Bài toán quy v ch ng minh bc 2 3.
Th t v y, áp d ng b t đ ng th c Holder ta có:
3 2 3 2
3
3 bc 2 = 1+1+1 bc 2 bc = 27 .
3
bc 2 3.
Đ ng th c x y ra khi và ch khi a = b = c = 1 .

75. Cho . Ch ng minh r ng:


2 2 2 2
a b c d 4
+ + + .
a +b+c b+c+d c+d +a d +a +b 9
Phạm Kim Hùng

Lời giải.

Nguy n Quang Huy K49 THPT Chuyên ĐHSP


L Ư U H À N H N Ộ I B Ộ | 92

. Áp d ng b t đ ng th c Holder ta có:
2 4 3
a2
2
a a+b+c a 3
.
a ,b ,c , d a+b+c a ,b ,c , d a ,b ,c ,d

4 3

a3
a2 a ,b , c , d
2 2
a ,b ,c, d a +b+c
a a +b+c
a ,b , c , d

4 3

a3
a ,b ,c , d
= 2
2 2
a+c + b+d + a+c b+d
4 4 3

a+c 3 b+d 3
2 +2
2 2
2 2 2
a +c + b+d + a+c b+d
4 4 3

a+c 3 + b+d 3

= 2
.
2 2
2 a+c + b+d + a+c b+d
Bài toán quy v ch ng minh
4 4 3

a+c 3 + b+d 3
4
2
.
2 a+c + b+d
2 2
+ a+c b+d 9

1
a+c 3
Đ tu= . B t đ ng th c tr thành
b+d
3 2
9 u4 +1 8 u6 + u3 + 1 .
1
Ti p t c đ t v = t + 2 . Khi đó b t đ ng th c tr thành
t
3 2
9 v2 - 2 8 v3 - 3u + 1 .
2
v-2 v4 + 4v3 + 6v2 - 8v - 20 0
2
v-2 v4 + 4v 2 v - 2 + 4v v - 2 + 10 v 2 - 2 0 Đúng .
. Do b t đ ng th c trên là đ ng b c nên không m t tính t ng quát, ta gi s a = 1.
a ,b , c , d

B t đ ng th c tr thành
2
a 4
.
a ,b , c , d 1- d 9
Gi s x, y, z, t là 1 b hoán v c a a, b, c, d th a mãn .
Áp d ng b t đ ng th c hoán v ta có:
2 2
a x
.
a ,b , c , d 1- d x , y , z ,t 1- t
2 2 2 2
x t z y
Đ tU= + và V = + .
1- t 1- x 1- y 1- z
Đ t s = x + t, r = y + z và p = xt . Khi đó ta có: s + r = 1 và s, r 0,1 .
Ta có:

Nguy n Quang Huy K49 THPT Chuyên ĐHSP


L Ư U H À N H N Ộ I B Ộ | 93

2
2 p 2 - 2 1 - s 1 - 2s p + s 2 1 - s
U= 2
.
p2 + 2 1 - s p + 1 - s
2
f p = 2 - U p 2 - 2 1 - s 1 - 2s + U p + s 2 - U 1 - s =0.
2 2
D f = 1- s 1 - 2s + U - 2 -U s2 -U 0.

-1 + 4 s - 2 s 2
U 2
.
2-s
-1 + 4r - 2r 2
T ng t , ta có: V 2
.
2-r

-1 + 4 s - 2 s 2 -1 + 4r - 2r 2 5 s2 + r 2 - 2 s4 + r 4
U +V 2
+ 2
= 2
.
2-s 2-r 2 + sr
Nên ta c n ch ng minh
5 s2 + r 2 - 2 s 4 + r 4 4
2
.
2 + sr 9
Th t v y, ta có:
2
5 s2 + r 2 - 2 s 4 + r 4 4 45 s + r - 18 s + r - 4 2 + sr
2 2 4 4

2
- = 2
2 + sr 9 9 2 + sr
2 4 2
45 s 2 + 1 - s - 18 s 4 + 1 - s - 4 2 + s 1- s
= 2
9 2 + sr
2
- 2s - 1 10s 2 - 10s - 11
= 2
9 2 + sr
0.

Đ ng th c x y ra khi và ch khi a = b = c = d .

Nhận xét 2.15. Ta có Bài toán Tổng quát cho bài toán này.
Cho , trong đó không có 3 s nào đ ng th i b ng 0. Ch ng minh r ng:
k k k k
a b c d 4 1
+ + + min 1, , .
a +b+c b+c+d c+d +a d +a+b 3k 2k -1
Võ Quốc Bá Cẩn

Lời giải.
. N u k 1 . Ta có:
k
a a a
=1.
a ,b , c , d a+b+c a ,b , c , d a + b+c a ,b ,c ,d a + b + c+d
. N u 1 k 2.
Áp d ng b t đ ng th c Holder ta có:
k 2k k +1
ak
k
a a +b+c a k +1 .
a ,b , c , d a+b+c a ,b ,c , d a ,b , c , d

2k k +1

a k +1
ak a ,b , c , d
k k
a ,b , c , d a +b+c
a a +b+c
a ,b ,c , d

Nguy n Quang Huy K49 THPT Chuyên ĐHSP


L Ư U H À N H N Ộ I B Ộ | 94

2k k +1

a k +1
a ,b , c , d
= k
2 2
a+c + b+d + a+c b+d
2k 2k k +1

a +c k +1 b+d k +1
2 +2
2 2
2k
(Vì 1)
a +c + b+d
2 2
+ a+c b+d
k
k +1

2k 2k k +1

a +c k +1 b+d k +1
2 +2
2 2
2 2 k
a +c + b+d + a+c b+d
2k 2 k k +1
a+c k +1 + b+d k +1
1
= . .
2k -1 a + c 2 + b + d 2
+ a+c b+d
k

1
a+c k +1
Không m t tính t ng quát, gi s a + c b + d . Đ t t = 1 . Ta có:
b+d
k +1
ak 1 t 2k + 1 1
. = .f t .
a ,b , c , d a +b+c
k
2k -1 t 2 k + 2 + t k +1 + 1 k
2k -1
k
kt k k + 1 t 2 k + 1 t 2 k - 2t k +1 + 2t k -1 - 1
f t = k +1
.
t 2 k +2 + t k +1 + 1
t =0
f t =0 t >0 .

t =0
t >0
.
t 2 k + 2t k -1 - 1
g t = -2=0
t k +1
k - 1 t 2 k - 4t k -1 + k + 1 h t
Ta có: g t = = k +2 .
t k +2 t
h k -2 kt k +1 - 2 0.
Do h t 0 suy ra h t là hàm đ ng bi n. Do đó g t có t i đa 1 nghi m thu c 0,1 . Suy ra
g t có t i đa 1 nghi m thu c 0,1 , trong đó có 1 nghi m là 1. T đây, ta có:
2k +1
f t min f 0 , f 1 = min 1, .
3k
k
a 1 4
min , .
a +b+c 2k -1 3k
. N u k 2.
Áp d ng b t đ ng th c Holder ta có:
k
2 2
a
k
a a +b+c 4
4 a ,b , c , d
.
a ,b , c , d a +b+c 4 3k

Nguy n Quang Huy K49 THPT Chuyên ĐHSP


L Ư U H À N H N Ộ I B Ộ | 95

B t đ ng th c đ c ch ng minh.

Ví dụ 2.45. Cho th a mãn a + b + c = 3 . Ch ng minh r ng:

ab bc ca 1 a +b b+c c +a
+ + + .
b+c c+a a +b 2 2 2

Trần Quốc Anh

Lời giải.

. Áp d ng b t đ ng th c Holder và b t đ ng th c (2.10) ( Ví dụ 2.34) ta có:


2
ab 3
ab b + c ab .
b+c

2 3 3 3
ab ab ab ab
=
b+c ab b + c ab2 + abc + 2abc 2abc + 4

3 2 2
ab ab ab 2 ab
= .
b+c 2 abc + 2 2. ab abc + 2 2 ab + abc + 2

2
4 ab
Bài toán quy v ch ng minh +2 3 ab - abc .
ab + abc + 2

2
4 ab
+ ab + abc + 2 4 ab Đúng theo b t đ ng th c AM - GM ).
ab + abc + 2

. Áp d ng b t đ ng th c AM - GM ta có:

1 1 b+c 3
+ + .
b+c b+c 2 2 2

1 3 b+c 6- b+c
- = .
b+c 2 2 4 2 4 2

ab 6ab - ab b + c
.
b+c 4 2

ab 1 1 1
+ 6ab - ab b + c + .
b+c 2 4 2 2

Bài toán quy v ch ng minh

6ab - ab b + c + 4 2 b+c

6ab - ab b + c + 4 2 a bc - abc

6ab - ab b + c + 4 6 bc - 2abc

ab2 + abc 4 Đúng theo b t đ ng th c (2.10)).

Nguy n Quang Huy K49 THPT Chuyên ĐHSP


L Ư U H À N H N Ộ I B Ộ | 96

Đ ng th c x y ra khi và ch khi a = b = c = 1 ho c a, b, c = 0,1, 2 và các hoán v .

Ví dụ 2.46. Cho th a mãn . Ch ng minh r ng:

Lời giải.

Áp d ng b t đ ng th c Holder ta có:
3 2
a 4b3 a 6b3 a3b3 .

Đ t .

2
Ta c n ch ng minh x2 y xy 27 .

Mà ta có:
2
2 x
x2 y xy x2 y xy = 3 x2 y xy .
3

Nên ta quy bài toán v ch ng minh

x2 y xy 9.

(2.16)

T i đây ta có nhi u h ng gi i.

. Gi s a, b, c là 1 b hoán v c a th a mãn a b c .

Vì ab ca bc nên theo b t đ ng th c hoán v ta có:

x2 y = x.xy + y. yz + z.zx a.ab + b.ca + c.bc = b a 2 + ac + c 2 .

Nên ta c n ch ng minh b a 2 + ac + c 2 ab + bc + ca 9.

Th t v y, áp d ng b t đ ng th c AM - GM ta có:
4
a 2 + ac + c 2 + ab + bc + ca
b a + ac + c
2 2
ab + bc + ca b
2

2 2
b c+a a+b+c
=
4
2
9b c + a
=
4

c+a c+a
= 9.b. .
2 2

Nguy n Quang Huy K49 THPT Chuyên ĐHSP


L Ư U H À N H N Ộ I B Ộ | 97

3
c+a c+a
b+ +
2 2
3

3
9 a+b+c
=
27

=9.

Không m t tính t ng quát, gi s y là s n m gi a và z . Khi đó, ta có:

z x- y y-z 0.

2
x2 y + xyz y z+x

2
x2 y y z + x - xyz = y z 2 + zx + x 2 .

Nên ta ch c n ch ng minh y z 2 + zx + x 2 . xy 9.

Ta ch ng minh

b a 2 + ac + c 2 ab + bc + ca - 9 0

2
b a + c - ac b a + c + ca - 9 0

2
b 3 - b - ac b 3 - b + ca - 9 0 .

2
f ac = -b ac + 2b3 - 9b 2 + 9b ac - b5 + 9b4 - 27b3 + 27b2 - 9 0 .

Nh ng đi u này đúng do
2
D = 2b3 - 9b2 + 9b - 4b b5 - 9b4 + 27b3 - 27b2 + 9

2
= 9b b - 1 b-4

2
< -9b b - 1

<0.

Đ ng th c x y ra khi và ch khi a = b = c = 1 .

Lưu ý 2.12. B t đ ng th c (2.16) trong bài toán trên:

Cho th a mãn . Khi đó ta có:

x 2 y + y 2 z + z 2 x xy + yz + zx 9.

Vasile Cirtoaje

Nguy n Quang Huy K49 THPT Chuyên ĐHSP


L Ư U H À N H N Ộ I B Ộ | 98

Đ ng th c x y ra khi và ch khi .

Ví dụ 2.47. Cho . Ch ng minh r ng:

a 4 + 2b2c 2 b4 + 2c 2 a 2 c 4 + 2a 2b2
+ + a+b+c .
a + 2bc
2
b + 2ca
2
c 2 + 2ab

Jedaihan

Lời giải.

Áp d ng b t đ ng th c Holder ta có:
3
a 4 + b2c 2 + b2c2 a + b + c a + c + b a 2 + bc + bc .

a 4 + 2b 2c 2 a 2 + 2bc
a 2 + 2bc a+b+c

a 4 + 2b2c 2 a 2 + 2bc
= a.
a 2 + 2bc a+b+c

Do b t đ ng th c trên là đ ng b c nên không m t tính t ng quát, ta gi s abc = 1 .

Áp d ng b t đ ng th c Holder ta có:
3
a6 + 1 + 1 a 2 + a + 1 a 2 + 1 + a a a3 + 2 .

a6 + 2 a3 + 2 3
= a -1+ 2
a a3 + 2 a + a +1
2
a + a +1

a6 + 2 1
a +3 -1 .
a a3 + 2 a2 + a +1

1
Bài toán quy v ch ng minh: 1.
a2 + a +1

(2.17)

yz zx xy
Đ t a= ,b = 2 ,c = 2 , v i
x2 y z

x4
1.
x 4 + x 2 yz + y 2 z 2

Th t v y, áp d ng b t đ ng th c ta có:

2 2 2
x4 x2 x2 x2
= =1.
x 4 + x 2 yz + y 2 z 2 x 4 + x 2 yz + y 2 z 2 x 4 + xyz x+ y2 z2 x4 + 2 y2z2

. Ta ch ng minh b t đ ng th c

a 4 + 2b2c 2 a 4 + 2b2c 2 b4 + 2c 2 a 2
+2 a2 + 2 bc
a 2 + 2bc a 2 + 2bc b2 + 2ca

Nguy n Quang Huy K49 THPT Chuyên ĐHSP


L Ư U H À N H N Ộ I B Ộ | 99

3a 2bc a 4 + 2b 2c 2 b4 + 2c 2 a 2
a 2 + bc - +2 a2 + 2 bc
a 2 + 2bc a 2 + 2bc b2 + 2ca

a a 4 + 2b2c 2 b4 + 2c 2 a 2
a2 + bc - 3abc +2 a2 + 2 bc .
a 2 + 2bc a 2 + 2bc b2 + 2ca

Áp d ng b t đ ng th c (2.11) ( Ví dụ 2.39) ta có:

a a 4 + 2b 2c 2 b4 + 2c 2a 2 3abc a a 4 + 2b2c 2 b 4 + 2c 2 a 2
a2 + bc - 3abc +2 a2 + bc - +2
a 2 + 2bc a 2 + 2bc b 2 + 2ca bc a 2 + 2bc b2 + 2ca

a2 .

a 4 + 2b2 c 2 b4 + 2c 2 a 2
Nên ta ch c n ch ng minh bc .
a 2 + 2bc b 2 + 2ca

Áp d ng b t đ ng th c AM - GM và b t đ ng th c ta có:

a 4 + 2b2c 2 b4 + 2c 2 a 2 a 4 + 2b2c 2
33
a + 2bc b + 2ca
2 2
a 2 + 2bc

2
a 2 + 2bc
3
3 3
a 2 + 2bc

=3 a 2 + 2bc .

3
V y ta c n ch ng minh a 2 + 2bc bc

2
b3c3 + 3 abc + 4abc a3 3abc bc b + c .

Th t v y, áp d ng b t đ ng th c Schur và b t đ ng th c AM - GM ta có:
2
b3c3 + 3 abc + 4abc a3 abc bc b + c + 4abc a3

= abc bc b + c + 2abc a 3 + b3

abc bc b + c + 2abc bc b + c

= 3abc bc b + c .

Đ ng th c x y ra khi và ch khi a = b = c .

Lưu ý 2.13. B t đ ng th c (2.17) trong bài toán trên:

Cho th a mãn abc = 1 . Khi đó ta có:

1 1 1
+ 2 + 2 1.
a + a +1 b + b +1 c + c +1
2

Nguy n Quang Huy K49 THPT Chuyên ĐHSP


L Ư U H À N H N Ộ I B Ộ | 100

Võ Quốc Bá Cẩn-Vasile Cirtoaje

Đ ng th c x y ra khi và ch khi a = b = c = 1 .

Đây là 1 bài toán r t ch t và quan tr ng, có nhi u ng d ng giúp ta gi i đ c nhi u bài toán khó và thú v .

Nhận xét 2.16. Ta có Bài toán Tổng quát cho bài toán này.
n
Cho n 3 , a1 , a2 ,..., an > 0 th a mãn ai = 1 và 2 s p, q 0 th a mãn . Khi đó ta có:
i =1

n
1
1.
i =1 1 + pai + qai
2

Vasile Cirtoaje

Ví dụ 2.48. Cho . Ch ng minh r ng:

b2 + c2 3 c2 + a 2 3 a 2 + b2 a+b+c
3 + 2 + 2 .
a 2 + bc b + ca c + ab 3
abc

Phạm Hữu Đức

Lời giải.
3 3
b2 + c 2 a
Ta ch ng minh 3 .
a 2 + bc abc

Áp d ng b t đ ng th c Holder ta có:
3
b2 + c2 1
3 3 b2 + c2
a 2 + bc a + bc
2

1
=6 a2 .
a 2 + bc

3
1 a
Ta c n ch ng minh 6 a 2

a + bc
2
abc

3
a abc
6
a 2
a + bc
2

3
a a a 2 + bc - a3
6
a2 a 2 + bc
3
a a3
+6 6 a.
a 2
a + bc
2

Áp d ng b t đ ng th c ta có:

2 2
a3 a2 a2
= .
a + bc
2
a a 2 + bc a3 + 3abc

Bài toán quy v ch ng minh

Nguy n Quang Huy K49 THPT Chuyên ĐHSP


L Ư U H À N H N Ộ I B Ộ | 101

3 2
a 6 a2
+ 6 a.
a2 a 3 + 3abc

Do b t đ ng th c là đ ng b c nên không m t tính t ng quát, ta gi s a = 1.

3q 2 1 - q
Ta s d ng ph ng pháp r (Do p = a = 1 ).
2 - 3q

B t đ ng th c tr thành
2
1 6 1 - 2q
+ 6.
1 - 2q 1 - 3q + 6r

Ta có:
2 2
1 6 1 - 2q 1 6 1 - 2q
+ +
1 - 2q 1 - 3q + 6r 1 - 2q 3q 2 1 - q
1 - 3q +
2 - 3q

14 - 99q + 264q 2 - 315q3 + 144q 4


= .
1 - 2q 2 - 9q + 12q 2 - 3q 3

V y ta ch c n ch ng minh r ng

14 - 99q + 264q 2 - 315q3 + 144q 4


6
1 - 2q 2 - 9q + 12q 2 - 3q 3
2
2 - 9q + 12q 2 1 - 3q 0 Đúng .

Đ ng th c x y ra khi và ch khi a = b = c .

Lưu ý 2.14. Đánh giá trong bài toán trên:

Đ t .

Ta có
2 2 2
0 P = a -b b-c c-a

= p 2 q 2 - 4q3 + 2 p 9q - 2 p 2 r - 27r 2 = f r .

Do đó ta thu đ c

p 9q - 2 p 2 - 2 p 2 - 3q p 2 - 3q p 9q - 2 p 2 + 2 p 2 - 3q p 2 - 3q
r .
27 27

(2.18)

T đây ta có các k t qu quan tr ng sau:

1) N u f r là hàm đ ng bi n ( f r 0 khi đó ta ch c n ch ng minh

p 9q - 2 p 2 - 2 p 2 - 3q p 2 - 3q
f max 0, 0.
27

(2.19)

Nguy n Quang Huy K49 THPT Chuyên ĐHSP


L Ư U H À N H N Ộ I B Ộ | 102

Hay ta ch c n xét tr ng h p có 2 bi n b ng nhau và tr ng h p p 2 4q thì f 0 0.


2) N u f r là hàm ngh ch bi n ( f r 0 khi đó ta ch c n ch ng minh

p 9q - 2 p 2 + 2 p 2 - 3q p 2 - 3q
f 0.
27

(2.20)
Hay ta ch c n xét tr ng h p có 2 bi n b ng nhau.
3) N u f r là hàm lõm ( f r 0 khi đó ta ch c n ch ng minh

p 9q - 2 p 2 + 2 p 2 - 3q p 2 - 3q p 9q - 2 p 2 - 2 p 2 - 3q p 2 - 3q
min f , f max 0, 0.
27 27

(2.21)
Hay ta ch c n xét tr ng h p có 2 bi n b ng nhau và tr ng h p p 2
4q thì f 0 0.

p 9q - 2 p 2 + 2 p 2 - 3q p 2 - 3q
4) r
27
3
2 p 2 - 3q p 2 - q p p 2 - 3q
1 2
= p 9q - 2 p 2 +
27 3
p p - q
2

2
2
3
p 2 - 3q p2 - q + p 2 p 2 - 3q
1 2
p 9q - 2 p 2 +
27 3
p p2 - q
2

q2 p2 - q
= .
2 p 2 p 2 - 3q
q2 p2 - q
V y, r .
2 p 2 p 2 - 3q
(2.22)
p 9q - 2 p - 2 p - 3q
2 2
p - 3q
2

5) r
27
1 2 p 2 - 3q p 2 - 2q p p 2 - 3q
= p 9q - 2 p 2 -
27 p p 2 - 2q
2

1 2 p 2 - 3q p 2 - 2q + p 2 p 2 - 3q
p 9q - 2 p -
2

27 p p 2 - 2q

4q - p 2 4 p 4 - 10 p 2 q + 3q 2
=
p p 2 - 2q

4q - p 2 4 p 4 - 10 p 2 q + 3q 2
V y, r max 0, .
p p 2 - 2q
(2.23)

Bây gi , ta th xem cách s d ng b t đ ng th c Holder đ gi i m t s bài toán có d ng căn th c Nesbitt .

Nguy n Quang Huy K49 THPT Chuyên ĐHSP


L Ư U H À N H N Ộ I B Ộ | 103

Ví dụ 2.49. Cho . Ch ng minh r ng:

a b c ab bc ca
+ + + + + 3.
b +c c +a a +b b+c c+a c +a a +b a +b b +c

Lời giải.

B t đ ng th c t ng đ ng v i
2
a a
+ 6.
b+c b+c

Áp d ng b t đ ng th c ta có:

2
a a
.
b+c 2 bc

Áp d ng b t đ ng th c Holder ta có:
2 3
a a
.
b+c a2 b + c

2 3
a a
Nên ta ch ng minh + 6.
2 bc a2 b + c

Do b t đ ng th c là đ ng b c nên không m t tính t ng quát, ta gi s a = 1.

4q - 1 1 - q
Ta s d ng ph ng pháp và r max 0, (Do p = a = 1 ).
6

B t đ ng th c tr thành

1 2
+ 12 .
q q - 3r

1
N uq . Ta có:
4

1 2 3
+ 12 .
q q - 3r q

1
N uq . Ta có:
4
2
1 2 1 2 1 - 3q 1 - 4q
+ + = 12 + 12 .
q q - 3r q 4q - 1 1 - q q 4q 2 - 3q + 1
q-
2

Đ ng th c x y ra khi và ch khi a = b = c ho c và các hoán v .

Nguy n Quang Huy K49 THPT Chuyên ĐHSP


L Ư U H À N H N Ộ I B Ộ | 104

Ví dụ 2.50. Cho , trong đó không có 2 s nào đ ng th i b ng 0. Ch ng minh r ng:

a b c ab bc ca 9
+ + + + + .
b+c c+a a +b a +b
2
b+c
2
c+a
2
4

Trần Quốc Anh

Lời giải.

Áp d ng b t đ ng th c Holder ta có:
2
a 3
a2 b + c a .
b+c

3 3
a a a a
= .
b+c a2 b + c a 2 b + c + 2abc bc

M t khác, áp d ng b t đ ng th c ta có:

2 2
ab ab 2ab 1 3 a+b 3 c 3
= + - = - - .
2 a 2 + b2 4 a 2 + b2 4 a 2 + b2
2
a +b 4 4 4 2 c2 4

2
a a
Ta ch ng minh + 3.
bc 2 a2

Áp d ng b t đ ng th c AM - GM ta có:

2 2 4 4 4
a a a a a a a a
+ = + + 33 33 2
= 33 4
=3.
bc 2 a2 2 bc 2 bc 2 a2 8 bc a2 2 bc + a2 a

Đ ng th c x y ra khi và ch khi và các hoán v .

Ví dụ 2.51. Cho , trong đó không có 2 s nào đ ng th i b ng 0. Ch ng minh r ng:

a b c abc
+ + 2 1+ .
b+c c+a a+b (b + c)(c + a)(a + b)

Vũ Đình Quý

Lời giải.

. Áp d ng b t đ ng th c Holder ta có:
2 3 3
a a2 a a
= .
b+c a+b a+c 3 (b + c)(c + a )( a + b ) b+c

Ta c n ch ng minh
3
a abc a2 a bc a2
1+ = . .
b+c b+c a +b a +c b+c a +b a +c

Nguy n Quang Huy K49 THPT Chuyên ĐHSP


L Ư U H À N H N Ộ I B Ộ | 105

2
a a2
Hay .
bc a+b a+c

Do b t đ ng th c này là thu n nh t nên không m t tính t ng quát, ta có th gi s bc = 1 . Đây chính là


bài toán đã đ c trình bày Cách 1 ở Ví dụ 2.22.

. Đ t x = a , y = b , z = c , ta ph i ch ng minh:

x2 x2 y 2 z 2
2 1+ .
y + z2
2
y2 + z 2

Áp d ng b t đ ng th c Holder ta có:
2
x2 3
x y2 + z2 x .
y + z2
2

Do đó, ta c n ch ng minh:
3
x 4 x2 y 2 z 2
4+
x y2 + z2 y2 + z2

3
x 4x2 y2 z 2
-4 .
x y2 + z2 y2 + z2

Vì ta có:
3
x x x - y x - z + 3xyz 3 xyz
-4 = .
x y +z 2 2
x y +z 2 2
x y2 + z2

4x2 y2 z 2 4x2 y 2 z 2 9 xyz. xyz x 9 xyz y2 z 2 9 xyz


= = .
y2 + z2 8
x2 y2 z2 2 x 2 2 2
y z x 2 x 2 2 2
y z x 2 x2 x
9

3 xyz 9 xyz
Nên ta c n ch ng minh r ng: .
x y2 + z2 2 x2 x

2
x+ y x- y 0 Đúng).

. Không m t tính t ng quát, ta gi s c = min a, b, c . Ta ch ng minh b t đ ng th c:

a b a+b
+ 2 .
b+c c+a a + b + 2c

(2.24)

Áp d ng b t đ ng th c Holder ta có:
2
a b 3
+ a 2 b + c + b2 c + a a+b .
b+c c+a

V y ta c n ch ng minh r ng:

Nguy n Quang Huy K49 THPT Chuyên ĐHSP


L Ư U H À N H N Ộ I B Ộ | 106

3
a + b + 2c a + b 4 a + b a 2 b + c + b2 c + a .

Th t v y,
2
4 a + b a 2 b + c + b2 c + a = 4 a + b ab a + b - 2c + c a + b

2 2 3
a+b a+b a + b - 2c + 4c a + b = a + b + 2c a + b .

a b a +b
V y, + 2 .
b+c c+a a + b + 2c

Ta l i có:

abc ab c
1+ = 1+ .
a +b b+c c +a a +b

c 1
= 1+ .
a +b c a +b+c
+1
ab

c 1
1+ .
a + b 4c a b + c
+
+1
( a + b) 2

c (a + b) 2
= 1+ .
a + b a + b + 2c 2

c a+b
= 1+ 2
.
a + b + 2c

a +b c c a+b
Bài toán quy v ch ng minh: 2 + 2 1+ .
a + b + 2c a+b a + b + 2c
2

2c
Đ tt= 1.
a+b

Tr ng h p t = 0 , b t đ ng th c hi n nhiên đúng.

t 2t 2t 2t + 3
Ta xét tr ng h p 0 < t 1 . B t đ ng th c đ c vi t l i thành +2 .
2 t +1 t +1
2

t 4 2t 2t + 3
Hay + 2
.
2 2t t + 1 t +1

Áp d ng b t đ ng th c AM - GM ta có:

t 4 t 8 t 8
+ + + .
2 2t t + 1 2 2 2t t + 1 2 3t + 1

t 8 2t 2t + 3
V y, ta c n ch ng minh + . B t đ ng th c này đúng do
2 3t + 1 t +1
2

Nguy n Quang Huy K49 THPT Chuyên ĐHSP


L Ư U H À N H N Ộ I B Ộ | 107

2
t 8 2t 2t + 3 3t + 5 t - 1
+ - = 0.
2 3t + 1 t +1
2
3t - 5 t + 1
2

Đ ng th c x y ra khi và ch khi a = b = c ho c và các hoán v .

Nhận xét 2.17. Khi ta ti n hành quy đ ng các phân th c và chu n hóa ab + bc + ca = 1 , ta thu đ c Ví dụ
2.22.

Nhận xét 2.18. Ta có m t s bài toán sau c a tác gi Võ Quốc Bá Cẩn là h qu c a bài toán trên:

Cho , trong đó không có 2 s nào đ ng th i b ng 0. Ch ng minh r ng:

a b c abc
a) 3 +3 +3 2 3 1+ .
b+c c+a a+b (b + c)(c + a )(a + b)
a b c 44abc
b) 3 +3 +3 3 8+ .
b+c c+a a+b (b + c)(c + a )(a + b)
a b c 196abc
c) 4 +4 +4 4 16 + .
b+c c+a a+b (b + c)(c + a)(a + b)

Lưu ý 2.15. B t đ ng th c (2.24) trong bài toán trên:

Cho , trong đó không có 2 s nào đ ng th i b ng 0 sao cho c = min a, b, c . Khi đó ta có:

a b a+b
+ 2 .
b+c c+a a + b + 2c

Đ ng th c x y ra khi và ch khi a = b > 0 .

Ví dụ 2.52. Cho , trong đó không có 2 s nào đ ng th i b ng 0. Ch ng minh r ng:

48a 48b 48c


1+ + 1+ + 1+ 15 .
b+c c+a a +b

Vasile Cirtoaje

Lời giải.

. Không m t tính t ng quát, ta gi s c = min a, b, c .

Áp d ng b t đ ng th c Minkowski ta có:

2
48a 48b 2 48a 48b
1+ + 1+ 1+1 + +
b+c c+a b+c c+a

2
a b
= 4 + 48 +
b+c c+a

48.4 a + b 48 a + b
4+ = 2 1+ (Theo b t đ ng th c (2.24) Ví dụ 2.51).
a + b + 2c a + b + 2c

48c 48 a + b
Bài toán quy v ch ng minh 1+ + 2 1+ 15 .
a+b a + b + 2c

Nguy n Quang Huy K49 THPT Chuyên ĐHSP


L Ư U H À N H N Ộ I B Ộ | 108

48c
Đ t 1+ = t +1 0 t 4.
a+b

t 2 + 2t + 1176
B t đ ng th c t ng đ ng v i t +1+ 2 15 .
t 2 + 2t + 24

t 2 + 2t + 1176
2 14 - t
t 2 + 2t + 24

4 t 2 + 2t + 1176 2
14 - t
t 2 + 2t + 24
2
t 18 - t t - 4 0 Đúng).

. Không m t tính t ng quát, ta gi s a b c .

Do b t đ ng th c là đ ng b c nên không m t tính t ng quát, ta gi s a = 1.

B t đ ng th c tr thành

1 + 47a
15 .
1- a

2
.N u c .
27

Ta ch ng minh
2
1 + 47 x 54 7 2
x+ (V i x 1)
1- x 5 5 27

3
12 27 x - 2 3x - 1
0 Đúng .
25 1 - x

1 + 47a 54 7
a+ = 15 .
1- a 5 5

2
.N u c .
27

Ta ch ng minh
2
1 + 47 x 96 1
x+ (V i 0 x 1)
1- x 7 7

3
48 48x + 1 2 x - 1
0 Đúng .
49 1 - x

1 + 47a 96 1 96 1 1 + 47c
a+ + b+ +
1- a 7 7 7 7 1- c

96 1 + 47c
= 14 - c+ .
7 1- c

Bài toán quy v ch ng minh

Nguy n Quang Huy K49 THPT Chuyên ĐHSP


L Ư U H À N H N Ộ I B Ộ | 109

96 1 + 47c
14 - c+ 15
7 1- c

1 + 47c 96
1+ c.
1- c 7

1 + 47c 11
Đ t =t 1 t . B t đ ng th c tr thành
1- c 5

96 t 2 - 1
t 1+
7 t 2 + 47

t - 1 7t 2 - 96t + 233
0 Đúng .
7 t 2 + 47

Đ ng th c x y ra khi và ch khi a = b = c ho c và các hoán v .

BÀI TẬP

76. Cho . Tìm min:


a b c
P = 1+ + 1+ + 1+ .
b+c c+a a+b
77. Cho , trong đó không có 2 s nào đ ng th i b ng 0. Ch ng minh r ng:
16a 16b 16c
1+ + 1+ + 1+ 9.
b+c c+a a+b

Ví dụ 2.53. Cho , trong đó không có 2 s nào đ ng th i b ng 0. Ch ng minh r ng:

a b c 3 ab + bc + ca 7 2
+ + +3 .
b+c c+a a+b a2 + b2 + c2 2

Võ Quốc Bá Cẩn

Lời giải.

Không m t tính t ng quát, ta gi s a = max a, b, c .

Áp d ng b t đ ng th c Holder ta có:
2
b c 3
+ b2 c + a + c2 a + b b+c .
c+a a+b

3
b c b+c
+
c+a a+b b2 c + a + c2 a + b

3
b+c
=
a b 2 + c 2 + bc b + c

Nguy n Quang Huy K49 THPT Chuyên ĐHSP


L Ư U H À N H N Ộ I B Ộ | 110

3
b+c
a b 2 + c 2 + 2abc

3
b+c
= 2
a b+c

b+c
= .
a

V y

b c b+c
+ .
c+a a+b a

(2.25)

3 bc 3a b + c 3a b + c
M t khác, ta có .
a 2 + b2 + c2 a 2 + b2 + c2 a2 + b + c
2

a b+c 3a b + c 7 2
Do đó, ta ch c n ch ng minh + +3 2 .
b+c a a + b+c
2
2

a b+c
Đ t + = x + 2, x 0 .
b+c a

3 7 2
x + 2+3 .
x2 + 4 x + 2 2
2
Ti p t c đ t x + 2 - 2 = t 2 .

3 3 7 2
t2 + 2 + .
t 2

Th t v y, áp d ng b t đ ng th c AM - GM và ta có:

3 3 t2 + 2 6 + 2 3 3 6t + 2 3 3 3 3 3 1 1 7 2
t2 + 2 + = + + = t+ + 3 2+ = .
t 2 2 t 2 2 t 2 t 2 2 2

Đ ng th c x y ra khi và ch khi a, b, c = 3 + 2 2,1,0 và các hoán v .

Nhận xét 2.19. Ta có Bài toán Tổng quát cho bài toán trên.

Cho , trong đó không có 2 s nào đ ng th i b ng 0 và k 0 . Ch ng minh r ng:

a b c ab + bc + ca k
+ + +k min x + .
b+c c+a a+b a2 + b2 + c2 x 2
x2 - 2

Lưu ý 2.16. B t đ ng th c (2.25) trong bài toán trên:

Cho , trong đó không có 2 s nào đ ng th i b ng 0 sao cho a = max a, b, c . Khi đó ta có:

Nguy n Quang Huy K49 THPT Chuyên ĐHSP


L Ư U H À N H N Ộ I B Ộ | 111

b c b+c
+ .
c+a a+b a

Đ ng th c x y ra khi và ch khi a = b = c ho c bc = 0 .

Lời giải.

b c b+c
+
c+a a+b a

b c bc b+c
+ +2
c +a a +b c +a a +b a

bc bc bc
2 +
c +a a +b a a+b a a+c

2a 2a + b + c
bc a+c a+b

2a 2a + b + c
-2 -2
bc a+c a+b

2 2
2a - b - c + b- c a+c - a+b
bc a+c a+b

2
2a - b - c b- c b -c
2

+ 2
bc bc a+c a+b a+c + a+b

2
2a - b - c b - c 1 1
+ 2
- 2
0.
bc bc bc b+ c a+c a+b a +c + a +b

Ta có:

1 1 1 1
2
- 2 2
- 2
bc b+ c a+c a+b a+c + a+b bc b+ c b+c c+c c+c + b+b

3
= 2
4 bc b+ c

>0.

Đ ng th c x y ra khi và ch khi a = b = c ho c bc = 0 .

Ví dụ 2.53. Cho . Ch ng minh r ng:

3
b+c c+a a+b 16 a + b + c
+ + .
a b c 3 a +b b+c c+a

Lời giải.

Nguy n Quang Huy K49 THPT Chuyên ĐHSP


L Ư U H À N H N Ộ I B Ộ | 112

. Áp d ng b t đ ng th c Holder ta có:
2
b+c 2 3 3
a b+c b+c =8 a .
a

3 3
8 a 16 a
Do đó ta c n ch ng minh .
a b+c
2
3 b+c

2
3 b+c 2 a b+c

a bc 9abc Đúng theo b t đ ng th c AM - GM ).

. Áp d ng b t đ ng th c Holder ta có:
2 3
b+c 1 1
.
a a b+c
2
a

3 3
1 16 a 1
Nên ta c n ch ng minh .
a 3 b+c a2 b + c

1 1 1
Đ t a = ,b = ,c = .
x y z

3
3 16 yz x
B t đ ng th c tr thành x .
3 y+z y+z

Áp d ng b t đ ng th c (2.1) ( Bài tập 13) ta quy bài toán v ch ng minh


2
3 6 yz x
x .
x y+z

4
x 1
6 x 2 + 6 xyz .
yz y+z

Áp d ng b t đ ng th c ta có:

1 1 1 1
6 xyz 6 xyz + =3 yz .
y+z 4 y z

4
x
Ta c n ch ng minh 6 x2 + 3 yz .
yz

x 2 - yz
0 Đúng .
yz

B t đ ng th c t ng đ ng v i

a+b a+c 4 a
b+c .
a a 3

Nguy n Quang Huy K49 THPT Chuyên ĐHSP


L Ư U H À N H N Ộ I B Ộ | 113

Đ t .

p
a = p - x, b = p - y , c = p - z , = x+ y+z .
2

Xét có là đ dài 3 c nh t ng ng đ i di n các đ nh p là n a chu vi c a tam giác.

Khi đó đ ý r ng:

a+b a+c yz 1
= = .
a a p p-x cos
X
2

Nên b t đ ng th c c n ch ng minh t ng đ ng v i

x 4p
X 3
cos
2

X sin X
sin
2 3

3 X X X
sin sin cos .
2 2 2 2

Không m t tính t ng quát, ta gi s X Y Z.

X Y Z
sin sin sin
2 2 2
.
X Y Z
cos cos cos
2 2 2

Áp d ng b t đ ng th c ta có:

X X 1 X X
sin cos sin cos
2 2 3 2 2

1 X 3 3
sin .
3 2 2

3 X
= sin .
2 2

Đ ng th c x y ra khi và ch khi a = b = c .

BÀI TẬP

78. Cho . Ch ng minh r ng:


b+c c+a a+b 6 a +b+c
+ + 3
.
a b c abc
79. Cho . Ch ng minh r ng:

Nguy n Quang Huy K49 THPT Chuyên ĐHSP


L Ư U H À N H N Ộ I B Ộ | 114

2
2a 2b 2c a+b+c
+ + .
b+c c+a a+b a 2 + b2 + c2
80. Cho . Ch ng minh r ng:
b+c c+a a+b 6 a +b+c
+ + .
a b c bc + ca + ab
81. Cho . Ch ng minh r ng:
2 b+c 2 c+a 2 a+b 27 a + b b + c c + a
+ + .
a b c 4 a + b + c ab + bc + ca

Nguy n Quang Huy K49 THPT Chuyên ĐHSP


L Ư U H À N H N Ộ I B Ộ | 115

Tr c h t, ta nh c l i m t s l u ý v k thu t đ i bi n s .

Trong khuôn kh bài vi t, ta ch nh c l i v k thu t đ i bi n v i nh ng bài toán b t đ ng th c 3 bi n s


hay đ c s d ng và ph bi n.

Cho .
Không m t tính t ng quát, đ i v i các bài toán đ i x ng hay hoán v v i 3 bi n ta đ u có
th gi s x = min x, y, z .
y = x+a
Khi đó: sao cho .
z = x+b
Phép đ i bi n ngh ch đ o: Cho .
1 1 1
Khi đó: sao cho x = , y = ,z = .
a b c
Đ c biêt, khi ta có , ta s có abc = 1 .
Tr ng h p xyz = k ( k > 0 đ
3
c gi i quy t t ng t .
Cho .
x = am
Khi đó: $ và m R sao cho y = bm .
z = cm
1 1
Thông th ng, ta thng ch n m Q + nh là m = , , 2,3,... ho c m Q - nh là
3 2
(Phép đ i bi n ngh ch đ o) ,... .
Đ c biêt, khi ta có , ta s có abc = 1 .
Tr ng h p xyz = k ( k > 0 đ
3
c gi i quy t t ng t .
Phép đ i bi n ki u hoán v :
Cho th a mãn .
b c a
x = ,y = ,z =
a b c
b a c
x = ,y = ,z =
a c b
Khi đó: sao cho .
a b c
x = ,y = ,z =
b c a
a c b
x = ,y = ,z =
b a c
Các phép đ t trên là t ng đ ng v m t đ i s nh ng ta c n ch n cách đ t sao cho các bi u th c
càng đ c l p v i nhau càng t t, nh v y s d nhìn h n, thu n l i h n khi gi i toán.
Thông th ng, ta th ng s d ng phép đ t này v i nh ng bài toán mà các bi n hoán v . Nh ng
nhi u khi, ta cũng s d ng phép đ t này v i c nh ng bài toán mà các bi n đ i x ng.
Tr ng h p xyz = k 3 ( k > 0 đ c gi i quy t t ng t .
Ng c l i, khi cho và sao cho
b c a
x = , y = ,z =
a b c
b a c
x = ,y = ,z =
a c b
a b c
x = , y = ,z =
b c a
a c b
x = , y = ,z =
b a c

Nguy n Quang Huy K49 THPT Chuyên ĐHSP


L Ư U H À N H N Ộ I B Ộ | 116

thì ta có .
Phép đ i bi n ki u đ i x ng:
Cho th a mãn .
a2 b2 c2
x=
,y = ,z =
bc ca ab
Khi đó: $ sao cho .
bc ca ab
x = 2 ,y = 2 ,z = 2
a b c
Các phép đ t trên là t ng đ ng v m t đ i s nh ng ta c n ch n cách đ t sao cho các bi u th c
càng đ c l p v i nhau càng t t, nh v y s d nhìn h n, thu n l i h n khi gi i toán.
Thông th ng, ta th ng s d ng phép đ t này v i nh ng bài toán mà các bi n đ i x ng.
Tr ng h p xyz = k 3 ( k > 0 đ c gi i quy t t ng t .
Ng c l i, khi cho và sao cho
a2 b2 c2
x= ,y = ,z =
bc ca ab
bc ca ab
x = 2 ,y = 2 ,z = 2
a b c
thì ta có .
Phép đ i bi n Ra - vi :
Cho .
Khi đó: là đ dài 3 c nh c a 1 tam giác
x =b+c
sao cho y = c+a.
z = a+b
Ngoài ra ta còn có m t s ki u đ i bi n khác ph bi n và hay đ c s d ng.
x+ y z
(*) Cho th a mãn y+z x.
z+x y
x =b+c
Khi đó: sao cho y = c+a.
z = a+b
(*) Cho .
Khi đó: là đ dài 3 c nh c a 1 tam giác
a+b
x=
2
b+c
sao cho y=
2
c+a
z=
2
a 2 + b2
x=
2
b2 + c 2
ho c y= .
2
c2 + a2
z=
2
Ngoài ra ta còn có r t nhi u phép đ i bi n khác.
Cho .
Khi đó:

Nguy n Quang Huy K49 THPT Chuyên ĐHSP


L Ư U H À N H N Ộ I B Ộ | 117

b+c
x=
a
c+a
$ sao cho y= .
b
a +b
z=
c
Cho .
Khi đó:
a
x=
b+c
b
sao cho y= .
c+a
c
z=
a +b
Cho th a mãn .
1
a = x+
x
1
Đ t b= y+ .
y
1
c=z+
z
Khi đó ta có: th a mãn .

min a , b , c 2
Ng c l i, n u cho th a mãn .

1
a = x+
x
1
Khi đó: v i th a mãn b= y+ .
y
1
c=z+
z
Cho th a mãn .
Khi đó:
2a
x=
a +b a +c
2b
(*) $ sao cho y= .
b+c b+a
2c
z=
c+a c+b

2 bc
x=
a+b a+c

2 ca
ho c sao cho y= .
b+c b+a

2 ab
z=
c+a c+b

Nguy n Quang Huy K49 THPT Chuyên ĐHSP


L Ư U H À N H N Ộ I B Ộ | 118

x = 2cos A
(*) DABC nh n sao cho .
z = 2cos C

Ví dụ 2.54. Cho . Ch ng minh r ng:

a2 b2 c2
+ + 1.
a 2 + 7ab + b2 b2 + 7bc + c 2 c 2 + 7ca + a 2

Lê Hữu Điền Khuê

Lời giải.

b c a
.Đ t = x, = y, =z xyz = 1 .
a b c

1
B t đ ng th c tr thành 1.
x4 + 7 x2 + 1

1 1
Ta s ch ng minh r ng . B t đ ng th c này đúng do
x4 + 7 x 2 + 1 x2 + x + 1

2 2
x2 + x + 1 - x 4 + 7 x2 + 1 = 2 x x - 1 0.

1 1
1 (Theo b t đ ng th c (2.17) Ví dụ 2.47).
x4 + 7 x 2 + 1 x2 + x + 1

b c a
.Đ t = x, = y, = z xyz = 1 .
a b c

1
B t đ ng th c tr thành 1.
x + 7x +1
2

n2 p 2 p 2 m2 p 2n 2
Đ t x= , y = , z = . B t đ ng th c đ c vi t l i thành
m4 n4 p4

m4
1.
m + 7m n p 2 + n4 p 4
8 4 2

Áp d ng b t đ ng th c Holder ta có:
2
m4 3
m m8 + 7 m 4 n 2 p 2 + n 4 p 4 m3 .
m + 7m n p + n p
8 4 2 2 4 4

Ta c n ch ng minh
3
m3 m m8 + 7m4 n2 p 2 + n4 p 4 .

3 m3n3 m3 + n3 + 6m3n3 p3 7 m5 n 2 p 2 + mn 4 p 4

3 m3n3 p3 7m 2 n 2 p 2 p3 + mnp m3n3 + 3m3n3 p 3 0

7 p3 m3n3 - 3m2 n2 p 2 + m3 n 3 p3 - 9m3n3 p3 + m3 n 3 p3 - 3mnp 0 Đúng .

Nguy n Quang Huy K49 THPT Chuyên ĐHSP


L Ư U H À N H N Ộ I B Ộ | 119

Đ ng th c x y ra khi và ch khi a = b = c .

Nhận xét 2.20. Ta có Bài toán Tổng quát cho bài toán trên.

Cho và k -2 . Ch ng minh r ng:

a2 b2 c2 3
+ 2 + 2 min 1, .
a + kab + b
2 2
b + kbc + c 2
c + kca + a 2 k +2

Lời giải.

Theo ch ng minh bài toán trên thì rõ ràng ta ch c n ch ng minh bài toán này v i k 7.

b c a
Đ t = x, = y, = z xyz = 1 .
a b c

1
B t đ ng th c tr thành 1.
x + kx 2 + 1
4

n2 p 2 p 2 m2 p 2n 2
Đ t x= 4
, y = 4 , z = 4 . B t đ ng th c đ c vi t l i thành
m n p

m4
1.
m8 + km4 n2 p 2 + n4 p 4

Áp d ng b t đ ng th c Holder ta có:
2
m4 3
m m8 + km 4 n2 p 2 + n4 p 4 m3 .
m + km n p + n p
8 4 2 2 4 4

Ta c n ch ng minh
3
k +2 m3 9 m m8 + km4 n 2 p 2 + n4 p 4 .

Th t v y, ta có:
3
k +2 m3 - 9 m m8 + km4 n 2 p 2 + n4 p 4

2 3
=k m3 m3 - 9m 2 n 2 p 2 + 2 m3 -9 m9 - 9 m4n4 p

2 3
7 m3 m3 - 9m 2 n 2 p 2 + 2 m3 - 9 m9 - 9 m4 n 4 p

3
=9 m3 - m m8 + 7m4 n2 p 2 + n4 p 4

0 (Ch ng minh Cách 2 Ví dụ 2.54).

Đ ng th c x y ra khi và ch khi a = b = c .

BÀI TẬP

Nguy n Quang Huy K49 THPT Chuyên ĐHSP


L Ư U H À N H N Ộ I B Ộ | 120

82. Cho th a mãn . Ch ng minh r ng:


1 1 1
+ + 1.
2x + 6x + 1
2
2 y + 6 y +1
2
2z + 6z + 1
2

Nguyễn Văn Thạch

Lời giải.
bc ca ab
Đ t x = 2 , y = 2 , z = 2 , b t đ ng th c tr thành:
a b c
a2
1.
a 4 + 6a 2bc + 2b2c 2
Áp d ng b t đ ng th c Holder ta có:
2
a2 3
a 2 a 4 + 6a 2bc + 2b 2c 2 a2 .
a + 6a bc + 2b c
4 2 2 2

3
Bài toán quy v ch ng minh a2 a 2 a 4 + 6a 2bc + 2b 2c 2
a 2b 2 a 2 + b 2 2abc a3

2
c4 a - b 0 Đúng .

Đ ng th c x y ra khi và ch khi .

83. Cho th a mãn abc = 1 . Ch ng minh r ng:


1 1 1
+ + 1.
3
a 2 + 25a + 1 3
b2 + 25b + 1 3
c 2 + 25c + 1

Lời giải.
Đ t .
B t đ ng th c tr thành
1
1.
x + 25 x 6 + 1
3 6

1 1
Ta ch ng minh b t đ ng th c
3
x + 25 x + 1
6 6 x 2
+ x +1
3
x 6 + 25x 6 + 1 x2 + x + 1
2
3x x - 1 x2 + 4 x + 1 0 Đúng .
1 1
1 (Theo b t đ ng th c (2.17) Ví dụ 2.47).
3
x + 25 x + 1
6 6 x2 + x + 1
Đ ng th c x y ra khi và ch khi a = b = c = 1 .

84. Cho , trong đó không có 2 s nào đ ng th i b ng 0. Ch ng minh r ng:


a2 b2 c2 3
+ 2 + 2 .
a + 14ab + b
2 2
b + 14bc + c 2
c + 14ca + a 2 4

Ví dụ 2.55. Cho th a mãn a + b + c = 1 . Ch ng minh r ng:

2
a 2 + b2 b 2 + c 2 c 2 + a 2 8 a 2b 2 + b 2 c 2 + c 2 a 2 .

Titu Andreescu-Gabriel Dospinescu

Lời giải.

Nguy n Quang Huy K49 THPT Chuyên ĐHSP


L Ư U H À N H N Ộ I B Ộ | 121

1 1 1 1
Đ t m = ,n = , p = =1.
a b c m

Khi đó b t đ ng th c tr thành
2
m2 + n 2 8 p2

2
1 2
m2 + n2 8 p2
p

1
m2 + n2 2 2 p2 .
p

Ti p t c đ t .

p = y + z - x, m = z + x - y , n = y + z - x .

B t đ ng th c tr thành

xyz
z.
x+ y-z

v i là đ dài 3 c nh c a 1 tam giác.

T i đây ta có nhi u h ng gi i.

. Áp d ng b t đ ng th c Schur và b t đ ng th c Radon ta có:

xyz x y + z - x x3

x3
= 2
1
y+z-x

3
x
2
.
1
y+z-x

1
xyz x.
y+z-x

. Ta đ t u = y + z - x, v = z + x - y, w = y + z - x .

Khi đó b t đ ng th c tr thành

u+v 1
w
8 w

2 2
u+v uv 8uvw w

2
u+v u+v w
= .
2 uv 8uvw uv

Vì u + v 2 uv nên áp d ng b t đ ng th c (2.2) ( Ví dụ 2.22) ta có:

Nguy n Quang Huy K49 THPT Chuyên ĐHSP


L Ư U H À N H N Ộ I B Ộ | 122

3
u +v u +v+w w
= .
2 uv 2 uv + w 2 uv + w

Nên bài toán quy v ch ng minh


2
w uv 2 uv + w

2
u- v 0 Đúng .

1
Đ ng th c x y ra khi và ch khi a = b = c = .
3

BÀI TẬP

85. Cho th a mãn . Ch ng minh r ng:


.

Lời giải.
Đ t x = a , y =b ,z =c
2 2 2
x = 3.
B t đ ng th c c n ch ng minh tr thành
x yz .
. Áp d ng b t đ ng th c Holder ta có:
2 3
x x2 x = 27 .
2
Bài toán quy v ch ng minh x2 yz 27 .
Th t v y, áp d ng b t đ ng th c AM - GM ta có:
2
x2 yz = x2 yz yz
3
x2 + yz + yz
3
2 3
x
= = 27 .
3

. Ta ch ng minh b t đ ng th c
2
2 x+ x2 x =3 x.
B t đ ng th c này đúng do áp d ng b t đ ng th c AM - GM ta có:
2
2 x + x2 33 x .x 2 = 3x .

Đ ng th c x y ra khi và ch khi a = b = c = 1 .

86. Cho th a mãn . Ch ng minh r ng:


x+2+ y+2+ z+2 3 3 .

Lời giải.

Nguy n Quang Huy K49 THPT Chuyên ĐHSP


L Ư U H À N H N Ộ I B Ộ | 123

1
Đ t ab = abc hay = 1.
a
B t đ ng th c c n ch ng minh tr thành
a 3 3.
Th t v y, áp d ng b t đ ng th c Holder ta có:
2 2 1
a = a 27 .
a

Đ ng th c x y ra khi và ch khi .

87. Cho th a mãn abc = 1 . Ch ng minh r ng:


3
a+3b+3c 3 3 3 + a + b + c + ab + bc + ca .
Cezar Lupu

Lời giải.
x2 y2 z2
Đ t a = ,b = ,c = ( ).
yz zx xy
B t đ ng th c tr thành
x2 x2 yz
3 3 3 3+ + .
yz yz x2
Th t v y, áp d ng b t đ ng th c Holder ta có:
x2 x x x x x2 yz
3 = 3 1. . 3 3 = 3 3 3+ + .
yz y z y z yz x2
Đ ng th c x y ra khi và ch khi a = b = c = 1 .

88. Cho th a mãn . Ch ng minh r ng:


x+ y y+z z+x 3
+ + .
x2 + y 2 + 6 z y2 + z2 + 6x z 2 + x2 + 6 y 2

Lời giải.
Ta có:
x+ y x+ y
=
x2 + y 2 + 6 z x + y +2
2 2
z z
x+ y
=
z + 2 zx + x + y 2 + 2 yz + z 2
2 2

2
x+ y
= 2 2
.
y+z + z+x
2 2 2
Đ t y+z = a, z + x = b, x + y =c ( ) a =6.
B t đ ng th c tr thành
a 3
.
b+c 2
Áp d ng b t đ ng th c Holder ta có:
2
a 3
a2 b + c a .
b+c
Bài toán quy v ch ng minh
3
2 a 9 a2 b + c .

Nguy n Quang Huy K49 THPT Chuyên ĐHSP


L Ư U H À N H N Ộ I B Ộ | 124

B t đ ng th c này đúng. Ch ng minh còn l i r t đ n gi n, xin nh ng l i cho b n đ c.


Đ ng th c x y ra khi và ch khi .

Ví dụ 2.56. Cho a, b, c là đ dài 3 c nh c a 1 tam giác. Ch ng minh r ng:

a b c 5
+ + .
13 13 13 2
a 2 + bc b 2 + ca c 2 + ab
5 5 5

Lời giải.

Áp d ng b t đ ng th c Holder ta có:
2
a 3
a 5a 2 + 13bc a .
5a + 13bc
2

3 1
Bài toán quy v ch ng minh a a 5a 2 + 13bc .
2

Vì a, b, c là đ dài 3 c nh c a 1 tam giác nên sao cho .

Khi đó b t đ ng th c tr thành

x3 + 3xyz yz y + z (B t đ ng th c Schur ).

Đ ng th c x y ra khi và ch khi a = b = c .

Ví dụ 2.57. Cho DABC . Ch ng minh r ng:

sin A sin B sin B sin C sin C sin A


+ + 9.
C A B
sin 2 sin 2 sin 2
2 2 2

CĐT Olympic 30-4 THPT Chuyên Lương Thế Vinh Đồng Nai 2014

Lời giải.

A B C
Đ t a = tan , b = tan , c = tan ( ). Ta có: ab = 1 .
2 2 2

2a A a
sin A = ,sin =
1 + a2 2 1 + a2
2b B b
Và sin B = ,sin = .
1 + b2 2 1 + b2
2c C c
sin C = ,sin =
1+ c 2
2 1 + c2

Đ ý r ng

sin A sin B 4ab 1 + c 2 4ab


= = .
c 1+ a 1+ b
2
C 2 2 2
c a +b
2
sin 2
2

Nên ta ch ng minh

Nguy n Quang Huy K49 THPT Chuyên ĐHSP


L Ư U H À N H N Ộ I B Ộ | 125

ab 9
2
.
c2 a + b 4

Áp d ng b t đ ng th c Holder ta có:

ab 2 3
2
abc a + b ab = 1 .
c a +b
2

ab 1 1 1 9
2 2
= 2 2
= .
c a +b
2
abc a + b 4 abc a bc
2 4
4
3

Đ ng th c x y ra khi và ch khi DABC là tam giác đ u.

Ví dụ 2.58. Cho . Ch ng minh r ng:

1 1 1 3
+ + .
a a +b b b+c c c +a 2abc

Phan Thành Nam

Lời giải.

. Do b t đ ng th c là đ ng b c nên không m t tính t ng quát, ta gi s abc = 1 . Khi đó,


x z y
sao cho a = ,b = ,c = .
y x z

B t đ ng th c tr thành

y y 3
.
x x + yz 2 2

. Áp d ng b t đ ng th c Holder ta có:
2

y y 2 4
xy x 2 + yz y .
x x + yz
2

Nên ta ch c n ch ng minh
4
2 y 9 xy x 2 + yz

2
2 x2 + 2 yz 9 xy x2 + yz

x2 - yz 2 x2 + yz 0 Đúng .

. Áp d ng b t đ ng th c ta có:

2
y y y
x x 2 + yz xy x 2 + yz

Nguy n Quang Huy K49 THPT Chuyên ĐHSP


L Ư U H À N H N Ộ I B Ộ | 126

2
y
xy x 2 + yz

2
z
=
1
2 xy z2 + xy
2
2
z
1
z2 + xy + 2 xy
2 2
2
z
=
1 2
z + xy
2 2
2
z
2
1 2 z
z +
2 2 3

2
z
=
2 2
z
3 2

3
= .
2

. Ta ch ng minh r ng
2
1 9
.
a a +b 2abc

2
1 1
Mà ta có: 3 .
a a +b ab a +b b +c

Do đó ta ch c n ch ng minh

1 9
3
ab a+b b+c 2abc

c 3
.
a+b b+c 2

Áp d ng b t đ ng th c AM - GM và b t đ ng th c ta có:

c 2c
a +b b+c a +b + b+c

2
2 c
c a + 2b + c

Nguy n Quang Huy K49 THPT Chuyên ĐHSP


L Ư U H À N H N Ộ I B Ộ | 127

2
2 c
= 2
c + ab

2
2 c
2
2 c
c +
3

3
= .
2

Đ ng th c x y ra khi và ch khi a = b = c .

Ví dụ 2.59. Cho th a mãn abc = 1 . Ch ng minh r ng:

ab + bc + ca
4
2a 2 + bc + 4 2b 2 + ca + 4 2c 2 + ab 4
a+ b+ c .
3

Lời giải.

1 1 1
Đ t a = ,b = ,c = . Khi đó b t đ ng th c tr thành
x y z

x 2 + 2 yz x 1
4
4
.
x 3 x

4 4 2
x 2 + 2 yz x 1
4 .
x 3 x

Áp d ng b t đ ng th c Holder ta có:

4 4
2 2
x 2 + 2 yz 1 1
4 = 4 x 2 + 2 yz . .1 3 x 2 + 2 yz
x x x

2
1 2
=3 x .
x

4
2 x
Nên bài toán quy v ch ng minh 3 x .
3

x 3 Đúng theo b t đ ng th c AM - GM ).

Đ ng th c x y ra khi và ch khi a = b = c = 1 .

Ví dụ 2.60. Cho th a mãn . Ch ng minh r ng:

2
1 1 1
3 + + x+2 y+2 z+2 .
x y z

Gabriel Dospinescu

Nguy n Quang Huy K49 THPT Chuyên ĐHSP


L Ư U H À N H N Ộ I B Ộ | 128

Lời giải.

x y y z z x x y z
. xy + yz + zx + xyz = 4 . + . + . + 2 . . = 1.
2 2 2 2 2 2 2 2 2

2a 2b 2c
th a mãn x = ,y= ,z = . Khi đó, b t đ ng th c tr thành:
b+c c+a a+b
2 3
b+c 16 a
3 .
a b+c

Đây chính là Ví dụ 2.53 đã đ c gi i b ng b t đ ng th c Holder .

.Đ t .

2bc 2ca 2ab


và x = ,y= ,z = .
a b c

B t đ ng th c tr thành
2
3 a 16 a + bc .

M t khác, ta th y r ng DABC nh n sao cho .

Ta g i p là n a chu vi, r là bán kính đ ng tròn n i ti p, R là bán kính đ ng tròn ngo i ti p DABC .

Đ ý r ng:

c + ab = cos C + cos A cos B

= - cos A + B + cos A cos B

= sin A.sin B .

Khi đó, b t đ ng th c tr thành

3 cos A 4 sin A .

Ta s d ng m t s đăng th c quen thu c.

A
p = 4R cos
2
A
r = 4R sin . B t đ ng th c t ng đ ng v i
2
r
cos A = 1 +
R

r+R 2rp
3
R R2

R + r R 3 2rp .

3 3
S d ng b t đ ng th c Euler và k t qu quen thu c p R ta thu đ c đi u ph i ch ng
2
minh.

Nguy n Quang Huy K49 THPT Chuyên ĐHSP


L Ư U H À N H N Ộ I B Ộ | 129

Đ ng th c x y ra khi và ch khi .

Ví dụ 2.61. Cho . Ch ng minh r ng:

2 2
x2 + y 2 y2 + z 2 z 2 + x2 xy + yz + zx 8x 2 y 2 z 2 x 2 + y 2 + z 2 .

Lời giải.

B t đ ng th c t ng đ ng v i
2
1 2
x2 + y2 . 8 z2 .
z

Đ t là đ dài 3 c nh c a 1 tam giác.

Và .

2
1 2
B t đ ng th c trên tr thành abc a .
b+c-a

Do b t đ ng th c là đ ng b c nên không m t tính t ng quát, ta gi s a = 1.

Áp d ng b t đ ng th c Holder ta có:
2
1 3
a3 b + c - a a .
b+c-a

V y ta c n ch ng minh

abc a3 b + c - a

a2 a - b a - c 0 Đúng theo b t đ ng th c Schur ).

Đ ng th c x y ra khi và ch khi .

Ví dụ 2.62. Cho th a mãn abc = 1 . Ch ng minh r ng:

a b c 3
+ + .
b +3
2
c +3
2
a +3
2 2

Lời giải.

x z y
Đ t a= ,b = , c = .
y x z

x 3
Khi đó b t đ ng th c tr thành .
3xy + yz 2

Áp d ng b t đ ng th c Holder ta có:
2
x 3
x 3xy + yz x .
3 xy + yz

Nguy n Quang Huy K49 THPT Chuyên ĐHSP


L Ư U H À N H N Ộ I B Ộ | 130

3 27
Bài toán quy v ch ng minh x x 2 y + xyz .
4

Đây chính là b t đ ng th c (2.10) ( Ví dụ 2.34).

. Áp d ng b t đ ng th c AM - GM và b t đ ng th c ta có:

a a
b +3
2
b +
2
ca

a
=
b+a b+c

2a
a + 2b + c
2
2 a
a a + 2b + c

3
(Ch ng minh Cách 2 Ví dụ 2.58).
2

Đ ng th c x y ra khi và ch khi a = b = c = 1 .

Ví dụ 2.63. Cho th a mãn abc = 1 . Ch ng minh r ng:

a b c 3
1. + + .
b+3 c+3 a+3 2

a b c 3
2. 3 +3 +3 .
b+7 c+7 a+7 2

3. (Bài toán Tổng quát).

Cho th a mãn và k 0 . Ch ng minh r ng:

x y z 3
3 +3 +3 .
y+k z+k x+k 3
k +1

Vasile Cirtoaje

4. Cho th a mãn và k 0 . Ch ng minh r ng:

x y z 3
4 +4 +4 .
y+k z+k x+k 4
k +1

Võ Quốc Bá Cẩn

Lời giải.

x z y
a) Đ t a = , b = , c = . B t đ ng th c tr thành
y x z
x 3
.
y 3x + z 2

Nguy n Quang Huy K49 THPT Chuyên ĐHSP


L Ư U H À N H N Ộ I B Ộ | 131

Áp d ng b t đ ng th c Holder ta có:
2
x 3
xy 3x + z z .
y 3x + z
Ta ch c n ch ng minh:
3
4 a 27 a 2b + abc .
Đây chính là b t đ ng th c (2.10) ( Ví dụ 2.34).

Đ ng th c x y ra khi và ch khi a = b = c = 1 .

x4 z4 y4
b) Đ t a = 4
, b = 4 , c = 4 . B t đ ng th c tr thành
y x z
x8 3
3 .
y 7 x4 + z 4
4
2
Áp d ng b t đ ng th c Holder ta có:
3
4
x8 x2
3 7 x4 + z 4 .
y 7 x4 + z 4
4
y

Ta ch c n ch ng minh:
4
x2
27 z4 .
y
(2.26)
Do b t đ ng th c này là thu n nh t nên không m t tính t ng quát, ta gi s z 4 = 3 . V y, ta c n
ch ng minh
x2
3.
y
Áp d ng b t đ ng th c Holder ta có:
2
x2 3
x2 y 2 x2 .
y
3
T ng đ ng v i vi c ch ng minh x2 9 x2 y2
3
x2 3 x2 y2 3 z4 .

Đ t a= x2 , b = y2 z2 .

a3 3b 3 a 2 - 2b
2
a 2 - 3b a 2 + 6b 0 Đúng).
Đ ng th c x y ra khi và ch khi a = b = c = 1 .

a4 c4 b4
c) Đ t x= , y = , z = ( ). B t đ ng th c tr thành
b4 a4 c4
8
a3 3
4
.
3
k +1
b 3 3
c 4 + ka 4
Áp d ng b t đ ng th c Holder ta có:
8 3
4
a3 a2
4
c 4 + ka 4 .
b
b 3 3
c + ka
4 4

Nguy n Quang Huy K49 THPT Chuyên ĐHSP


L Ư U H À N H N Ộ I B Ộ | 132

Nên ta c n ch ng minh
4
a2
27 a4
b

a2 a4
34 Đúng .
b 3
Đ ng th c x y ra khi và ch khi .

a5 c5 b5
d) Đ t x = 5
, y = 5 ,z = 5 ( ). B t đ ng th c tr thành
b a c
5
a2 3
.
k +1
5 4
b 4 4
c + ka
5 5

Áp d ng b t đ ng th c Holder ta có:
5 4
5
a 2
a2
5
c5 + ka5 .
b
b 4 4
c + ka
5 5

Nên ta c n ch ng minh
5
a2
81 a5
b

a2 a5
35 .
b 3
(2.27)
Do b t đ ng th c là đ ng b c nên không m t tính t ng quát, ta gi s a = 1 . Khi đó ta c n ch ng
minh
a2
3.
b

Ta có 1 b đ : N u cho . Khi đó ta có:


a b c2
2 2 6 a 2 + b2 + c 2 - 3 ab + bc + ca
+ + .
b c a a +b+c
Võ Quốc Bá Cẩn
(2.28)
Đ ng th c x y ra khi và ch khi a = b = c .

Chứng minh.

. Không m t tính t ng quát, ta gi s a = min a, b, c . Khi đó, đ t b = a + x và c = a + y v i


. B t đ ng th c tr thành
x 2 - xy + y 2 a 3 + 3xy 2 y - x a + x4 - 5x3 y + 6 x2 y 2 + xy 3 + y 4 0.
Ta s ch ng minh
f a = x 2 - xy + y 2 a 2 + 3xy 2 y - x a + x 4 - 5x 3 y + 6 x 2 y 2 + xy 3 + y 4 0.
Ta có:
D f = g x = - 4 x 6 - 24 x 5 y + 39 x 4 y 2 - 4 x 3 y 3 - 12 x 2 y 4 + 4 y 6 .
N u x 3 y . Ta có:
g x = -12 x x - 2 y 2 x 2 x - 3 y + xy 2 + y 3 0.
Df = g x g 3 y = -31y 6
0.

Nguy n Quang Huy K49 THPT Chuyên ĐHSP


L Ư U H À N H N Ộ I B Ộ | 133

N u x 3 y . Áp d ng b t đ ng th c AM - GM ta có:
2 2
D f = g x = - 2 x3 - 6 x 2 y + xy 2 + y 3 + x3 y 2 3 y - x + y 3 x3 + 4 y 3 - y x + y
2
- y 3 x3 + 4 y 3 - y x + y
3
x+ y 2
- y3 + 3 y3 - y x + y
4
3 3
x+ y x+ y 2
- y3 3 3 . .3 y 3 - y x + y
8 8

33 3 2
= - y3 -1 y x + y
4
0.
. Không m t tính t ng quát, ta gi s a b c .
Ta ch ng minh
a2 6 a2 - 3 bc
- a - a
b a
2 2
a +b a-b b+c a-c b-c 5 a -b +5 a-c b-c
+
ab ac a
a+b 5 2 b+c 5
- a -b + - a-c b-c 0.
ab a ac a
a -b
N u b -c .
2
2
3 a -b 3 a -b
a-c a-c b-c .
2 4
2 a +b 5 b+c 5
a -b 4 - +3 -
a+b 5 2 b+c 5 ab a ac a
- a -b + - a-c b-c .
ab a ac a 4
Do đó ta ch c n ch ng minh
a +b 5 b+c 5
4 - +3 - 0
ab a ac a
3b 2 + 7bc + 4ca a + b + c 35abc .
Th t v y, áp d ng b t đ ng th c AM - GM ta có:
7bc 7bc a a a
3b2 + 7bc + 4ca a + b + c 5 5 3b2 . . .2ca.2ca 55 . . .bc
2 2 3 3 3

147
= 25 5
abc
27
> 35abc .
a -b
N u b -c .
2
2 a-c 2 4 a -c b-c
a -b a -b .
3 3
a +b 5 b+c 5
a -c b-c 4 - +3 -
a +b 5 2 b+c 5 ab a ac a
- a-b + - a-c b-c
ab a ac a 3
Do đó ta ch c n ch ng minh

Nguy n Quang Huy K49 THPT Chuyên ĐHSP


L Ư U H À N H N Ộ I B Ộ | 134

a+b 5 b+c 5
4 - +3 - 0.
ab a ac a
Đ ng th c x y ra khi và ch khi a = b = c .
Tr l i bài toán chính.
Áp d ng b đ ta quy bài toán v ch ng minh
6 a2 - 3 bc a5
33 .
a 3
1 - q2
Đ t bc = q 0,1 .
3
2
1- q
1 + 2q
Và abc = r r (Do a = 1 ). Xem bi n đ i b t đ ng th c (2.18).
27
1
Đ ý r ng: a5 = 15 q 2 + 2 r + 35q 4 - 25q 2 - 1 .
9
B t đ ng th c tr thành
5
9 15 q 2 + 2 r + 35q 4 - 25q 2 - 1 5q 2 + 1 .
Ta có:
2
5 q2 + 2 1 - q 1 + 2q
9 15 q 2 + 2 r + 35q 4 - 25q 2 - 1 9 + 35q 4 - 25q 2 - 1 .
9
Nên ta c n ch ng minh
2
5 5 q2 + 2 1 - q 1 + 2q
5q + 1
2
9 + 35q 4 - 25q 2 - 1
9

Đúng .
Đ ng th c x y ra khi và ch khi .

Nhận xét 2.21. Ta có Bài toán Mở rộng cho bài toán (c) trên.

Cho th a mãn abcd = 1 . Ch ng minh r ng:

a b c d 4
4 +4 +4 +4 .
b+k c+k d +k a+k 4
k +1

Nhận xét 2.22. Ta có các bài toán nh sau:

a) Cho . Ch ng minh r ng:

a 2 b2 c2 6 a 2 + b2 + c 2
+ + +a+b+c .
b c a a+b+c

Phạm Hữu Đức

Đ ng th c x y ra khi và ch khi a = b = c .

3 bc
Ta th y r ng b t đ ng th c này là 1 h qu c a b đ trong bài toán (d) trên do a .
a

b) Cho . Ch ng minh r ng:

ab 2 bc 2 ca 2 6 a 2 + b2 + c2
+ 2 + 2 +a+b+c .
c2 a b a +b+c

Nguy n Quang Huy K49 THPT Chuyên ĐHSP


L Ư U H À N H N Ộ I B Ộ | 135

Đ ng th c x y ra khi nào ?

Nhận xét 2.23. Ta có Bài toán Tổng quát cho các bài toán trên:

Cho th a mãn abc = 1 , 0 m 5 và k 0 . Ch ng minh r ng:

a b c 3
m +m +m .
b+k c+k a+k m
k +1

Đ ng th c x y ra khi và ch khi a = b = c = 1 .

Lưu ý 2.17. B t đ ng th c (2.26) trong bài toán trên:

Cho . Khi đó ta có:

a 2 b2 c 2 a 4 + b4 + c 4
+ + 34 .
b c a 3

Alexey Gladkich

Đ ng th c x y ra khi và ch khi a = b = c .

Lời giải.

Do b t đ ng th c này là thu n nh t nên không m t tính t ng quát, ta gi s a4 = 3 a, b, c < 4 3 < 2 .

V y, ta c n ch ng minh

a2
3.
b

Ta có 1 b đ : N u cho . Khi đó ta có:

a 2 b2 c2 3 a 3 + b3 + c 3
+ + .
b c a a 2 + b2 + c 2

(2.29)

Đ ng th c x y ra khi và ch khi a = b = c .

Chứng minh.

Vi t b t đ ng th c trên v d ng nh sau:

a 2 + b2 2
-b b -c 0.
c

. N u a b c.

a 2 + b2 c2 + a2
Ta d ch ng minh -b , -a 0.
c b

Ta có:

a 2 + b2 b2 + c2 a 2 b2 b b - c b2 + 2c 2
-b + 2 - c = + - 2c + +
c a c a c a

Nguy n Quang Huy K49 THPT Chuyên ĐHSP


L Ư U H À N H N Ộ I B Ộ | 136

a 2 b2
+ - 2c
c a

a 2 b2
+ - 2c
b a
2
a+b
- 2c
a +b

= a + b - 2c

0.

N u a2 2b 2 . Ta có:

c2 + a 2 b2 + c 2 a - b a 2 - 2b2 c 2 2c 2
-a +2 -c = + + +2 b-c
b a ab b a

0.

N u a2 2b 2 . Ta có:

c2 + a2 b2 + c2 a a - b c 2 2c 2 2b2
-a +2 -c = + + + - 2c .
b a b b a a

N u a 2c . Ta có:

a a - b c 2 2c 2 2b 2 2b2
+ + + - 2c - 2c
b b a a a

a - 2c

0.

N u a 2c . Ta có:

a a - b c 2 2c 2 2b 2 a a - b a 2 a 2b2
+ + + - 2c + + + - 2c
b b a a b 4b 2 a

a - b 5a - 4b 3a b2
= + + + b - 2c
4ab 4 a

3a a
+ -c
4 2

0.

N u c b a.

b2 + c 2 c2 + a2
Ta d ch ng minh -c , -a 0.
a b

Ta có:

b2 + c2 a 2 + b2 b b - a + c c - a a 2 + b2
-c + -b = +
a c a c

0.

Nguy n Quang Huy K49 THPT Chuyên ĐHSP


L Ư U H À N H N Ộ I B Ộ | 137

Đ ng th c x y ra khi và ch khi a = b = c .

Tr l i bài toán chính.

Áp d ng b đ ta quy bài toán v ch ng minh

a3 a2 .

Th t v y, ta có:

4 a3 - a2 = 4 a3 - a2 - a4 - 1

2
= a -1 1 + 2a - a 2

0.

Đ ng th c x y ra khi và ch khi a = b = c .

Bài toán này cũng là m t ví d đi n hình cho k thu t s d ng b t đ ng th c Holder .

Lưu ý 2.18. B t đ ng th c (2.27) trong bài toán trên:

Cho . Khi đó ta có:

a 2 b2 c2 a 5 + b 5 + c5
+ + 35 .
b c a 3

Đ ng th c x y ra khi và ch khi a = b = c .

Nhận xét 2.24. Ta có Bài toán Tổng quát cho 2 bài toán trên.

21
Cho và m 0, . Ch ng minh r ng:
4

a 2 b2 c 2 a m + bm + c m
+ + 3m .
b c a 3

Đ ng th c x y ra khi và ch khi a = b = c .

Nhận xét 2.25. Ta có m t s bài toán sau:

a) Cho th a mãn . Ch ng minh r ng:


a b c2 d 2
2 2
+ + + 4.
b c d a

Lời giải.
Áp d ng b t đ ng th c Holder ta có:
2 3
a2
a 2b 2 a2 = 64 .
a ,b , c , d b a ,b ,c , d a ,b , c , d

a2 64
a ,b , c , d b a 2b 2
a ,b , c , d

64
=
a 2 + c 2 b2 + d 2

Nguy n Quang Huy K49 THPT Chuyên ĐHSP


L Ư U H À N H N Ộ I B Ộ | 138

64
2
a + c + b2 + d 2
2 2

4
= 4.
Đ ng th c x y ra khi và ch khi a = b = c = d = 1.

b) (Bài toán Mở rộng).


Cho th a mãn . Khi đó ta có:
a b c2 d 2
2 2
+ + + 4.
b c d a
Phạm Kim Hùng
Đ ng th c x y ra khi và ch khi a = b = c = d = 1.
c) Cho th a mãn . Ch ng minh r ng:
a 2 b2 c 2 d 2
+ + + 4.
b c d a
d) Cho th a mãn . Ch ng minh r ng:
a 2 b2 c 2
+ + 3.
b c a
Võ Quốc Bá Cẩn
e) Cho th a mãn . Ch ng minh r ng:
a 5 b5 c 5
+ + 3.
b c a
Trần Quốc Anh
Xem Ví dụ 2.82.
f) Cho , trong đó không có 2 s nào đ ng th i b ng 0. Ch ng minh r ng:
a2 b2 c2 3 4 a 4 + b4 + c4
+ + .
b+c c+a a+b 2 3
Alexey Gladkich

Lời giải.
Do b t đ ng th c là đ ng b c nên không m t tính t ng quát, ta gi s a 4 = 3 . Khi đó ta c n
ch ng minh
a2 3
.
b+c 2
Áp d ng b t đ ng th c Holder ta có:
2
a2 2 3
a2 b + c a2 .
b+c
Nên ta ch c n ch ng minh
3 9 2
a2 a2 b + c .
4
3
Mà a2 9 b2c 2 (Ch ng minh phần b Ví dụ 2.63).
Do đó ta quy bài toán v ch ng minh
2
4 b2c 2 a2 b + c
2
a2 b - c 0 Đúng .
Đ ng th c x y ra khi và ch khi a = b = c = 1 .

g) Cho . Ch ng minh r ng:

Nguy n Quang Huy K49 THPT Chuyên ĐHSP


L Ư U H À N H N Ộ I B Ộ | 139

a2 b2 c2 3 5 a 5 + b5 + c 5
+ + .
b+c c+a a+b 2 3
Michael Rozenberg

2
Ví dụ 2.64. Cho th a mãn x + y + z + xy + yz + zx = 2 . Ch ng minh r ng:

x+ y y+z z+x 3 2
+ + .
z + xy + 1
2
x + yz + 1
2
y + zx + 1
2 2

Lời giải.

Đ t và ab = 2 .

B t đ ng th c tr thành

a 3
.
a + 3bc
2 2

Không m t tính t ng quát, ta gi s a b c.

Áp d ng b t đ ng th c Holder ta có:
2
a 3
a a 2 + 3bc a .
a + 3bc
2

Bài toán quy v ch ng minh


3
4 a 9 a a 2 + 3bc .

1 1 3 1
Đ ng th c x y ra khi và ch khi x = y = z = ho c x, y, z = , , và các hoán v .
6 4 4 4

1 1 1 1
Ví dụ 2.65. Cho th a mãn + + = . Ch ng minh r ng:
a 2 + 47 b2 + 47 c 2 + 47 24

47
a + b + c 10 .
23

Yongyao

Lời giải.

1 1 1 1 1 1- x 1 1- y 1 1- z
Vì , , , ta đ t 2 = , = , = .
a 2 + 47 b2 + 47 c 2 + 47 47 a + 47 47 b 2 + 47 47 c 2 + 47 47

25
và x= .
24

B t đ ng th c c n ch ng minh tr thành

x 10
.
1- x 23

Không m t tính t ng quát, ta gi s z = min x, y, z .

Nguy n Quang Huy K49 THPT Chuyên ĐHSP


L Ư U H À N H N Ộ I B Ộ | 140

2 2
x+ y x+ y+z > .
3 3
2 2 3
3x + 3 y - 2 x - y x+ y x+ y
0 = - - x 2 + y 2 - x3 + y 3
4 2 4

2 3
x+ y x+ y
= - - x2 1 - x + y 2 1 - y .
2 4

2 3
x+ y x+ y
x2 1 - x + y 2 1 - y - .
2 4

Áp d ng b t đ ng th c Holder ta có:
2
x y 3
+ x2 1 - x + y2 1 - y x+ y .
1- x 1- y

3
x y x+ y
+
1- x 1- y x2 1 - x + y 2 1 - y

3
x+ y
2 3
x+ y x+ y
-
2 4

25 - 24 z
=2 .
23 + 24 z

Nên bài toán quy v ch ng minh

z 25 - 24 z 10
+2 .
1- z 23 + 24 z 23

Th t v y, ta có:

z 25 - 24 z 10 z 48 10
+2 - = + 2 -1 -
1- z 23 + 24 z 23 1- z 23 + 24 z 23

192 100
-4-
=
z
+ 23 + 24 z 23
1- z 48 10
2 -1 +
23 + 24 z 23

z 24 z
= -
1- z 48 10
23 23 + 24 z 2 -1 +
23 + 24 z 23

1 24 z
= z -
1- z 48 10
23 23 + 24 z 2 -1 +
23 + 24 z 23

Nguy n Quang Huy K49 THPT Chuyên ĐHSP


L Ư U H À N H N Ộ I B Ộ | 141

24
> z 1-
48 10
23.23. 2 -1 +
23 + 24 23

z 232 23 + 5 47 - 12 1081
=
232 23 + 5 47

>0.

1375 1375
Đ ng th c x y ra khi và ch khi a, b, c = , , 0 và các hoán v .
23 23

Nhận xét 2.26. Ta có 1 bài toán nh sau:

1 1 1 1
Cho th a mãn min a, b, c 1 và + + = . Ch ng minh r ng:
a 2 + 47 b2 + 47 c 2 + 47 24

a + b + c 15 .

Sau khi th c hi n k thu t đ i bi n t ng t , ta thu đ c Ví dụ 2.52.

Ví dụ 2.66. Cho , trong đó không có 2 s nào đ ng th i b ng 0. Ch ng minh r ng:

x y z xyz
+ + + 2 2 2 -3 2.
y+z z+x x+ y x+ y y+z z+x

Dương Đức Lâm

Lời giải.

Đ t ( ).

B t đ ng th c tr thành

a 2 2 2 - 3 abc
+ 2.
b +c2 2
b2 + c 2

Áp d ng b t đ ng th c Holder , b t đ ng th c (2.1) ( Bài tập 13) và b t đ ng th c ta


có:

3
a 2 2 2 - 3 abc a 2 2 2 - 3 abc
+ +
b2 + c2 b2 + c 2 a b +c2 2
8
a2 b 2c 2
9

3
a 2 2 2 - 3 abc
= +
a b +c2 2
4
b2 b 2c 2 + c2 b2c 2
9

3
a 2 2 2 - 3 abc
+
a b +c2 2
4 2 2
b2c + bc 2
9

Nguy n Quang Huy K49 THPT Chuyên ĐHSP


L Ư U H À N H N Ộ I B Ộ | 142

3
a 2 2 2 - 3 abc
+
a b +c
2 2
4 1 2
b2c + bc 2
9 2

3
a 2 2 2 - 3 abc
= + .
a b +c
2 2
2 2
a2 b + c
9

Do b t đ ng th c là đ ng b c nên không m t tính t ng quát, ta gi s a = 1.

q2 4q - 1
Ta s d ng ph ng pháp r max 0, (Do p = a = 1 ).
3 9

B t đ ng th c tr thành

1 3 4-3 2 r
+ 2
q - 3r q - 3r

q - 3r 2q - 9 2 - 2 r

2
q - 3r 2q - 9 2 - 2 r

2
f r = 81 2 - 2 r 2 - 3 23 - 12 2 r + 4q 2 - q 0 .

Ta d ki m tra đ c f r 0 , suy ra f r là hàm l i.

N u . Ta có:

q2
f r max f ,f 0 .
3

q2
f = 3 - 2 2 q 3q - 1 6q 2 + 3 + 2 2 1 - 2q 0.
3

f 0 = q 4q - 1 0.

N u . Ta có:

q2 4q - 1
f r max f ,f .
3 9

q2
f = 3 - 2 2 q 3q - 1 6q 2 + 3 + 2 2 1 - 2q 0.
3

4q - 1 17 - 12 2
f = 4q - 1 3q - 1 0.
9 3

Đ ng th c x y ra khi và ch khi ho c và các hoán v .

Nhận xét 2.27. Ta có Bài toán Tổng quát cho bài toán này.

Nguy n Quang Huy K49 THPT Chuyên ĐHSP


L Ư U H À N H N Ộ I B Ộ | 143

Cho , trong đó không có 2 s nào đ ng th i b ng 0 và . Ch ng minh r ng:

x y y 8xyz
n +n +n + 2n 2 - 3 n 2n 2 .
y+z z+x z+x x+ y y+z z+x

Nhận xét 2.28. bài toán này, có 1 đi u đ c bi t.

N u ta áp d ng b t đ ng th c Holder nh sau:

3
x x
.
y+z x2 y + z

Sau đó quy bài toán v ch ng minh

3
x xyz
+2 2 2 -3 2.
x 2
y+z y+z

Thì b t đ ng th c này l i không đúng !!

Th nh ng, đây, n u s d ng k thu t đ i bi n nh trên thì ta l i có th v t qua rào c n này và s


d ng b t đ ng th c Holder khá hi u qu .

Ví dụ 2.67. Cho th a mãn xyz = 8 . Ch ng minh r ng:

x2 y2 z2 4
+ + .
x +1 y +1
3 3
y +1 z +1
3 3
z +1 x +1
3 3 3

APMO 2005

Lời giải.

a c b
Đ t x = 23 , y = 2 3 , z = 2 3 . B t đ ng th c c n ch ng minh tr thành
b a c
7
a6 1
1
.
3
b 6
8a + b 8c + a

Áp d ng b t đ ng th c Holder ta có:
6
7
4 10
6
a 3
1
8a + b 8c + a ab a5 .
b 6
8a + b 8c + a

Nên ta c n ch ng minh

4 10
3 3
36
a 5
8a + b 8c + a ab = 8 a 2 + 73 ab ab .

Ta có:
3 3 3
8 a 2 + 73 ab ab 9 a 2 + 72 ab ab = 36 a2 + 8 ab ab .

Nguy n Quang Huy K49 THPT Chuyên ĐHSP


L Ư U H À N H N Ộ I B Ộ | 144

Nên bài toán quy v ch ng minh

4 10
3
a 5
a2 + 8 ab ab .

Do có các k t qu Lưu ý 2.14, ta ch c n xét các tr ng h p a = 0 ho c b = c .

. a =0.

Không m t tính t ng quát, ta gi s b c = 1 .

B t đ ng th c tr thành

4 10
3
b +1
5
b b 2 + 8b + 1

4 10

b5 +1
f b = 3
1.
b b 2 + 8b + 1

Th t v y, ta có:

4 9 2 12 8 7 6 4 2
b5 +1 b 5 - 1 b 5 + b 2 - 6b 5 + 32b 5 - 6b 5 + 32b - 6b 5 + b 5 + 1
f b = 4
b 2 b 2 + 8b + 1

0.

128
f b f 1 = >1.
125

. b=c.

Không m t tính t ng quát, ta gi s b = c = 1 .

B t đ ng th c tr thành

4 10
3
a5 + 2 2a + 1 a 2 + 16a + 10

4 10

a5 + 2
g a = 3
1.
2a + 1 a 2 + 16a + 10

Ta có:

4 9 1 1
2 a5 + 2 a 5 -1 k a 5
g a = 1
2 4
a 5 2a + 1 a 2 + 16a + 10

Trong đó

k x = x14 + x13 + x12 + x11 - 13x10 + 52 x9 + 52 x8 + 52 x7 + 52 x 6 - 82 x5 + 28x 4 + 28x3 + 28x 2 + 28x - 40 .

Nguy n Quang Huy K49 THPT Chuyên ĐHSP


L Ư U H À N H N Ộ I B Ộ | 145

Ta d ki m tra đ ck x 0 , suy ra k x là hàm đ ng bi n.

Mà k 0 .k 1 = -40 .189 < 0

x0 0,1 sao cho k x0 = 0 .

g a có đúng 2 ngi m a = 1 và a = x05 0,1 .

Ta có b ng bi n thiên:

0 x05 1

g a min g 0 , g 1
g a + 0 - 0 +
128
= min ,1 = 1 .
125 128
g a g x05
125
. Áp d ng b t đ ng th c AM - GM ta có:

x2 x2
=
x3 + 1 y 3 + 1 x + 1 x2 - x + 1 . y +1 y2 - y +1

x2
x +1 + x - x +1
2
y +1 + y2 - y +1
2 2

4x2
= .
x + 2 y2 + 2
2

Bài toán quy v ch ng minh

x2 1
x +2 y +2
2 2
3

3 x2 z 2 + 2 z2 + 2 .

B t đ ng th c này đúng. Ch ng minh còn l i r t đ n gi n, xin nh ng l i cho b n đ c.

Đ ng th c x y ra khi và ch khi .

Nguy n Quang Huy K49 THPT Chuyên ĐHSP


L Ư U H À N H N Ộ I B Ộ | 146

Ví dụ 2.68. Cho . Ch ng minh r ng:

a 2 - bc b2 - ca c 2 - ab
+ + 0.
a 2 + 2b 2 + 3c 2 b2 + 2c 2 + 3a 2 c 2 + 2a 2 + 3b2

Nguyễn Anh Tuấn

Lời giái.

Áp d ng b t đ ng th c ta có:

8 a 2 - bc 8 a 2 - bc + b + c 6 a 2 + 2b 2 + 3c 2
+b+c =
6 a 2 + 2b 2 + 3c 2 6 a 2 + 2b2 + 3c 2

8 a 2 - bc + b + c a + 2b + 3c
6 a 2 + 2b2 + 3c 2

2
8a 2 + c 2 + ab + bc + ca + 2 b - c
=
6 a 2 + 2b2 + 3c 2

8a 2 + c 2 + ab + bc + ca
.
6 a 2 + 2b 2 + 3c 2

Bài toán quy v ch ng minh

8a 2 + c 2 + ab + bc + ca
2 6 a.
6 a 2 + 2b2 + 3c 2

Áp d ng b t đ ng th c Holder ta có:
2

8a + c + ab + bc + ca
2 2
3
8a 2 + c 2 + ab + bc + ca a 2 + 2b2 + 3c 2 27 3 a2 + bc .
6 a + 2b + 3c
2 2 2

Ta c n ch ng minh
3
27 3 a2 + bc 24 8a 2 + c 2 + ab + bc + ca a 2 + 2b2 + 3c 2 .

Do b t đ ng th c này là đ ng b c nên không m t tính t ng quát, ta gi s a = 1 . S d ng ph ng pháp


4q - 1 1
r max 0, và q .
9 3

8a 2 + c 2 + ab + bc + ca a 2 + 2b2 + 3c 2 = 53q 2 - 38q + 11 - 42r


.
3 a2 + bc = 3 - 5q

3
Ta ch ng minh 27 3 - 5q 24 53q 2 - 38q + 11 - 42r

Nguy n Quang Huy K49 THPT Chuyên ĐHSP


L Ư U H À N H N Ộ I B Ộ | 147

1
N uq . Ta có:
4

1 623
=- 4q - 1 18000q 2 - 21116q + 9297 +
64 64

623
> 0.
64

1
N uq . Ta có:
4

336 4q - 1
155 - 911q + 1601q 2 - 1125q3 + 336r 155 - 911q + 1601q 2 - 1125q3 +
9

1
= 1 - 3q 1125q 2 - 1226q + 353
3

0.

Đ ng th c x y ra khi và ch khi a = b = c .

Ví dụ 2.69. Cho th a mãn a + b + c = 3 . Ch ng minh r ng:

3
a + bc + 3 b + ca + 3 c + ab 3
2 ab + bc + ca .

Trần Quốc Anh

Lời giải.

Ta ch ng minh r ng

a + bc
3 bc .
2

a + bc b + ca c + ab
Đ t x= ,y= ,z = . Khi đó ta có:
2 2 2

a + bc
x=
2

3 bc
= +
2 2
2
3 a
+
2 6

= 3.

1
yz = b + ca c + ab
4

1
= bc + a b2 + c 2 + a 2bc
4

1
= bc + a b2 + c 2 + abc a
4

Nguy n Quang Huy K49 THPT Chuyên ĐHSP


L Ư U H À N H N Ộ I B Ộ | 148

1
= bc + a b 2 + c 2 + 3abc
4

1
= bc + a bc
4

1
= bc + 3 bc
4

= bc .

Nên b t đ ng th c c n ch ng minh tr thành

3
x yz

3
x yz
.
3 3

Áp d ng b t đ ng th c Holder ta có:
1
1 3 8 3
5
3 5 3
3
x 3
x x 3 3
x x x 4
= .
3 3 3 3 3 3

Bài toán quy v ch ng minh


8
3 3
4
x yz
.
3 3

3
Do b t đ ng th c này là đ ng b c nên không m t tính t ng quát, ta gi s x4 = 3 .

B t đ ng th c tr thành

yz 3 .

3 3 3
Đ t m = x 4 , n = y 4 , p = z 4 . Ta có: m = 3 . B t đ ng th c tr thành

4
np 3 3.

Áp d ng b t đ ng th c AM - GM và b t đ ng th c Schur ta có:
4 1
np 3 = np. np 3

n + p +1
np.
3

np 4 - m
=
3

4 np - 3mnp
=
3

Nguy n Quang Huy K49 THPT Chuyên ĐHSP


L Ư U H À N H N Ộ I B Ộ | 149

4 m np - 9mnp
=
9
3
m
9

= 3.

Đ ng th c x y ra khi và ch khi a = b = c = 1 .

Ví dụ 2.70. Cho th a mãn a + b + c = 3 . Ch ng minh r ng:

a b c
+ + 3.
1 + b + bc 1 + c + ca 1 + a + ab

Phan Thành Việt

Lời giải.

. Áp d ng b t đ ng th c Holder ta có:
2 3
2
a
a 1 + b + bc a3 .
1 + b + bc

Nên ta c n ch ng minh

2 3

a 3
3 a 1 + b + bc .

Ta có:

2 3 2 2 2 2 2 2 2
a3 = a2 + 3 b 3 c 3 b 3 + c 3 + 6a 3 b 3 c 3

2 2 2
a2 + 6 bc + 6a 3 b 3 c 3

2 2 2
= 9+ 4 bc + 6a 3 b 3 c 3 .

Bài toán quy v ch ng minh


2 2 2
9+4 bc + 6a 3 b 3 c 3 3 a 1 + b + bc

2 2 2
bc + 6a 3 b 3 c 3 9abc .

Th t v y, áp d ng b t đ ng th c AM - GM ta có:
2 2 2 2 2 2 2 2 2
bc + 6a 3 b 3 c 3 9a 3 b 3 c 3 = 3a 3 b 3 c 3 a 9abc .

. Áp d ng b t đ ng th c Holder ta có:
2
a 3 3 6
. a 2 1 + b + bc 2a + b + 3c a 2a + b + 3c =8 a .
1 + b + bc

Nguy n Quang Huy K49 THPT Chuyên ĐHSP


L Ư U H À N H N Ộ I B Ộ | 150

6 3
Bài toán quy v ch ng minh 8 a 3 a 2 1 + b + bc 2a + b + 3c

7 3 2
8 a a 2 2a + b + 3c a + 3b a + 9bc .

Ta có:
7 3 2
8 a - a 2 2a + b + 3c a + 3b a + 9bc

Do đó ta c n ch ng minh

4 ab a 4 + b4 + 26 a 2b 4 + 39 a 4b 2 + 54 a3b3 + 261a 2b2c 2 24abc a3 + 93abc a 2b + 97abc ab 2 .

B t đ ng th c này đúng theo b t đ ng th c AM - GM và b t đ ng th c Schur .

Đ ng th c x y ra khi và ch khi a = b = c = 1 ho c a, b, c = 3,0,0 và các hoán v .

BÀI TẬP

89. Cho th a mãn a + b + c = 3 . Ch ng minh r ng:


a b c
+ + 3.
1 + 2bc 1 + 2ca 1 + 2ab
Phan Thành Việt

Lời giải.
Áp d ng b t đ ng th c Holder ta có:
2 3
2
a
a 1 + 2bc a3 .
1 + 2bc
Nên ta c n ch ng minh
2 3

a3 3 a 1 + 2bc .

Ta có:
2 3 2 2 2 2 2 2 2
a 3
= a2 + 3 b 3 c 3 b 3 + c 3 + 6a 3 b 3 c 3

2 2 2
a2 + 6 bc + 6a 3 b 3 c 3
2 2 2
= 9+ 4 bc + 6a 3 b 3 c 3 .
Bài toán quy v ch ng minh
2 2 2
9+4 bc + 6a 3 b 3 c 3 3 a 1 + 2bc
2 2 2
2 bc + 3a b c 3 3 3
9abc .
Th t v y, áp d ng b t đ ng th c AM - GM ta có:
2 2 2 2 2 2 2 2 2
2 bc + 3a 3 b 3 c 3 9a 3 b 3 c 3 = 3a 3 b 3 c 3 a 9abc .
Đ ng th c x y ra khi và ch khi a = b = c = 1 ho c a, b, c = 3,0,0 và các hoán v .

Nguy n Quang Huy K49 THPT Chuyên ĐHSP


L Ư U H À N H N Ộ I B Ộ | 151

90. Cho th a mãn abc = 1 . Ch ng minh r ng:


a b c
+ + 1
a + 6b + 2bc b + 6c + 2ca c + 6a + 2ab
Nguyễn Văn Quý-Vasile Cirtoaje

Lời giải.
Áp d ng b t đ ng th c Holder ta có:
2 3
2
a
a a + 6b + 2bc a3 .
a + 6b + 2bc
Bài toán quy v ch ng minh
2 3

a 3
a a + 6b + 2bc

2 2 2 2
b3c 3 b3 + c 3 2 bc

2 2 1 1 2

b3c3 b3 - c3 0 Đúng .

Đ ng th c x y ra khi và ch khi a = b = c = 1 .

Ví dụ 2.71. Cho th a mãn a + b + c = 1 . Ch ng minh r ng:

1 1 1
+ + 5.
2 2 2
b-c c-a a-b
a+ b+ c+
4 4 4

Walther Janous

Lời giải.

Áp d ng b t đ ng th c Holder ta có:
2

2
1 3 b-c 3
a + 2b + 2c a+ a + 2b + 2c = 125 a = 125 .
b-c
2 4
a+
4

Ta c n ch ng minh
2
3 b-c
5 a + 2b + 2c a+ .
4

Ta s d ng ph ng pháp q2 3r (Do p = a = 1 ).

B t đ ng th c tr thành

3q 2 q
5q + 3r 11 - .
4 4

Th t v y, ta có:

3q 2 q 3q 2 q
5q - - 3r 11 - 5q - - q 2 11 -
4 4 4 4

Nguy n Quang Huy K49 THPT Chuyên ĐHSP


L Ư U H À N H N Ộ I B Ộ | 152

q q 2 - 47q + 20
=
4

0.

Đ ng th c x y ra khi và ch khi a, b, c = 1,0,0 và các hoán v .

Ví dụ 2.72. Cho . Ch ng minh r ng:

2 a + b + c ab + bc + ca 3 2a + b 2b + c 2c + a .
a 2 + b2 + c 2 + ab + bc + ca

Sung Yoon Kim

Lời giải.

Ta ch ng minh
3 3 3
8 a bc 2a + b a2 + bc .

. Áp d ng b t đ ng th c Holder ta có:
3 3 3
8 a bc = 8 b + c + abc

3
8 a + b + a3

=8 2a + b a 2 + ab + b2 .

Do đó bài toán quy v ch ng minh


3
8 a 2 + ab + b 2 a2 + bc .

3
a6 + 3 a5 b + c + abc a3 - 2 a4 b2 + c2 - bc - abc a 2 b + c - 3a 2b 2c 2 0

3
a 6 + 3a 2b2c 2 - a4 b2 + c2 + a5 b + c - a 4 b2 + c 2 +2 a5 b + c - 2 bc

3
+3 bc - 3a 2b2c 2 + abc a3 + 3abc - a2 b + c 0.

B t đ ng th c này đúng theo b t đ ng th c Schur và b t đ ng th c AM - GM .

. Áp d ng b t đ ng th c ta có:

2a + b = 2 a 2b + 2 a ab + 3abc

2
2 ab
+2 a ab + 3abc .
a

Ta c n ch ng minh
2 3 3
2 ab 8 a bc
+2 a ab + 3abc 3 .
a a2 + bc

Nguy n Quang Huy K49 THPT Chuyên ĐHSP


L Ư U H À N H N Ộ I B Ộ | 153

Do b t đ ng th c này là đ ng b c nên không m t tính t ng quát, ta gi s a = 1 . S d ng ph ng pháp


4q - 1
r max 0, (Do p = a = 1 ).
9

B t đ ng th c tr thành

8q3
2q 2 + 2q + 3r 3
.
1- q

N u . Ta có:

8q 3 8q 3
2q 2 + 2q + 3r - 3
2q 2 + 2q - 3
1- q 1- q

2 q 1 - 2q - 4q 2 + 2q 3 - q 4
= 3
1- q

2q 1 - 2q - 4q 2
3
1- q

2q 1 - 3q
3
1- q

0.

N u 1 4q . Ta có:

8q 3 4q - 1 8q3
2q 2 + 2q + 3r - 3
2 q 2 + 2q + - 3
1- q 3 1- q

1 - 3q 2q 4 - 2q 3 + 3q 2 + 10q - 1
= 3
1- q

0.

Đ ng th c x y ra khi và ch khi a = b = c .

Ví dụ 2.73. Cho , trong đó không có 2 s nào đ ng th i b ng 0. Ch ng minh r ng:

a 2 + bc b 2 + ca c 2 + ab
+ 2 + 2 6.
b + bc + c
2 2
c + ca + a 2
a + ab + b2

Võ Quốc Bá Cẩn

Lời giải.

Áp d ng b t đ ng th c Holder ta có:
2
a 2 + bc 2 3
a 2 + bc b2 + bc + c 2 a 2 + bc .
b + bc + c 2
2

Nguy n Quang Huy K49 THPT Chuyên ĐHSP


L Ư U H À N H N Ộ I B Ộ | 154

Bài toán quy v ch ng minh


3 2
a 2 + bc 6 a 2 + bc b 2 + bc + c 2 .

Do b t đ ng th c này là đ ng b c nên không m t tính t ng quát, ta gi s a = 1.

1 - q2
Đ t bc = q 0,1 .
3
2
1+ q 1 - 2q
Và abc = r r max 0, (Do a = 1 ). Xem bi n đ i b t đ ng th c (2.18).
27

Đ ý r ng:

B t đ ng th c tr thành

17 6 8 4 20 2 10
-2 4q 2 + 5 r + q - q - q + 0.
27 9 9 27

N u . Ta có:

17 6 8 4 20 2 10 17 6 8 4 20 2 10
-2 4q 2 + 5 r + q - q - q + q - q - q + 0.
27 9 9 27 27 9 9 27

N u 1 2q . Ta có:

2
17 6 8 4 20 2 10 -2 4 q 2 + 5 1 + q 1 - 2q 17 6 8 4 20 2 10
-2 4q 2 + 5 r + q - q - q + + q - q - q +
27 9 9 27 27 27 9 9 27

1 2
= q 17q 4 + 16q3 + 20q - 38
27

0.

Đ ng th c x y ra khi và ch khi a = b = c .

Ví dụ 2.74. Cho , trong đó không có 2 s nào đ ng th i b ng 0. Ch ng minh r ng:

a b+c b c+a c a+b


+ + 2.
2b + c b + 2c 2c + a c + 2a 2a + b a + 2b

Vasile Cirtoaje

Lời giải.

. Áp d ng b t đ ng th c Holder ta có:
2
a b+c a 2 2b + c b + 2c 3
a .
2b + c b + 2c b+c

Ta c n ch ng minh

3 a 2 2b + c b + 2c
a 2
b+c

Nguy n Quang Huy K49 THPT Chuyên ĐHSP


L Ư U H À N H N Ộ I B Ộ | 155

2
2a 2 b + c + a 2bc
a3 + 3 bc b + c + 6abc 2
b+c

2a
a3 + 3 bc b + c + 6abc 4 bc b + c + abc
b+c

2a
a3 - bc b + c + 3abc abc -3
b+c

abc 2a + b + c a - b a - c
a a -b a -c
b+c

abc 2a + b + c
a- a-b a-c 0
b+c

a 2 b2 + c 2 + ab + ac
a -b a -c 0.
b+c

Không m t tính t ng quát, ta gi s a b c .

Ta có:

a 2 b2 + c 2 + ab + ac a 2 b2 + c 2 + ab + ac b 2 c 2 + a 2 + bc + ba
a -b a -c a -b a -c + b-c b-a
b+c b+c b+c
a - b a 2 b2 + c 2 + ab + ac a - c - b2 c 2 + a 2 + bc + ba b - c
=
b+c

a - b a 2 b2 + c 2 + ab + ac b - c - b 2 c 2 + a 2 + bc + ba b - c
b+c

a - b b - c a 2 b2 + c 2 + ab + ac - b 2 c 2 + a 2 + bc + ba
=
b+c

2
a -b b-c c 2 + ab a + b + c a 2 + ab + b 2
=
b+c

0.

. Đ t a = x2 , b = y2 , c = z 2 ( ). B t đ ng th c tr thành

2 y2 + z2
x 2.
2 y2 + z 2 y 2 + 2z 2

Ta ch ng minh

2 y2 + z2 y+z
2y + z
2 2
y + 2z
2 2
y 2 + yz + z 2

2 2
2 y2 + z2 y 2 + yz + z 2 y+z 2 y2 + z2 y 2 + 2z 2

2
y2 z2 y - z 0 Đúng .

Nguy n Quang Huy K49 THPT Chuyên ĐHSP


L Ư U H À N H N Ộ I B Ộ | 156

Áp d ng b t đ ng th c (2.4) ( Ví dụ 2.32) ta có:

2 y2 + z 2 x y+z
x 2.
2y + z
2 2
y + 2z
2 2
y + yz + z 2
2

Đ ng th c x y ra khi và ch khi a = b = c ho c và các hoán v .

Ta có 1 bài toán nh sau:

Cho , trong đó không có 2 s nào đ ng th i b ng 0. Ch ng minh r ng:

a b+c b c+a c a+b


+ + 2.
b +c
2 2
c +a
2 2
a2 + b2

Võ Quốc Bá Cẩn

Đ ng th c x y ra khi và ch khi và các hoán v .

Ta th y r ng b t đ ng th c này là 1 h qu c a bài toán trên do b2 + c 2 b 2 + 5bc + c 2 = 2b + c b + 2c ,


c2 + a2 2c + a c + 2a và a 2 + b2 2a + b a + 2b .

Ví dụ 2.75. Cho . Ch ng minh r ng:

a b c 1 1 1
+ + + + .
b2 c 2 a 2 a 2 - ab + b2 b2 - bc + c 2 c 2 - ca + a 2

Lời giải.

. Áp d ng b t đ ng th c Holder ta có:
3
a a
2
.
b2 ab

Áp d ng b t đ ng th c ta có:

1 1
3 .
a 2 - ab + b2 a 2 - ab + b2

Do đó ta ch ng minh
3
a 1
3
ab
2
a 2 - ab + b2

6
a 1
3 .
ab
4
a - ab + b2
2

Do b t đ ng th c này là đ ng b c nên không m t tính t ng quát, ta gi s a = 1 . S d ng ph ng pháp

4q - 1 3q - 10q + 4
2
q2 1 - q
max 0, r (Theo các k t qu (2.22), (2.23)).
1 - 2q 2 2 - 3q

Đ ý r ng:

Nguy n Quang Huy K49 THPT Chuyên ĐHSP


L Ư U H À N H N Ộ I B Ộ | 157

a 2 - ab + b2 = -3q3 + q 2 + 10rq - 8r 2 - 3r .

b2 - bc + c2 c 2 - ca + a 2 = 7q 2 - 5q - 2r + 1 .

B t đ ng th c tr thành

f r = 8r 2 - 6q 4 + 10q - 3 r + 21q 6 - 15q5 + 3q 4 + 3q3 - q 2 0.

Ta có:
2
q2 1 - q q 2 1 - 2q 3q - 1 2 + q - 22q 2 + 30q3
f =
2 2 - 3q 2 2 - 3q
2

0.

1 4q - 1 3q 2 - 10q + 4
Vì q nên > 0.
3 1 - 2q

. Áp d ng b t đ ng th c ta có:

1 1 ab
a - ab + b
2 2 ab a 2 - ab + b 2

1
3
ab

1
.
c

Bài toán quy v ch ng minh

1 a
c b2

Áp d ng b t đ ng th c AM - GM ta có:

a a 1 1
= + -
b2 b2 a b

1 1 1
2 - = .
b b b

Đ ng th c x y ra khi và ch khi a = b = c .

Ta có 1 bài toán nh sau:

Cho . Ch ng minh r ng:

a 2 b2 c2
+ + a 2 - ab + b 2 + b2 - bc + c 2 + c 2 - ca + a 2 .
b c a

Nguyễn Mạnh Tuấn-THTT 389

Nguy n Quang Huy K49 THPT Chuyên ĐHSP


L Ư U H À N H N Ộ I B Ộ | 158

Ví dụ 2.76. Cho , trong đó không có 2 s nào đ ng th i b ng 0. Ch ng minh r ng:

a3 b3 c3
2 + + 2 a 2 + b 2 + c 2 - 3abc a + b + c .
b+c c+a a+b

Nguyễn Công Minh

Lời giải.

Áp d ng b t đ ng th c Holder ta có:

a3 3
a a2 b + c a2 .
b+c

3
a3 a2
.
b+c a a2 b + c

Ta c n ch ng minh
3
a2
+ 3abc a 2 a2 .
a a2 b + c

Do b t đ ng th c này là đ ng b c nên không m t tính t ng quát, ta gi s a = 1 . S d ng ph ng pháp


4q - 1 1 - q
r max 0, (Do p = a = 1 ).
6

B t đ ng th c tr thành
3
2 1 - 2q
+ 3r 2 1 - 2q .
q - 3r

N u . Ta có:

3 3
2 1 - 2q 2 1 - 2q
+ 3r - 2 1 - 2q - 2 1 - 2q
q - 3r q

2 1 - 2q 1 - 4q 1 - q
=
q

0.

N u . Ta có:

3 3
2 1 - 2q 2 1 - 2q 4q - 1 1 - q
+ 3r - 2 1 - 2q + - 2 1 - 2q
q - 3r 4q - 1 1 - q 2
q-
2

2 2q - 1 4 q - 1 1 - q 4q - 1 1 - q
= +
4q 2 - 3q + 1 2

4q - 1 1 - q
= 4q 2 - 3q + 1 + 2 2q - 1 2 4q - 1 1 - q
2 4q 2 - 3q + 1

Nguy n Quang Huy K49 THPT Chuyên ĐHSP


L Ư U H À N H N Ộ I B Ộ | 159

4q - 1 1 - q
4q 2 - 3q + 1 + 2q - 1 2 4q - 1 + 1 - q
2 4q 2 - 3q + 1

2
2 4q - 1 1 - q 1 - 3q
=
4q 2 - 3q + 1

0.

Đ ng th c x y ra khi và ch khi a = b = c ho c và các hoán v .

Ví dụ 2.77. Cho , trong đó không có 2 s nào đ ng th i b ng 0. Ch ng minh r ng:

a 2 + ab + b 2 b 2 + bc + c 2 c 2 + ca + a 2 3 6
+ + .
c + ab
2
a + bc
2
b2 + ca 2

Nguyễn Văn Thạch

Lời giải.

Áp d ng b t đ ng th c Holder ta có:
2 3
a 2 + ab + b 2 a +b c 2 + ab 3 3
a +b =8 c .
c 2 + ab a 2 + ab + b2

Ta c n ch ng minh
3
16 3 a +b c 2 + ab
c
27 a + ab + b 2
2

16 3 ab a + b c 2 + ab
c 2 ab a + b + .
27 a 2 + ab + b 2

Ta có:

ab a + b c 2 + ab ab a + b c 2 + ab
2 ab a + b + 2 ab a + b +
a + ab + b
2 2
3
a+b
2

4 a 2b 2 4 c
=2 ab a + b + + abc
3 a +b 3 a+b

1 4 c
2 ab a + b + ab a + b + abc
3 3 a +b

7 4 c
= ab a + b + abc .
3 3 a +b

Bài toán quy v ch ng minh

16 3 c
c 7 ab a + b + 4abc .
9 a+b

Do b t đ ng th c này là đ ng b c nên không m t tính t ng quát, ta gi s a = 1 . S d ng ph ng pháp


4q - 1 1 - q q2
max 0, r (Do p = a = 1 ).
6 3

Nguy n Quang Huy K49 THPT Chuyên ĐHSP


L Ư U H À N H N Ộ I B Ộ | 160

B t đ ng th c tr thành

16 1+ q
7 q - 3r + 4r -3
9 q-r

f r = 297r 2 + 52 - 324q r - 16q + 63q 2 0.

4q - 1 1 - q q2
f r max max 0, f ,f .
6 3

Ta có:

q2 q 3q - 1 33q 2 - 97q + 48
f = 0.
3 3

1
N u0 q . Ta có:
4

f r = 297r 2 + 52 - 324q r - 16q + 63q 2 f 0 = -16q + 63q 2 = q 63q - 16 0.

1 1
N u q . Ta có:
4 3

4q - 1 1 - q 3q - 1 528q3 - 280q 2 + 29q + 5


f =
6 12

1 5 44
q3 3q - 1 -3 + - 148 + 84
q q2 q
=
12

1 5 44
q 3 3q - 1 -3 + - 148 + 84
q 1 1
9 3
12

1
q 3 3q - 1 29 - 3 + 84
q
=
12

0.

Đ ng th c x y ra khi và ch khi a = b = c .

Ví dụ 2.78. Cho , trong đó không có 2 s nào đ ng th i b ng 0 th a mãn a + b + c = 3 . Ch ng


minh r ng:

a3 b3 c3 3
+ 2 + 2 .
a + 3b
2 2
b + 3c 2
c + 3a 2 2

Phan Thành Việt

Lời giải.

. Áp d ng b t đ ng th c Holder ta có:

Nguy n Quang Huy K49 THPT Chuyên ĐHSP


L Ư U H À N H N Ộ I B Ộ | 161

2
a3 3 3 3
a 2 + 3b2 a + c a a+c = a2 + ab .
a 2 + 3b 2

Bài toán quy v ch ng minh

3 9 3
a2 + ab a 2 + 3b 2 a + c
4
3 3
4 a2 + ab 3 a a 2 + 3b 2 a + c

a6 + 9 a5b + 12 a 4b 2 + 27 a 4bc 3 a 2b 4 + 2 a 3b3 + 12 a 2b3c + 6 a 3b 2c + 78a 2b 2c 2 .

Ta có 1 b đ : N u cho . Khi đó ta có:

a3 + b3 + c3 - 3abc 4 a - b b - c c - a .

(2.30)

Đ ng th c x y ra khi và ch khi a = b = c .

Chứng minh

B t đ ng th c t ng đ ng v i

1 2
c a -b 4 a -b .
2

Không m t tính t ng quát, ta gi s c = min a, b, c . Đ t ( ).

B t đ ng th c tr thành

3c + x + y x 2 - xy + y 2 4 x - y xy .

N u xy = 0 , b t đ ng th c hi n nhiên đúng.

Xét tr ng h p xy > 0 . Đ t x = ty ( t > 0 ).

Ta có:

3c + x + y x 2 - xy + y 2 - 4 x - y xy x + y x 2 - xy + y 2 - 4 x - y xy

= y 3 t + 1 t 2 - t + 1 - 4t t - 1

= y 3 t 3 - 4t 2 + 4t + 1

0.

Đ ng th c x y ra khi và ch khi a = b = c .

Tr l i bài toán chính.

Áp d ng b đ , ta c n ch ng minh

9 a5b + 7 a 4b2 + 27 a 4bc 12 a 2b3c + 6 a 3b2c + 75a 2b 2c2 .

Áp d ng b t đ ng th c AM - GM ta có:

Nguy n Quang Huy K49 THPT Chuyên ĐHSP


L Ư U H À N H N Ộ I B Ộ | 162

9 a5b + 7 a 4b 2 + 27 a 4bc = 12 a 4bc + 6 a 4bc + 9 a 5b + 7 a 4b 2 + 9 a 4bc

12 a 2b3c + 6 a3b2c + 9 a 5b + 7 a 4b 2 + 9 a 4bc

12 a 2b3c + 6 a3b 2c + 75a 2b 2c 2 .

. Áp d ng b t đ ng th c AM - GM ta có:

a3 a2
=6
a 2 + 3b 2 4a a 3 a 2 + 3b2

a2
12
4a a + 3 a 2 + 3b2

a2
= 12 .
7 a + 9b + 4ab + 4ac
2 2

Áp d ng b t đ ng th c ta có:

a2 2 2
c + 2a 7a 2 + 9b2 + 4ab + 4ac a c + 2a
7a + 9b + 4ab + 4ac
2 2

2
= 2 a2 + ab .

2
a2 2 a2 + ab
.
7a + 9b + 4ab + 4ac
2 2
c + 2a
2
7 a 2 + 9b 2 + 4ab + 4ac

Bài toán quy v ch ng minh


2 2
8 2 a2 + ab c + 2a 7a 2 + 9b 2 + 4ab + 4ac

a4 + a 2b 2 + 3 a3b - 3 ab3 - 2abc a 0.

Không m t tính t ng quát, ta gi s a = min a, b, c . Đ t ( ).

B t đ ng th c tr thành

6 x 2 - xy + y 2 a 2 + 4 x3 + 9 x 2 y - 9 xy 2 + 4 y 3 a + x 4 + 3x 3 y + x 2 y 2 - 3xy 3 + y 4 0.

B t đ ng th c này đúng do

9 2 7
4 x3 + 9 x 2 y - 9 xy 2 + 4 y 3 = 4 x3 + y 2x - y + y3 0.
4 4
2
3 3
x 4 + 3 x3 y + x 2 y 2 - 3xy 3 + y 4 = x 2 + xy - y 2 + x2 y2 0.
2 4

Đ ng th c x y ra khi và ch khi a = b = c = 1 .

Nhận xét 2.29. Ta có 1 bài toán nh sau:

Cho . Ch ng minh r ng:

Nguy n Quang Huy K49 THPT Chuyên ĐHSP


L Ư U H À N H N Ộ I B Ộ | 163

a3 + b3 + c3 - 3abc 9+6 3 a-b b-c c-a .

b c
Đ ng th c x y ra khi và ch khi a = b = c ho c a = 0, + = 1 + 3, b < c và các hoán v .
c b

Đây là 1 k t qu ch t h n r t nhi u so v i b đ trong bài toán trên.

BÀI TẬP

91. Cho . Ch ng minh r ng:


1 1 1 9
+ + .
2a 2 + ab + bc 2b2 + bc + ca 2c 2 + ca + ab 2 a +b+c

Lời giải.
Áp d ng b t đ ng th c Holder ta có:
2
1 3 3 3
2a 2 + ab + bc a + 3b + 4c a + 3b + 4c = 512 a .
2a + ab + bc
2

Nên ta c n ch ng minh
5 3
2048 a 81 2a 2 + ab + bc a + 3b + 4c .
1886 a5 + 6514 a 4b + 925 ab 4 + 8236 a3bc 5521 a 2b3 + 3739 a 3b 2 + 8301 a 2b 2c .
Không m t tính t ng quát, ta gi s a = min a, b, c . Đ t ( ).
B t đ ng th c tr thành
.
Trong đó
A = 34560 x 2 - xy + y 2 0.
.
.
.
B t đ ng th c đ c ch ng minh.
Đ ng th c x y ra khi và ch khi a = b = c .

92. Cho . Ch ng minh r ng:


a b c 3
+ + .
b + 15ca
2
c + 15ab
2
a + 15bc
2 4
Park Doo Sung

Lời giải.
Áp d ng b t đ ng th c Holder ta có:
2
a 3 3 6
a b2 + 15ca a + 2b a a + 2b = a .
b + 15ca
2

Bài toán quy v ch ng minh


6 3
16 a 9 a b 2 + 15ca a + 2b .

.
Không m t tính t ng quát, ta gi s a = min a, b, c . Đ t ( ).

Nguy n Quang Huy K49 THPT Chuyên ĐHSP


L Ư U H À N H N Ộ I B Ộ | 164

B t đ ng th c tr thành
.
Trong đó
A = 567( x 2 - xy + y 2 ) 0 .
.
C = 18 11x + 47 x y + 204 x y - 82 xy 3 + 11y 4
4 3 2 2
0.
D = 9 9 x5 + 68 x 4 y + 118 x3 y 2 + 134 x 2 y 3 - 69 xy 4 + 9 y 5 0.
.
B t đ ng th c đ c ch ng minh.
Đ ng th c x y ra khi và ch khi a = b = c .

93. Cho , trong đó không có 2 s nào đ ng th i b ng 0. Ch ng minh r ng:


a b c 3
+ + .
b + 3c
2 2
c + 3a
2 2
a + 3b
2 2 2
Vasile Cirtoaje

Lời giải.
. Áp d ng b t đ ng th c Holder ta có:
2
a 3 3 3
a b 2 + 3c 2 2a + b a 2a + b = 2 a2 + ab .
b + 3c 2 2

Bài toán quy v ch ng minh


3 3
4 2 a2 + ab 9 a b2 + 3c 2 2a + b .

.
Không m t tính t ng quát, ta gi s a = min a, b, c . Đ t ( ).
B t đ ng th c tr thành
.
Trong đó
.
.
C = 9 101x + 146 x y - 111x y - 73xy + 101y 4
4 3 2 2 3
0.
D = 9 31x5 + 48 x 4 y + 33x 3 y 2 - 88 x 2 y 3 - xy 4 + 31y 5 0.
.
B t đ ng th c đ c ch ng minh.
2
x a2 y2 b2 z2 c2
.Đ t = 2 , = 2 , = 2 ( ).
4 b + 3c 4 c + 3a 4 a + 3b 2
2 2

Đ ý r ng
x2 y2 a 2b 2
=
16 b + 3c 2 c 2 + 3a 2
2

a 2b 2 a 2 + 3b 2
=
a 2 + 3b 2
28a 2b 2c 2
= 1-
a 2 + 3b 2
x2 y2 z 2
= 1 - 28. . .
4 4 4

Nguy n Quang Huy K49 THPT Chuyên ĐHSP


L Ư U H À N H N Ộ I B Ộ | 165

7 x2 y2 z 2
= 1- .
16
3 x2 y 2 + 7 x2 y 2 z 2 = 16 .
B t đ ng th c c n ch ng minh tr thành
x 3.
Ta s d ng ph ng pháp ph n ch ng. Ng c l i, ta gi s x < 3 . Khi đó k > 1 sao cho
3=k x= kx .
Đ t và u + v + w = 3 .
7u 2 v 2 w2 3 u v + v w + w u
2 2 2 2 2 2

16 = + < 7u 2 v 2 w2 + 3 u 2 v 2 + v 2 w2 + w2u 2 .
k6 k4
Ta s ch ng minh
16 7u 2v 2 w2 + 3 u 2v 2 + v 2 w2 + w2u 2 .
4q - 9 q2
Ta s d ng ph ng pháp max 0, r .
3 9
B t đ ng th c tr thành
f r = 7r 2 - 18r + 3q 2 - 16 0 .
4q - 9 q 2
f r max 0, , .
3 9
Ta có:
13
f 0 = 3q 2 - 16 = -< 0.
16
4q - 9 q - 3 139 q - 303
f = 0.
3 9
q2 q - 3 q + 3 7q 2 + 144
f = 0.
9 81
Đi u này d n đ n mâu thu n. Do đó b t đ ng th c c n ph i ch ng minh đúng.

Đ ng th c x y ra khi và ch khi a = b = c .

Nhận xét 2.30. Ta có Bài toán Tổng quát cho bài toán này.
Cho , trong đó không có 2 s nào đ ng th i b ng 0 và k > 0 . Ch ng minh r ng:
a b c 3 2
+ + min , .
kb 2 + c 2 kc 2 + a 2 ka 2 + b2 k +1 4 k
94. Cho , trong đó không có 2 s nào đ ng th i b ng 0. Ch ng minh r ng:
a b c
+ + 4
8.
2b + c
2 2
2c + a
2 2
2a + b 2
2

Phạm Kim Hùng


Đ ng th c x y ra khi nào ?
95. Cho , trong đó không có 2 s nào đ ng th i b ng 0. Ch ng minh r ng:
a b c 3
+ + .
3b + ca
2
3c + ab
2
3a + bc
2 2
Vasile Cirtoaje

Ví dụ 2.79. Cho . Ch ng minh r ng:

a 2 b2 c2 3 a 3 + b3 + c 3
+ + 4
. a 2 + b2 + c2 .
b c a abc

Nguy n Quang Huy K49 THPT Chuyên ĐHSP


L Ư U H À N H N Ộ I B Ộ | 166

Lời giải.

Ta s ch ng minh

8 2 4
a2 9 a3 a2
.
b a 2b 2 c 2

Áp d ng b t đ ng th c và b t đ ng th c AM - GM ta có:

8 6 2
a2 a2 a2
=
b b b

3 2
a4 a 2b a4
= +2
b2 c a 2b

2 2
2
a4 a 2b a2
27 . .
b2 c a 2b

Ta c n ch ng minh

2 2 2 4
2
a4 a 2b a2 9 a3 a2
27 .
b2 c a 2b a 2b 2 c 2

a4 2 2 2
3 a3b2 a 2b a3 .
b2

Không m t tính t ng quát, ta gi s a b c .

Áp d ng b t đ ng th c Holder , b t đ ng th c và b t đ ng th c ta có:

a4
a 3b 2 a2 a3b2 a 3 a3
a 4
3 2 2
b2
3 ab =
b2 a3 a2 a

3 2
a3 . a 2b . a2 a
3 2
a a a

2 2
= a 2b a3 .

Đ ng th c x y ra khi và ch khi a = b = c .

Ví dụ 2.80. Cho a, b, c là đ dài 3 c nh c a 1 tam giác th a mãn . Ch ng minh r ng:

b+c c+a a+b


+ + 6.
b+c-a c + a -b a +b-c

Lời giải.

Ta có 1 b đ : N u cho , trong đó không có 2 s nào đ ng th i b ng 0. Khi đó ta có:

Nguy n Quang Huy K49 THPT Chuyên ĐHSP


L Ư U H À N H N Ộ I B Ộ | 167

4a 3 b + c - a 4b3 c + a - b 4c 3 a + b - c
ab + bc + ca 2
+ 2
+ 2
.
b+c c+a a +b

(2.31)

Đ ng th c x y ra khi và ch khi a = b = c .

Chứng minh.

a2 b2
2 2
Không m t tính t ng quát, ta gi s a b c b+c c+a .
2 2
2a - b - c 2b - c - a

Ta ch ng minh

4a3 b + c - a
a2 - 2
a2 - bc
b+c

2
a 2 2a - b - c
2
c2 - ab .
b+c

Áp d ng b t đ ng th c và b t đ ng th c ta có:

2 2
a 2 2a - b - c 1 a2 b2 2 2 c 2 2c - a - b
2 2
+ 2
2a - b - c + 2b - c - a + 2
b+c 2 b+c c+a a+b

2
c 2 2c - a - b
2
1 a b 2 2
+ 2a - b - c + 2b - c - a +
4 b+c c+a a+b
2

2 2
1 4 a +b 2 2 c 2 2c - a - b
. . 2a - b - c + 2b - c - a +
4 a + b + 2c 2 a+b
2

2 2 2
a +b 2c - a - b c 2 2c - a - b
2
+ 2
.
2 a + b + 2c a+b

Ta l i có:

c2 - ab = c 2 - c a + b + a 2 - ab + b 2

2
a+b
c2 - c a + b +
4
2
2c - a - b
= .
4

B t đ ng th c c n ch ng minh t ng đ ng v i
2
2 a+b c2 1
2c - a - b 2
+ 2
- 0.
2 a + b + 2c a+b 4

Nên ta c n ch ng minh

Nguy n Quang Huy K49 THPT Chuyên ĐHSP


L Ư U H À N H N Ộ I B Ộ | 168

2
a +b c2 1
2
+ 2
.
2 a + b + 2c a+b 4

Do b t đ ng th c này là đ ng b c nên không m t tính t ng quát, ta gi s a = 1.

Ta có:
2 2
a+b c2 1 1- c c2 1
2
+ 2
- = 2
+ 2
-
2 a + b + 2c a+b 4 2 1+ c 1- c 4

5c 4 - 8c3 + 18c 2 - 8c + 1
= 2 2
2 1+ c 1- c

>0.

Đ ng th c x y ra khi và ch khi a = b = c .

Tr l i bài toán chính.

Áp d ng b t đ ng th c Holder ta có:
2
b+c 4a3 b + c - a 3
2
a .
b+c-a b+c

3
b+c a
b+c-a 4a 3 b + c - a
2
b+c

3
4 a
bc

3
4 a
=
1
bc bc a2
3

3
4 a
3
1 bc + bc + a2
3 3

3
4 a
= 3
=6.
a
9

Đ ng th c x y ra khi và ch khi a = b = c = 1 .

Nguy n Quang Huy K49 THPT Chuyên ĐHSP


L Ư U H À N H N Ộ I B Ộ | 169

Ví dụ 2.81. Cho , trong đó không có 2 s nào đ ng th i b ng 0 th a mãn a + b + c = 3 . Ch ng


minh r ng:

3 a 2 b2 + c2 + 3 b 2 c 2 + a 2 + 3 c 2 a 2 + b 2 33 2 .

Michael Rozenberg

Lời giải.

Áp d ng b t đ ng th c Holder ta có:

3
2 b2 + c2
3 a2 b2 + c2 a b+c 2
b+c

2 bc
=4 bc 3-2 2
.
b+c

Ta c n ch ng minh

27 bc
2
+2 2
3
bc b+c

4
a bc
2
+2 2
3.
6 bc b+c

Áp d ng b t đ ng th c AM - GM ta có:

4 4
a bc a 3 bc 3
2
+2 2
= 2
+ +2 2
-
6 bc b+c 6 bc 2 b+c 2

2
a bc 3
+2 2
- .
bc b+c 2

Bài toán quy v ch ng minh


2
a bc 9
+2 2
.
bc b+c 2

(2.32)
2
a 2bc 1
-3 + 2
- 0
bc b+c 2

bc 2
1- 2
b-c 0.
b+c

2 a2 2
Không m t tính t ng quát, ta gi s a b c . Ta có: c - a b-c .
b2

Ta có:

Nguy n Quang Huy K49 THPT Chuyên ĐHSP


L Ư U H À N H N Ộ I B Ộ | 170

bc 2 bc 2 ca 2
1- 2
b-c 1- 2
b - c + 1- 2
c-a
b+c b+c c+a

2 b2 bc a2 ca
b-c b2 - 2
+ a2 - 2
b+c c+a
b2

2 b2 a2
b-c a 2 + b2 - bc 2 + 2
b+c c+a
=
b2

2 b2 a2
b-c a 2 + b2 - b + c c + a 2
+ 2
b+c c+a
b2

2 b+c c+a
b-c a2 1- + b2 1 -
c+a b+c
= 2
b
2 2
b-c a -b ac
=
b2 b + c c + a

0.

Đ ng th c x y ra khi và ch khi a = b = c = 1 .

Lưu ý 2.19. B t đ ng th c (2.32) trong bài toán trên:

Cho , trong đó không có 2 s nào đ ng th i b ng 0. Khi đó ta có:

2bc 2ca 2ab a 2 + b2 + c2 5


+ + + .
b+c
2
c+a
2
a +b
2
ab + bc + ca 2

Vasile Cirtoaje

Đ ng th c x y ra khi và ch khi a = b = c ho c và các hoán v .

Ví dụ 2.82. Cho th a mãn . Ch ng minh r ng:

a 5 b5 c 5
+ + 3.
b c a

Trần Quốc Anh

Lời giải.

Áp d ng b t đ ng th c Holder ta có:
3
a5 4
a 9b 3 a6 = 81 .
b

Bài toán quy v ch ng minh

a 9b3 3.

Nguy n Quang Huy K49 THPT Chuyên ĐHSP


L Ư U H À N H N Ộ I B Ộ | 171

2
3 a 9b3 a6 .

Đ t . B t đ ng th c tr thành

2
3 x3 y x2 .

(2.33)

T i đây ta có nhi u h ng gi i.

. Do b t đ ng th c là đ ng b c nên không m t tính t ng quát, ta gi s x = 1 . Khi đó ta ch c n


xét b t đ ng th c trong tr ng h p x y z là đ .

Ta có:

x3 y + xy 3 + x3 y - xy 3
x y=
3

x3 y + xy 3 + xyz x - xyz x+ x3 y - xy 3
=
2

x2 yz - xyz x+ x3 y - xy 3
=
2
2
x -2 yz yz - xyz x+ x x2 y - xy 2
=
2
2 2
x yz - 2 yz - xyz x- x x- y
=
2
2 2
x yz - 2 yz - xyz x+ x x- y
= (Do x y z)
2

2 2 2
x yz - 2 yz - xyz x+ x x- y
= .
2

Ta s d ng ph ng pháp

q - 2q 2 - r + q 2 - 4q3 + 2 9q - 2 r - 27r 2
x3 y = = f r .
2

Ta có:

9q - 2 - 27r - q 2 - 4q3 + 2 9q - 2 r - 27 r 2
f r = .
2 q 2 - 4q3 + 2 9q - 2 r - 27 r 2

7 9q - 2 - 1 - 3q 7 1 - 3q
f r =0 r = r0 = .
189

Và 9q - 2 - 27r0 = q 2 - 4q3 + 2 9q - 2 r0 - 27r02 .

V i ta có:

Nguy n Quang Huy K49 THPT Chuyên ĐHSP


L Ư U H À N H N Ộ I B Ộ | 172

q - 2q 2 - r0 + q 2 - 4q3 + 2 9q - 2 r0 - 27r02
f r f r0 =
2

q - 2q 2 - r0 + 9q - 2 - 27 r0
=
2

= - q 2 + 5q + 1 - 14r0

-27 q 2 + 9q + 1 + 2 1 - 3q 7 1 - 3q
= .
27

Bài toán quy v ch ng minh


2
1 - 2q 3 f r0

2
9 1 - 2q -27q 2 + 9q + 1 + 2 1 - 3q 7 1 - 3q

1 - 3q 8 - 21q - 2 7 1 - 3q 0

2
9 1 - 3q 2 - 7q
0 Đúng .
8 - 21q + 2 7 1 - 3q

.Đ t . B t đ ng th c tr thành

f x = a 2 - ab + b2 x 2 + a3 - 5a 2b + 4ab 2 + b3 x + a 4 - 3a 3b + 2a 2b 2 + b 4 0.

Ta có:
2
D f = a3 - 5a 2b + 4ab2 + b3 - 4 a 2 - ab + b2 a 4 - 3a3b + 2a 2b2 + b 4

2
= -3 a3 - a 2b - 2ab2 + b3

0.

B t đ ng th c đ c ch ng minh.

.Đ t .

Ta có:

a2 = x4 + 2 y2 z 2 - 2 y3 z .

bc = y3 z .

Vì a2 bc x4 + 2 y2 z 2 3 y3 z

2
x2 3 y3 z .

. Đ t a = x x - 2 y - z , b = y y - 2z - x , c = z z - 2x - y .

Ta có:

a2 = x4 + 5 y 2 z 2 + 4 xyz x-4 y3 z - 2 yz 3 .

Nguy n Quang Huy K49 THPT Chuyên ĐHSP


L Ư U H À N H N Ộ I B Ộ | 173

bc = 3 y 2 z 2 + 4 xyz x- y3 z - 2 yz 3 .

Vì a2 bc x4 + 2 y2 z 2 3 y3 z

2
x2 3 y3 z .

.Đ t .

Ta có:

a= x2 .

bc = y3 z .

2 2
Vì a 3 bc x2 3 y3 z .

. Ta có các đ ng th c sau:

2 1 2
x2 -3 y3 z = x 2 - y 2 - xy + 2 yz - zx 0.
2

2 1 2
x2 -3 y3 z = 2 x 2 - y 2 - z 2 - 3xy + 3 yz 0.
6

2 1 2 3 2 2 2
x2 -3 y3 z = 2 x 2 - y 2 - z 2 - 3xy + 3 yz + y - z - xy - yz + 2 zx 0.
4 4

Đ ng th c x y ra khi và ch khi a = b = c = 1 .

Lưu ý 2.20. B t đ ng th c (2.33) trong bài toán trên:

Cho . Khi đó ta có:

2
3 x3 y + y 3 z + z 3 x x2 + y 2 + z 2 .

Vasile Cirtoaje

x y z
Đ ng th c x y ra khi và ch khi ho c = = và các hoán v .
4 2 2
2
sin sin sin 2
7 7 7

Đây là 1 bài toán r t khó và là 1 k t qu r t quan tr ng có nhi u ng d ng th ng đ c bi t đ n v i tên


g i “B t đ ng th c Vasile”.

Ví dụ 2.83. Cho , trong đó không có 2 s nào đ ng th i b ng 0 th a mãn a + b + c = 3 và n 2 .


Ch ng minh r ng:

Lời giải.

Áp d ng b t đ ng th c Holder và b t đ ng th c (2.16) ( Ví dụ 2.46) ta có:


n n -1 n -2 n -2
ab n a ab a 2b = ab ab a 2b 9 ab 3n .

Nguy n Quang Huy K49 THPT Chuyên ĐHSP


L Ư U H À N H N Ộ I B Ộ | 174

Đ ng th c x y ra khi và ch khi a = b = c = 1 .

Ví dụ 2.84. Cho , trong đó không có 2 s nào đ ng th i b ng 0. Ch ng minh r ng:

a3 b3 c3 3 a 2 + b2 + c2
+ 2 2+ 2 .
a + b b + c c + a2
2 2
2

Võ Quốc Bá Cẩn

Lời giải.

Áp d ng b t đ ng th c Holder ta có:
2
a3 3 3 3 1 2 3
a2 + c2 a 2 + b2 a 2 a2 + c2 = a 2 + b2 .
a + b2
2
8

Ta c n ch ng minh

2 3 3 3
a 2 + b2 6 a2 a2 + c2 a 2 + b2 .

2 2 2
Đ t x = a 2 + b2 , y = b2 + c 2 , z = c 2 + a 2 . B t đ ng th c tr thành

3
x 3 x xy x .

Áp d ng b t đ ng th c ta có: x 3 x.

Nên ta ch ng minh
5
x 2 3 3 xy x .

Do b t đ ng th c là đ ng b c nên không m t tính t ng quát, ta gi s x =3.

Bài toán quy v ch ng minh

xy x 3 (Ch ng minh Ví dụ 2.83).

Đ ng th c x y ra khi và ch khi a = b = c .

Nguy n Quang Huy K49 THPT Chuyên ĐHSP


L Ư U H À N H N Ộ I B Ộ | 175

Định lý 3.1. Cho , và . Khi đó, ta có:

p +1 p -1 2
x p +1 y p +1 x+ y p x+ y bx - ay
p
+ p - p p
.
a b a+b ab a + b

(3.1)

Chứng minh.

N u , b t đ ng th c hi n nhiên đúng.

Xét tr ng h p .

Ta có:
p +1 p p p
x p +1 y p +1 x+ y x bx a + b + y ay a + b - x + y ab x + y
+ p - p
= p
ap b a +b a pb p a + b

p p p p
x abx + b2 x - abx + aby - y abx + aby - aby + a 2 y
= p
.
a pb p a + b

Không m t tính t ng quát, ta gi s . S d ng h qu đ n gi n c a đ nh lý ,v i


và 0 < u v thì ta có:

pu p-1 v - u vp -up pv p-1 v - u .

Ta thu đ c
p p p p
x abx + b2 x - abx + aby - y abx + aby - aby + a 2 y
p
a pb p a + b

p -1 p -1
xp abx + aby b 2 x - aby - yp abx + aby abx - a 2 y
p
a pb p a + b

p -1 2
p x+ y bx - ay
= p
.
ab a + b

Định lý 3.2. Cho ak > 0, xk 0 ( "k = 1, n ) và p 0 . Khi đó, ta có:

Nguy n Quang Huy K49 THPT Chuyên ĐHSP


L Ư U H À N H N Ộ I B Ộ | 176

n p +1

xk
n
xkp +1 k =1
p
.
k =1 akp n
ak
k =1

(3.2)

Chứng minh.

. Áp d ng b t đ ng th c Holder ta có:

p p +1 p +1
n n
xkp +1 n
x p +1 n
ak p +1 a . k p
k = xk .
k =1 k =1 akp k =1 akp k =1

n p +1

xk
n
xkp +1 k =1
. Đ t dn = - p
.
k =1 akp n
ak
k =1

p +1
x1p +1 x2p +1 x +x
d1 = 0, d 2 = + p - 1 2 p
,...
a1p a2 a1 + a2

Ta s ch ng minh dn 0 v i "n 2 .

Hay ta ch ng minh d n n
là dãy không gi m v i "n 2 .

V i n = 2 b t đ ng th c tr thành
p +1
x1p +1 x2p +1 x1 + x2
+ p p
Đúng theo b t đ ng th c (3.1)).
a1p a2 a1 + a2

V i n > 2 ta có:

n +1 p +1 n p +1

xk xk
n +1
xkp +1 k =1
n
xkp +1 k =1
d n +1 - d n = - p
- - p
k =1 akp n +1
k =1 akp n
ak ak
k =1 k =1

n p +1 n +1 p +1

xk xk
xnp++11 k =1 k =1
= + p
- p
anp+1 n n +1
ak ak
k =1 k =1

0 Đúng theo b t đ ng th c (3.1)).

"n 2 .

B t đ ng th c Trung bình lũy th a suy r ng .

Nguy n Quang Huy K49 THPT Chuyên ĐHSP


L Ư U H À N H N Ộ I B Ộ | 177

Cho b1 , b2 ,..., bn 0 và p1 , p2 ,..., pn > 0 . V i r s > 0 , ta có:

1 1
n r n s
r s
pb k k pb k k
k =1
n
k =1
n
.
pk pk
k =1 k =1

(3.3)

Định lý 3.3. B t đ ng th c Radon ((3.2) t ng đ ng v i b t đ ng th c Trung bình lũy th a suy rông


((3.3)).

(3.4)

Chứng minh.

Áp d ng b t đ ng th c Radon ta có:

r
n s
r s
pb
n n r n pk bks s
k =1
k k
pk bkr = pk bks s = r r
.
k =1 k =1 k =1 -1 n -1
pks s
pk
k =1

1 1
n r n s
r s
pb k k pb k k
k =1
n
k =1
n
.
pk pk
k =1 k =1

xk
Áp d ng b t đ ng th c (3.3) v i pk = ak , bk = ( "k = 1, n ), r = p + 1 ( ), s = 1 ta có:
ak

1
p +1 n

n p +1 xk
1 x
n
k
p
k =1
n
.
k =1 a
ak k
ak
k =1 k =1

n p +1

xk
n
xkp +1 k =1
p
.
k =1 akp n
ak
k =1

Hệ quả 3.1. Cho ak , xk > 0 ( k = 1, n ) và . Khi đó, ta có:

n p +1

n p +1
xk
x k =1
k
p p
.
k =1 a k
n
ak
k =1

(3.5)

Nguy n Quang Huy K49 THPT Chuyên ĐHSP


L Ư U H À N H N Ộ I B Ộ | 178

Chứng minh.

Vì nên .

Áp d ng b t đ ng th c Radon ta có:

n -p n p +1

ak xk
n
xkp +1 n
ak- p k =1 k =1
= - p -1
= .
k =1 akp k =1 xk- p -1 n n p

xk ak
k =1 k =1

Lưu ý 3.1. Cho ( k = 1, n ) và . Khi đó, ta có:

n p +1

xk
n
xkp +1 k =1
p
.
k =1 akp n
ak
k =1

(3.6)

Hệ quả 3.2. Cho xk , bk > 0 ( "k = 1, n ) và . Khi đó, ta có:

n p +1

n
xi
xk k =1
p
.
k =1 bk n 1
p
xbk k
k =1

(3.7)

Chứng minh.
1

Trong (3.2) và (3.5) cho ak = xk bkp ( "k = 1, n . Ta thu đ c (3.7).

Định lý 3.4. Cho ( "k = 1, n ) và . Khi đó, ta có:

n p +1

xk p -1 2
n
xp +1
k =1
xi + x j ai x j - a j xi
k
p p
+ p. max p .
a 1 i< j n
k =1 k
n
ai a j ai + a j
ak
k =1

(3.8)

Chứng minh.

Cho dãy s n n 2 đ c cho b i công th c:

Nguy n Quang Huy K49 THPT Chuyên ĐHSP


L Ư U H À N H N Ộ I B Ộ | 179

n p +1

xk
n
xkp +1 k =1
n = p
- p
.
k =1 ak
n
ak
k =1

Ta có:

n p +1 n +1 p +1

xk xk
xnp++11 k =1 k =1
n +1 - n = + p
- p
0.
a pp+1 n n +1
ak ak
k =1 k =1

D a vào tính ch t đ n đi u này c a dãy n n 2 , suy ra n 2 ( "n 2 ).

p +1 p -1 2
x1p +1 x1p +1 x +x p x1 + x2 a1 x2 - a2 x1
n 2 = + p - 1 2 p p
.
a1p a2 a1 + a2 a1 a2 a1 + a2

Khi đó ta có b t đ ng th c:
p -1 2
p x1 + x2 a1 x2 - a2 x1
n p
a1 a2 a1 + a2

luôn đúng ngay c khi ta thay b a1 , a2 , x1 , x2 b i b b t kì ai , a j , xi , x j ( ). T đó Định lý 3.4


đ c ch ng minh.

N u đ t xk = k ak ( "k = 1, n thì ta thu đ c d ng t ng đ ng c a (3.8).

n p +1

ak p -1 2
n
p +1 k =1
k
ai a j i i a + j aj i - j
ak k p
+ p. max p
.
1 i< j n
k =1
n
ai + a j
ak
k =1

(3.9)

Trong (3.8) n u cho xk = 1 ( "k = 1, n thì ta thu đ c

Hệ quả 3.3. Cho ak > 0 ( "k = 1, n ) và . Khi đó, ta có:

2
n
1 n p +1 p -1
ai - a j
p
- p
p.2 . max p
.
1 i< j n
k =1 ak ai a j ai + a j
n
ak
k =1

(3.10)

n n -1
C ng t t c b t đ ng th c
2
p -1 2
p xi + x j ai x j - a j xi
n p ( )
ai a j ai + a j

ta thu đ c

Nguy n Quang Huy K49 THPT Chuyên ĐHSP


L Ư U H À N H N Ộ I B Ộ | 180

Hệ quả 3.4. Cho ak > 0, xk 0 ( "k = 1, n ) và p 0, n N 2 . Khi đó, ta có:

n p +1

xk p -1 2
n
xkp +1 k =1
xi + x j ai x j - a j xi
- p
p. max p
k =1 akp n 1 i< j n
ai a j ai + a j
ak
k =1

p -1 2
2p xi + x j ai x j - a j xi
.
n n -1 1 i< j n ai a j ai + a j
p

(3.11)
n
Hệ quả 3.5. Cho ( "k = 1, n ) và .Đ t s= xk . Khi đó, ta có:
k =1

p -1 2
n
xk 1 n
p
xi x j xi + x j xi - x j
. + p. max .
k =1 s - xk
p
s p -1 n - 1 1 i< j n
s - xi s - xj xi + x j s - xi2 + x 2j
p

(3.12)

Chứng minh.

Áp d ng b t đ ng th c (3.8) ta có:
n
xk n
xkp +1
p
= p
k =1 s - xk k =1 sxk - xk2

n p +1

xk p -1 2
k =1
xi x j xi + x j xi - x j
p
+ p. max p
1 i< j n
s - xi s - xj xi + x j s - xi2 + x 2j
n
s2 - xk2
k =1

n p +1

xk p -1 2
k =1
xi x j xi + x j xi - x j
p
+ p. max p
1 i< j n
s - xi s - xj xi + x j s - xi2 + x 2j
n 2
1
s - 2
xk
n k =1

p -1 2
1 n
p
xi x j xi + x j xi - x j
= p -1 . + p. max .
s n -1 1 i< j n
s - xi s - xj xi + x j s - xi2 + x 2j
p

Ta có các bài toán sau:


n
Cho xk > 0 ( "k = 1, n . Đ t s = xk . Ch ng minh r ng:
k =1

2
n
xk n xi x j xi - x j
a) + max .
k =1 s - xk n - 1 1 i< j n s - xi s - xj xi + x j s - xi2 + x 2j

Nguy n Quang Huy K49 THPT Chuyên ĐHSP


L Ư U H À N H N Ộ I B Ộ | 181

n
xk n
b) .
k =1 s - xk n -1

Ta th y r ng các Bài toán Mở rộng trên c a b t đ ng th c Nesbitt là h qu c a Hệ quả 3.5.

Định lý 3.5. Cho ak > 0, xk 0 ( "k = 1, n ) và . Khi đó, ta có:

n r

xk
n
xkr 1 k =1
. .
k =1 ak
p
n r - p -1 n p

ak
k =1

(3.13)

Chứng minh.

r
. Ta có: -1 0 .
p +1

Áp d ng b t đ ng th c (3.2) ta có:

r p +1
n p +1

p +1
xk
r k =1
p +1 p +1 n p +1
r n r x k
r
-1 r
n p +1
n
xkp +1 xkp +1 k =1
r
p +1
-1 xk
n r n
x k =1 1 1 k =1
k
p
= p p
= p p
= r - p -1
. p
.
k =1 a a n
n n n n
k k =1 k
ak ak ak ak
k =1 k =1 k =1 k =1

. Ta có 1 b đ (B t đ ng th c Jensen ):

N u cho q1 , q2 ,..., qn > 0 và .

1. N u f x 0 v i "x I thì ta có:


n n
qk f bk qk bk
k =1
n
f k =1
n .
qk qk
k =1 k =1

2. N u f x 0 v i "x I thì ta có:


n n
qk f bk qk bk
k =1
n
f k =1
n
.
qk qk
k =1 k =1

(3.14)

Tr l i bài toán chính.

Đ t r = m +1 . B t đ ng th c tr thành

Nguy n Quang Huy K49 THPT Chuyên ĐHSP


L Ư U H À N H N Ộ I B Ộ | 182

n m +1

xk
n
xkm +1 k =1
n p -m . p
.
k =1 akp n
ak
k =1

(3.15)
n n
Đ t Xn x = xk , An a = ak .
k =1 k =1

B t đ ng th c tr thành
m +1
n
xkm +1 Xn x
n p -m . p
.
k =1 akp An a

Ta có:
m +1
n
xkm +1 n
ak x p +1
= An a tkp +1 , trong đó tk = k ( "k = 1, n ).
k =1 akp k =1 An a ak

Xét f : 0, + R đ c cho b i f x = x p +1 .

Ta có: f x 0v i x 0, + .

n
ak
Vì = 1 , áp d ng b t đ ng th c Jensen ta có:
k =1 An a

p +1 m +1 p +1
n n n
ak ak 1
tkp +1 tk = xkp +1 .
k =1 An a k =1 An a Anp +1 a k =1

m +1

Xét g : 0, + R đ c cho b i g x = x p +1 .

Ta có: g x 0v i x 0, + .

Áp d ng b t đ ng th c Jensen ta có:
m +1
n m +1 n p -m m +1
1 p +1
p +1
x k n xk = n p +1 X np +1 .
k =1 n k =1

n m +1

xk
n
xkm +1 k =1
n p -m . p
.
k =1 akp n
ak
k =1

-1
n
xkr
.Đ t Un x = p
k =1 ak

n -1

Và Hn a = ak .
k =1

Xét f : 0, + R đ c cho b i f x = ln x . Ta có f x là hàm lõm.

Nguy n Quang Huy K49 THPT Chuyên ĐHSP


L Ư U H À N H N Ộ I B Ộ | 183

xkr 1
b1 = Un x q1 =
akp r
p
Áp d ng b t đ ng th c Jensen (3.14) v i n = 3 : b2 = ak H n a và q2 = .
r
b3 = n -1 r - p -1
q3 = 0
r

Ta thu đ c:
r - p -1
1 p
1 xkr p r - p - 1 -1
xk U n x r . Hn a r . n -1 r . U n x + .ak H n a + .n .
r akp r r

r - p -1
n 1 p n
1 xkr p r - p - 1 -1
xk U n x r . Hn a r . n -1 r . p U n x + .ak H n a + .n = 1 .
k =1 k =1 r ak r r

T đây ta thu đ c (3.13).

. Ta s d ng ph ng pháp quy n p toán h c.

V i n = 1 b t đ ng th c hi n nhiên đúng.

Gi s (3.13) đúng v i n = m hay

m r

m r
xk
x 1 k =1
k
. .
k =1 a
p
k mr - p -1 m p

ak
k =1

Ta ch ng minh (3.13) đúng v i n = m + 1 hay

m +1 r

m +1
xk
xkr 1 k =1
p r - p -1
. p
.
k =1 ak m +1 m +1
ak
k =1

m r

xk m
1 k =1
Đ t Rm x = r - p -1
. p
, Sm a = ak .
m m
k =1
ak
k =1

Khi đó ta có:
1
m +1 r - p -1 r - p -1
1 p
xr r p
xk = Rm x r . Sm a r .m r
+ mp+1 . am +1 r .1 r
.
k =1 am +1

Ta có:
m +1
xkr m
xkr xmr +1
p
= p
+ p
k =1 ak k =1 ak am +1

m r

xk
1 k =1 xmr +1
r - p -1
. p
+
m m amp +1
ak
k =1

Nguy n Quang Huy K49 THPT Chuyên ĐHSP


L Ư U H À N H N Ộ I B Ộ | 184

1 r
r - p -1
xr r p
Rm x + mp +1 . S m a + am +1 r . m +1 r
am +1
1
= r - p -1
. p
m +1 Sm a + am +1

1 r
r - p -1
xr r p
Rm x + mp +1 . S m a + am +1 r . m +1 r
am +1
1
= r - p -1
. p
.
m +1 m +1
ak
k =1

Ta c n ch ng minh
1
m +1
xr r p r - p -1
xk Rm x + mp+1 . Sm a + am +1 r . m +1 r .
k =1 am +1

1 1
r - p -1 r - p -1 r - p -1
1 p
xr r p
xr r p
Rm x r . Sm a r .m r
+ mp+1 . am +1 r .1 r
Rm x + mp+1 . Sm a + am +1 r . m +1 r
am+1 am+1

1
r - p -1
1 p r - p -1 xmr +1 r p
r
. am +1 r .1
Rm x r . Sm a r .m r
amp +1
1
+ 1
1.
r p r - p -1 r p r - p -1
x r x r
Rm x + m +1
p
. Sm a + am +1 r . m +1 r Rm x + m +1
p
. Sm a + am +1 r . m +1 r
a
m +1 a m +1

1 p r - p -1
Vì + + = 1 , áp d ng b t đ ng th c AM - GM suy r ng ta có:
r r r
1 p r - p -1
Rm x r . Sm a r .m r
1 Rm x p Sm a r - p -1 m
. + . + .
1
r x r
r S m a + am +1 r m +1
xmr +1 r p r - p -1
Rm x + m +1
Rm x + . Sm a + am +1 r . m +1 r a p
m +1
amp +1
1
r - p -1
.
xmr +1 r p
r xmr +1
. am +1 r .1
amp +1 1 amp +1 p am +1 r - p -1 1
. + . + .
1
r xr r S m a + am +1 r m +1
xr r p r - p -1
Rm x + mp +1
Rm x + m +1
p
. Sm a + am +1 r . m +1 r am +1
am +1

T đây ta thu đ c

m +1 r

m +1
xk
xkr 1 k =1
. .
k =1 ak
p
n r - p -1 m +1 p

ak
k =1

Định lý 3.6. Cho ak , xk > 0 ( "k = 1, n ) và . Khi đó, ta có:

Nguy n Quang Huy K49 THPT Chuyên ĐHSP


L Ư U H À N H N Ộ I B Ộ | 185

n r

xk
n
xkr 1 k =1
. .
k =1 ak
p
n r - p -1 n p

ak
k =1

(3.16)

Chứng minh.

Vì , áp d ng b t đ ng th c (3.13) ta có:

n -p n r

ak xk
n
xkr n
ak- p 1 k =1 1 k =1
= -r
. -r
= . .
k =1 ak
p
k =1 xk n - p - - r -1 n n r - p -1 n p

xk ak
k =1 k =1

Ví dụ 3.1. Cho , và . Ch ng minh r ng:

m - 2 p +1
x m - p +1 y m- p +1 z m- p +1 x+ y+z
+ + .
a 2 x 2 + byz
p
a 2 y 2 + czx
p
a 2 z 2 + dxy
p ap

Lời giải.

Áp d ng b t đ ng th c (3.15) và b t đ ng th c ta có:

x m - p +1 x m+1
= p
p
a 2 x 2 + byz x a 2 x 2 + byz

m +1
x
p
x a 2 x 2 + byz

m +1
x
p
x a2 x3 + b + c + d xyz

m +1
x
p
a x x 3 + 24 xyz

m +1
x
p
3
a x x

m - 2 p +1
x
= .
ap

Nguy n Quang Huy K49 THPT Chuyên ĐHSP


L Ư U H À N H N Ộ I B Ộ | 186

Khi cho ta thu đ c Ví dụ 2.23.

n
Ví dụ 3.2. Cho , n N , x1 , x2 ,..., xn 0 , a, b R và a xk - bxk > 0 ( "k = 1, n ). Ch ng minh r ng:
k =1

n m - 2 p +1

xk
n
xkm - p +1 k =1
p
n2 p -m . p
.
k =1
n
an - b
a xk - bxk
k =1

Lời giải.

Áp d ng b t đ ng th c (3.15) và b t đ ng th c ta có:

n
xkm - p +1 n
xkm +1
p
= p
n n
k =1 k =1
a xk - bxk axk xk - bxk2
k =1 k =1

n m +1

xk
k =1
n p -m . p
n 2 n
a xk -b xk2
k =1 k =1

n m +1

xk
k =1
n p -m . p
n 2

n 2 b xk
k =1
a xk -
k =1 n

n m - 2 p +1

xk
k =1
= n2 p -m . p
.
an - b

Ta có 1 bài toán nh sau:


n
Cho n N , x1 , x2 ,..., xn 0 , a, b R và a xk - bxk > 0 ( "k = 1, n ). Ch ng minh r ng:
k =1

n
xk n
.
n
an - b
k =1
a xk - bxk
k =1

Ta th y r ng Bài toán Mở rộng trên c a b t đ ng th c Nesbitt là h qu c a Ví dụ 3.2 khi cho .

Ví dụ 3.3. Cho , 1, r s > 0 . Ch ng minh r ng:

r
-1
n
xkr 1- n p s
n .
k =1 xls n -1 n
l k

Nguy n Quang Huy K49 THPT Chuyên ĐHSP


L Ư U H À N H N Ộ I B Ộ | 187

Lời giải.

Áp d ng b t đ ng th c (3.15) ta có:

n
xkr n
xkr
n1-
k =1 xls k =1 xls
l k l k

r
n xks s
= n1-
k =1 xls
l k

r
n s
r xks
2-
k =1
n1- n s
.
xls
l k

r
-1
1- n p s
=n .
n -1 n

r
-1
1- n p s
=n .
n -1 n

Ta có 1 bài toán nh sau:

Cho và . Ch ng minh r ng:

r -1
xr xr xr 3 x+ y+z
+ + .
y+z y+z y+z 2 3

Ta th y r ng bài toán này là h qu c a Ví dụ 3.3.


n
Ví dụ 3.4. Cho n N , th a mãn xk = 1 , quy c xn +1 = x1 . , . Ch ng minh
k =1

r ng:
n
xkp n
p p
.
k =1 xkm+-1p +1 axk +1 + bxk a +b

Lời giải.

Áp d ng b t đ ng th c (3.15) và b t đ ng th c AM - GM ta có:
m +1
1
n
xkp n
xk +1
p
= p
k =1 xkm+-1p +1 axk +1 + bxk k =1 a b
+
xk xk +1

Nguy n Quang Huy K49 THPT Chuyên ĐHSP


L Ư U H À N H N Ộ I B Ộ | 188

m +1
n
1
k =1 xk +1
n p -m . p
n
1
a+b
k =1 xk +1

m +1- p
n
1
k =1 xk +1
= n p -m . p
a +b

m +1- p
n
1
nn
k =1 xk +1
n p -m . p
a+b

n
= p
.
a+b

n
Ví dụ 3.5. Cho , th a mãn xk = 1 . Ch ng minh r ng:
k =1

1 n n -1
-1
.
1 i< j n n +1 n - 2
xk xu xv
1 k n 1 u <v n
k i, j u i, j

Lời giải.

Áp d ng b t đ ng th c (3.15) và b t đ ng th c AM - GM ta có:

1 1
-1
= -1
1 i< j n 1 i< j n xk
xk xu xv 1 k n
xu xv - xi x j
1 k n 1 u <v n xi x j 1 u <v n
k i, j u i, j

-1
xi x j
=
1 i< j n
xu xv - xi x j
1 u <v n

-1

3- xi x j
n n -1 1 i< j n
.
2
xu xv - xi x j
1 i< j n 1 u<v n

3- -1 -2
n n -1 n n -1
= . -1 . xi x j
2 2 1 i< j n

3- -1 -2
n n -1 n n -1 n n - 1 n n -1
. -1 . . 2 xi x j
2 2 2 1 i< j n

Nguy n Quang Huy K49 THPT Chuyên ĐHSP


L Ư U H À N H N Ộ I B Ộ | 189

3- -1 -2
n n -1 n n -1 n n -1
= . -1 .
2 2 2

n n -1
= .
n +1 n - 2

Ta có 1 bài toán nh sau:

Cho th a mãn abc = 1 . Ch ng minh r ng:

1 1 1 3
+ 3 + 3 .
a b+c b c+a c a+b
3
2

IMO 1995

Ta th y bài toán này là h qu c a Ví dụ 3.4, 3.5.

Fn
Ví dụ 3.6. Cho n N , Fn là dãy Fibonacci , a > và . Ch ng minh r ng:
n 0
Fn +2 - 1

n
Fkm +1 n p -m m - p +1
p p
. Fn + 2 - 1 .
k =1 a Fn +2 - 1 - Fk an - 1

Lời giải.

Ta có 1 b đ : N u cho dãy Fibonacci Fn n 0


. Khi đó, ta có:

n
Fk = Fn + 2 - 1 .
k =1

Lucas

Ch ng minh b đ trên r t quen thu c, xin nh ng l i cho b n đ c.

Tr l i bài toán chính.

Áp d ng b t đ ng th c (3.15) và b đ trên ta có:

n m +1

n m +1
Fk
F k =1
k
p
n p -m . p
k =1 a Fn +2 - 1 - Fk n
na Fn + 2 - 1 - Fk
k =1

m +1
Fn + 2 - 1
= n p -m . p
na Fn + 2 - 1 - Fn + 2 - 1

n p -m m - p +1
= p
. Fn + 2 - 1 .
an - 1

Định lý 3.7. Cho n N , và . Khi đó, ta có:

Nguy n Quang Huy K49 THPT Chuyên ĐHSP


L Ư U H À N H N Ộ I B Ộ | 190

n m+ p

xk akp -1
n
xkm + p k =1
m + p -1
.
k =1 akm n
p
a k
k =1

(3.17)

Chứng minh.
n n
Đ t Un p = xk akp -1 , An p = akp .
k =1 k =1

. Ta có:
m+ p
n
xkm + p n
akp xk
= An p .
k =1 akm k =1 An p ak

Xét f : 0, + R đ c cho b i f x = x m + p .

Ta có: f x 0v i x 0, + .

akp x
Áp d ng b t đ ng th c Jensen (3.14) v i qk = , bk = k ta thu đ c
An p ak

m+ p m+ p m+ p
n
akp xk n
akp xk Un p
. = .
k =1 An p ak k =1 An p ak An p

n m+ p

xk akp -1
n
xkm + p k =1
m + p -1
.
k =1 akm n
p
a k
k =1

. Ta ch ng minh
m+ p m + p -1
n
xk An p
1.
k =1 Un p akm

Áp d ng b t đ ng th c Bernoulli ta có:
m+ p m + p -1 m+ p
n
xk An p n
akp xk A p
= . n
k =1 Un p akm k =1 An p Un p ak

m+ p
n
akp x A p - akU n p
= 1+ k n
k =1 An p akU n p

n
akp x A p - akU n p
1+ m + p k n
k =1 An p akU n p

n
akp n
xk akp -1 akp
= + m+ p -
k =1 An p k =1 U n p An p

Nguy n Quang Huy K49 THPT Chuyên ĐHSP


L Ư U H À N H N Ộ I B Ộ | 191

N u cho xk = 1 ( "k = 1, n ) ta thu đ c

Hệ quả 3.6. Cho n N , ak > 0 ( "k = 1, n ) và . Khi đó, ta có:

n m+ p

n
akp -1
1 k =1
m + p -1
.
k =1 akm n
p
a k
k =1

(3.18)

N u đ t xk = k ak ( "k = 1, n thì ta thu đ c d ng t ng đ ng c a (3.17).

n n m+ p
m+ p p p
k k a k ak
k =1
n
k =1
n
.
p
ak akp
k =1 k =1

(3.19)

Định lý 3.8. Các b t đ ng th c sau là t ng đ ng:

i. B tđ ng th c Holder .
ii. B tđ ng th c Bernoulli .
iii. B tđ ng th c Trung bình lũy th a suy r ng.
iv. B tđ ng th c Radon .
v. B tđ ng th c (3.17).

Ví dụ 3.7. Cho . Ch ng minh r ng:

2
a b c ab + bc + ca ab + bc + ca
a2 + b2 + c2 .
b+c c+a a+b a 2 + b2 + c 2 2 a 2 + b2 + c2

Lời giải.

Áp d ng b t đ ng th c (3.17) ta có:
5 5 5 5

a a2 a b+c 2 2 bc 2 2 bc 2
a2 = = .
b+c b+c
1 1 1 5

a2 +
2
2
b+c 2
2 bc 2 4 a2 2

Đ ng th c x y ra khi và ch khi a = b = c .

Ví dụ 3.8. Cho th a mãn . Ch ng minh r ng:

3
a19 3 b19 3 c19 19
+ + 6 a 2b 6 + b 2 c 6 + c 2 a 6 .
b c a

Lời giải.

Nguy n Quang Huy K49 THPT Chuyên ĐHSP


L Ư U H À N H N Ộ I B Ộ | 192

Áp d ng b t đ ng th c (3.17) ta có:
1 19
+3
3
a19 a2 6 a 2b6 6 19
= 1 13
= a 2b 6 6 .
b 3 6 3 6
b b

1
Đ ng th c x y ra khi và ch khi a = b = c = 9
.
3

Ví dụ 3.9. Cho . Ch ng minh r ng:

6
a3 b3 c3 a+b+c
+ + .
a + 2b b + 2c c + 2a 81 a 2 + b2 + c 2
2

Lời giải.

Áp d ng b t đ ng th c (3.17) ta có:
3 6 6
a3 a a + 2b a a
= .
a + 2b a + 2b
2 2
5 a2 + 4 ab
2
81 a2
2

Đ ng th c x y ra khi và ch khi a = b = c .

BÀI TẬP

96. Cho th a mãn . Ch ng minh r ng:


4 4 4
b+c c+a a+b 4
+ + a + b + c a 2 + b2 + c 2 - 1 .
a b c
97. Cho th a mãn . Ch ng minh r ng:
1 1 1 27
3
+ 3
+ 3
.
a b+c b c+a c a +b 8
98. Cho a, b, c là đ dài 3 c nh c a 1 tam giác th a mãn . Ch ng minh r ng:
3
a3 b3 c3 2 ab + bc + ca - 1
+ + .
b +c - a c + a -b a +b -c 2 ab + bc + ca - 3
2

99. Cho , a + b > 0 và th a mãn x 2 + y 2 + z 2 = 1 . Ch ng minh r ng:


2 4

x4 y4 z4 2a + b x + y + z -b
2
+ 2
+ 2 3
.
a x + y + z + by a x + y + z + bz a x + y + z + bx 16 3a 2 + 2ab x + y + z
2
+ b2

n n
Định lý 3.9. Cho , xk , ak > 0 ( "k = 1, n . Đ t X n = xk , An = ak . N u sao
k =1 k =1

cho cAns > d max aks ( "k = 1, n ) thì khi đó, ta có:
1 k n

m +1 m +1
n aX np + bxkq xkr m +1 1 anq X np + r + bX nq + r
. .
n m +1 q + r -1 - ms
m m
k =1 cAnp - daks akm cn s - d Anm s +1

Nguy n Quang Huy K49 THPT Chuyên ĐHSP


L Ư U H À N H N Ộ I B Ộ | 193

(3.20)

Chứng minh.
n
Đ t uk = aX np + bxkq xkr , vk = cAnp - daks ak ( "k = 1, n ), Vn = vk .
k =1

Ta có:
m +1 m +1
n aX np + bxkq xkr m+1 n
ukm+1 n
vk uk
m
= m
= Vn .
k =1 cAnp - daks akm k =1 vk k =1 Vn vk

Xét f : 0, + 0, + đ c cho b i f x = x m+1 .

Ta có: f x 0v i x 0, + .

n
vk
Vì = 1 , áp d ng b t đ ng th c Jensen ta có:
V
k =1 n

m +1 m +1 m +1
n n n
vk uk vk uk uk
. = .
V
k =1 n vk k =1 Vn vk k =1 Vn

n m +1

aX np + bxkq xkr
n
ukm +1 k =1
m m
k =1 vk
n
cAnp - daks ak
k =1

n n m +1

aX np xkr + b xkq +r
k =1 k =1
= m
.
n
p +1 s +1
cA n -d a
k
k =1

Xét g , h, k : 0, + 0, + l nl tđ c cho b i g x = xr , h x = x q +r , k x = x s+1 .

Ta có: g x , h x , k x 0v ix 0, + .

Áp d ng b t đ ng th c Jensen ta có:

n
X nr Xr
xkr n. r
= r -n1
k =1 n n
n
X nq + r X nq + r
xkq + r n. = .
k =1 n q +r n q +r -1
n
Ans +1 Ans +1
aks +1 n. = s
k =1 n s +1 n

m +1 m +1
n n
X np + r X q+r
aX np xkr + b xkq + r a + b q +n r -1 m +1
k =1 k =1 n r -1
n 1 an q X np +r + bX nq + r
= . .
n m +1 q + r -1 - ms
m m m
n n
s +1 As +1 cn s - d Anm s +1
cA n
p
ak - d ak cA n
p +1
- d ns
k =1 k =1 n

Nguy n Quang Huy K49 THPT Chuyên ĐHSP


L Ư U H À N H N Ộ I B Ộ | 194

N u cho ta thu đ c (3.2).

Ví dụ 3.10. Cho th a mãn a + b + c = 3 . Ch ng minh r ng:

a b c 1
3
+ 3
+ 3
.
b+2 c+2 a+2 9

Lời giải.

Áp d ng b t đ ng th c (3.2) ta có:
4 4
a a4 a a 1
3
= 3 3 3
= .
b+2 ab + 2a ab + 2 a a
2 9
+2 a
3

Đ ng th c x y ra khi và ch khi a = b = c = 1 .

1 1
Ví dụ 3.11. Cho ( "k = 1, n ) và p, q > 0 th a mãn + = 1 . Ch ng minh r ng:
p +1 q

1
p +1 p +1
1
n n n
xk q
1
ak xk .
k =1 q k =1 k =1
a k

Lời giải.

Ta ch ng minh
p +1
p +1
n n n p +1
xk q

1
ak xk
k =1 k =1 k =1
akq

p +1
p +1
n
xkp +1 n q n

p +1
ak xk
k =1 k =1 k =1
ak q

1
p +1 -1 p +1
n
xkp +1 n p +1 n

1
ak xk
k =1 p +1 -1 k =1 k =1
p +1
ak

p p +1
n
xkp +1 n n
ak xk
k =1 akp k =1 k =1

n p +1

xk
n
xkp +1 k =1
p
Đúng theo b t đ ng th c ).
k =1 akp n
ak
k =1

x1 x2 x
Đ ng th c x y ra khi và ch khi = = ... = n .
a1 a2 an

Nguy n Quang Huy K49 THPT Chuyên ĐHSP


L Ư U H À N H N Ộ I B Ộ | 195

Ví dụ 3.12. Cho m > 0 , ak 0 ( "k = 1, n ) và ak +1 = a1 . Ch ng minh r ng:

2m m +1
n

n
2m akm +1
ak k =1
.
k =1 ak2 + ak ak +1 + ak2+1 n m

9m ak4
k =1

Lời giải.

Áp d ng b t đ ng th c AM - GM và b t đ ng th c (3.2) ta có:

2m m +1

2m akm+1
n n
ak
=
k =1 a + ak ak +1 + ak2+1
2
k k =1 ak4 + ak4+1 + 3ak2 ak2+1 + 2ak ak +1 ak2 + ak2+1
m

2m m +1

n
akm +1
m
k =1 3 4 2
ak4 + ak4+1 + ak + ak4+1 + ak2 + ak2+1
2

2m m +1

n
akm +1
= m
k =1 7 4
ak + ak4+1 + 2ak2 ak2+1
2

2m m +1

akm +1
n

m
k =1 9 4
ak + ak4+1
2

2m m +1
n

m akm +1
2 k =1
m
9 n
ak4 + ak4+1
k =1

2m m +1
n
akm +1
k =1
= m
.
n
9m ak4
k =1

Đ ng th c x y ra khi và ch khi .

BÀI TẬP

Nguy n Quang Huy K49 THPT Chuyên ĐHSP


L Ư U H À N H N Ộ I B Ộ | 196

100. Cho th a mãn x , y 1 . Ch ng minh r ng:

1 1 2
+ .
1 - x2 1 - y 2 1 - xy

101. Cho . Ch ng minh r ng:

a 3 b3 c 3
+ + a+b+c.
bc ca ab

Canada 2002

102. Cho th a mãn ab + bc + cd + da = 1 . Ch ng minh r ng:

a3 b3 c3 d3 1
+ + + .
b+c+ d c +d +a d +a +b a +b+c 3

IMO 1998 Shortlist

Nguy n Quang Huy K49 THPT Chuyên ĐHSP

You might also like